2005-2010(March) Mrcog Part 1 Pastpapers Recall Mrcog Total Guide

220
March 2010 MRCOG part 1 exam MCQ, EMQ, Separate items to revise large scale study was carried out to assess the benefits of a new drug which would prevent fractured neck of femur due to osteoporosois in post menopausal women. the study was carried out correctly using a treatment group and an appropriate control group. the number of women who sustained a fracture neck of femur due to osteoporosis was measured reliably in each group and result are group with fracture without fracture control n1 n3 study n2 n4 calculate the number of women who need to recieve new drug in order to prevent one new fracture of femoral neck due to postmenopausal osteoporosis for the items below n1=50,n2=25,n3=9950,n4=9975 answer 400 Successful lactation is a maintained by oestrogen F bmaintained by progesterone F initiated by LH surge F maintained by HPL F inhibited by dopamine T Following are required for haemostatic clot formation coversion of prothrombin to thrombin T platelet phospholipid T active conversion of plasminogen to plasmin F fibrin degradation products F antithrombin F severe oligohydramnios with limb defects 2.60-70% ASSOSIATED WITH CLEFT LIP/ cleft palate

description

MRCOG

Transcript of 2005-2010(March) Mrcog Part 1 Pastpapers Recall Mrcog Total Guide

Page 1: 2005-2010(March) Mrcog Part 1 Pastpapers Recall Mrcog Total Guide

March 2010 MRCOG part 1 exam

MCQ, EMQ, Separate items to revise

• large scale study was carried out to assess the benefits of a new drug

which would prevent fractured neck of femur due to osteoporosois in post menopausal women. the study was carried out correctly using a treatment group and an appropriate control group. the number of women who sustained a fracture neck of femur due to osteoporosis was measured reliably in each group and result are

group with fracture without fracture control n1 n3 study n2 n4 calculate the number of women who need to recieve new drug in order to prevent one new fracture of femoral neck due to postmenopausal osteoporosis for the items below n1=50,n2=25,n3=9950,n4=9975 answer 400

• Successful lactation is a maintained by oestrogen F bmaintained by progesterone F initiated by LH surge F maintained by HPL F inhibited by dopamine T

• Following are required for haemostatic clot formation coversion of prothrombin to thrombin T platelet phospholipid T active conversion of plasminogen to plasmin F fibrin degradation products F antithrombin F

• severe oligohydramnios with limb defects • 2.60-70% ASSOSIATED WITH CLEFT LIP/ cleft palate

Page 2: 2005-2010(March) Mrcog Part 1 Pastpapers Recall Mrcog Total Guide

• closure of neural groove - 30 days • closure of mid gut - 14 weeks • cobalt - 60 • tecnitium- 99

• The pelvic surface of the sacrum?

- gives origin to the piriform muscles - gives origin to the levator ani muscles - is broader in the male than in the female - trnasmit dorsal rami of sacral nerves - is in contact with the anal canal

• Prolactin.

- release is stimulated by TRH - plasma levels are raised in the first trim of preg. - release is increased by suckling. - maybe produced by decidua. - release is inhibited by metoclopromide.

• The foll disorders and org are correctly paired.

- opthalmia neonatorum: chlamydia trachomatis - chancroid: Haemophilus ducreyi. - sleeping sickness: Leishmania donovani. - ringworm: Trichenella spiralis. - non-specific urethritis: Toxoplasma gondii.

• The heart rate typically increases in response to:

- pain.

Page 3: 2005-2010(March) Mrcog Part 1 Pastpapers Recall Mrcog Total Guide

- hypoxia. - ventilatory expiration - increase in Intracranial pressure - decrease baroreceptor activity

• Antibodies.

- are proteins. - are formed in the fetus before 12 weeks of Intrauterine life. - have an average molecular weight of around 10 000 daltons. - of the rhesus type are genetically transmitted. - are produced by the ribosome of plasma cells.

• The following disorders have an X linked pattern of inheritance.

- G6PD Deficeincy. - Kleinfelter syndrome - adrenogenital syndrome - haemophilia B - familial hypercholestroleamia.

• The following are derived from the urogenital sinus:

- the bladder trigone. - the ureters. - the female urethra. - greater vestibular glands. - paraurethral glands.

• In the fetal CVS - the heart arises from endoderm - the heart is formed by fusion of endocardial tubes. - Cardiac pulsation is present by the 30th day after fertilization.

Page 4: 2005-2010(March) Mrcog Part 1 Pastpapers Recall Mrcog Total Guide

- oxygenated blood is tranferred to the left atrium through the foramen ovale.

- the ductus arteriosus closes during the last 4 weeks of pregnancy.

• Arginine vasopressin - reduces GFR. - controls water loss in the Proximal renal tubule. - is synthesised by the post pituitary gland. - is released in response to rise in plasma osmolality. - is released in response to fall in circulating plasma volume.

• Renin - is secreted by the zona glomerulosa of the adrenal coretx. - is a proteolytic enzyme. - is secreted at an increased rate if the renal perfusion pressure falls. - acts upon circulating angiotensinogen. - is released in response to an increase in extracellular fluid volume

• Actinomyces israelii - is a rickettsia. - forms yellow granules in pus. - is a commensal in the mouth. - is a commensal in the vagina. - is usually resistant to penicillin.

• Diseases caused by spirochaetes include. - Weil's disease. - lymphogranuloma venereum - pinta - Vincent's angina. - bilharzias

• The following are inherited as autosomal recessive conditions:

Page 5: 2005-2010(March) Mrcog Part 1 Pastpapers Recall Mrcog Total Guide

- tuberous sclerosis. - phenylketonuria. - achondroplasia - sickle cell anaemia. - Von gierke's disease.

• Antibodies play an important part in the development of: - phagocytosis. - Mantoux responce. - erythroblastosis fetalis. - hyperemesis gravidarum - anaphylaxis

• chemical mediators concerned in the production of an inflammatory response include:

- 5-hydroxytryptamine - aldosterone. - glucocorticoids. - bradykinin - leukotreines

• The parathyroid gland. - originate from the pharyngeal cleft ectoderm. - secrete parathyroid hormone via the chief( principal) cells. - secrete calcitonin via the oxyphil cells. - may become hyperplastic in the presence of intestinal malabsorption. - may develop adenomas in association with islet cell tumour of the

pancreas

• In the abdominal wall: - the rectus abdominis muscle is attched to the crest of the pubis. - the post border of the external oblique muscle ends in the linea

semilunaris.

Page 6: 2005-2010(March) Mrcog Part 1 Pastpapers Recall Mrcog Total Guide

- the aponeurosis of the external oblique muscle takes part in the formation of the conjoint tendon.

- the inferior epigastric artery is a branch of the internal iliac artery. - the conjoint tendon blends medially with the anterior layer of the

rectus sheath.

• The urogenital sinus in the female gives rise to the following: - ureter - paraurethral glands - Bartholin's gland - urachus - Gartner's duct.

• Concerning sex hormone: - the ovary secretes androstenedione. - The ovary secretes testosterone - The ovary secretes dihydrotestosterone. - SHBG conc. are higher in women more than men - Androgens bound to protein have high biological activity.

• Features of congenital rubella include: - excretion of virus by the neonates. - hepatomegaly - excessive production of growth hormone. - cataract - deafness.

• Clomifene citrate: - is an anti-androgens. - does not stimulate ovulation directly. - can produce visual disturbance. - is genereally prescribed throughtout the proliferative phase of the

menstrual cycle. - in the treatment of ovulation increases the risk of multiple pregnancy.

• The foll are cytotoxic alkylation agents:

Page 7: 2005-2010(March) Mrcog Part 1 Pastpapers Recall Mrcog Total Guide

- Cyclophosphamide. - mercaptopurine - chlorambucil - fluorouracil - methotrexate.

• If a distribution of results is markedly skewed to the left: - the mean is the same as the 50th centile. - the same number of values lie on either side of the median. - the mode is equal to the median. - the student's t test should be used to compare the distribution with

another. - logarithmic transformation of the result will produce a distribution

closer to the normal.

• Corcerning the analysis of clinical trials: - the 95% confidence interval indicates the range within which 19 out of

20 values will lie. - The P value illustrates how often the result would be expected to occur

by chance. - c)b The conventional level of statistical significance is set of P<0.005 - In a randomised trial, there must be equal numbers of results in each

arm of the study. - A relative risk reduction of 60% is significant irrespective of the value

of P.

• The following substance are normally synthesized in the liver: - glucagon - vitamin A - cholesterol. - immunoglobulins - prothrombin

• The pineal gland: - is situated at the anterior end of the 3rd ventricles. - is innervated by the parasymphathetic nervous system. - produces melatonin.

Page 8: 2005-2010(March) Mrcog Part 1 Pastpapers Recall Mrcog Total Guide

- maybe calcified in the adults. - is most active during daylight.

• In congenital adrenal cortical hyperplasia. - The commonest deficiency is C18 hydroxylase. - plasma cortisol concentration is raised. - urinary excretion of 17 oxysteroids is elevated. - dexamethasone will suppress the urinary excretion of 17 oxysteroids. - there is no virilising effects.

• 28)The following are RNA containing virus: - coxsackie - influneza - mumps - herpes simplex. - cytomegalovirus.

• Listeria monocytogenes: - is a gram negative organism. - is sensitive to ampicillin. - may cause a transplacental infection. - is sexually transmitted. - can be cultured from a high vagina swab.

• actinomyces israelii. - is a fungus. - forms yellow granules in pus. - is a mouth commensal - occurs in association with IUCD. - is resistant to penicillin.

• The following drugs may cause enlargement of the fetal thyroid gland:

- methyldopa

Page 9: 2005-2010(March) Mrcog Part 1 Pastpapers Recall Mrcog Total Guide

- thyroxine - carbimazole - propranolol - propylthiouracil

• The following statements about anticoagulant are correct: - Heparin inhibits the action of thrombin - The action of heparin is antagonised by vitamin K - Heparin increases antithrombin III activity. - The effects of anticoagulants are decreased by metronidazole. - Warfarin is greater than 80% protein bound in plasma

• The following drugs and side effects are associated: - methydopa: depression - paracetamol: thromboembolism - indomethacin: peptic ulcer - prednisolone: osteoporosis. - ritodrine: hypoglycemia

• Haematopoiesis in the fetus: - results in nucleated erythrocytes early in development. - occurs in the yolk sac in the first month. - does not occur in the bone marrow until term. - is predominantly hepatic during the 4th month. - does not require folic acid.

• Early blood borne dissemination is characterised feature of: - carcinoma of the endometrium. - osteosarcoma - basal cell carcinoma - carcinoma of the cervix - choriocarcinoma.

• The following cells maybe phagocytic: - neutrophils. - kupffer cells

Page 10: 2005-2010(March) Mrcog Part 1 Pastpapers Recall Mrcog Total Guide

- monocytes - Hofbauer cells. - plasma cell.

• The following are premalignant conditions: - diverticular disease of the large bowel. - ulcerative colitis. - pulmonary asbestosis. - Paget's disease of the bone. - condylomata of the vulva

• In the pituitary gland: - the anterior lobe is smaller than the posterior lobe. - the posterior lobe is ectodermal in origin. - the acidophil cells produces oxytocin. - the basophil cells produce growth hormone. - the blood supply is derived from the internal carotis artery.

• The obturator artery: - branches from the posterior trunk of the internal iliac artery. - passes through the greater sciatic foramen. - is crossed by the ureter. - supplies the hip joint. - may be replaced by a branch of the superior epigastric artery.

• In congenital adrenal hyperplasia: - the commonest cause is a deficiency of 21 hydroxylase. - the plasma cortisol conc is increased. - there may be excessive secretion of 17 alpha hydroxyprogesterone. - sodium retention is characteristic. - blood cathecholamine conc are increased.

• In the fetal lung: - bronchial cartilage formation commences at 18-24 weeks of gestation. - type II alveolar cells first appear at 16-20 weeks gestation.

Page 11: 2005-2010(March) Mrcog Part 1 Pastpapers Recall Mrcog Total Guide

- sphingomyelin is the most common phospholipid present at term - phospholipid release is increased by endogenous adrenaline. - phospholipid production is decreased by exogenous corticosteroids.

• In radiotherapy - 1 gray is equivalent to 1 joule/kg. - the skin usually receives a greater dose of radiation than the

underlying tissues. - the major effect of radiation energy is to damage the cytoplasm of the

cell. - cells in tissues which are hypoxic are more vulnerable to radiation. - Radiation induced changes in tissues may take 6 weeks to develop.

• Concerning the adrenal glands - cortex is derived from neural crest cells. - Zona fasiculata secretes aldosterone. - Cortical adenomas may cause Cushing syndrome. - Neuroblastoma arise in the medulla. - Addison's disease may result from autoimmune destruction of the

cortex.

• The following are autosomal recessive: - neurofibromatosis. - cystic fibrosis. - phenylketonuria - polyposis coli - sickle cell anaemia

• Uterine fibroids: - are defines histologically as fibromyxomas. - arise from endometrial stroma - maybe associated with polycythamia. - predispose to endometrial hyperplasia. - are liable to sarcomatous change in about 5% of cases.

• growth of the foll tumors are hormone dependent:

Page 12: 2005-2010(March) Mrcog Part 1 Pastpapers Recall Mrcog Total Guide

- squamous cell carcinoma of the cervix. - breast adenocarcinoma. - uterine leiomyoma. - prostatic adenocarcinoma - testicular carcinoma

• Surfactants: - is formed mainly in the placenta - levels in amniotic fluid diminish after 33 weeks of gestation - formation can be inferred from the lecithin-sphingomyelin ratio in

amniotic fluid. - contains palmitic acid - decreases the surface tension in pulmonary alveoli.

• Intracellular fluid differs from Extracellular fluids in that: - it forms the major proportion of total body water. - its volume can be measured easily. - it has a higher concentration of potassium than of sodium. - its volume is regulated primarily by the kidneys. - it has a higher phosphate concentration.

• The Anal canal: - has an upper part which is innervated by the inferior hypogastric

plexus. - has a lower part which is supplied by the superior rectal artery - drains lymph to the superficial inguinal nodes from its upperpart. - has its internal sphincter innervated by the infecrior rectal nerve. - has a superficial part of its external sphincter attached to the coccyx.

• The right ovarian artery: - Arises from the abdominal aorta above the renal artery. - passes posterior to the 3rd (horizontal) part of the duodenum - passes post to the genitofemoral nerve. - supplies the right ureter. - anastomoses with the right uterine artery.

Page 13: 2005-2010(March) Mrcog Part 1 Pastpapers Recall Mrcog Total Guide

• Vulva supplied by: - Internal pudendal artery. - Inferior rectal artery. - Genitofemoral artery - obturator artery - femoral artery..

• This is a question outside past papers..All r false Except internal pudendal artery..

• The vagina: - has an anterior wall longer than the post wall. - contains mucus secreting glands in its epithelium. - is related in its lower third to the bladder base. - during reproductive life has an acid pH. - is derived from mesonephric duct.

• The right ureter: - is approx 50cm in length. - ia partly covered by duodenum - crosses the genitofemoral nerve. - enters the bladder anteromedially. - receives part of its blood supply from the uterine artery.

• After birth: - allantois froms median umblical ligament - umbilical vein forms medial umbilical ligament. - umbilical artery forms superior vesical artery. - ductus venosus forms the ligamentum teres. - ductus arteriosus forms the arch of the aorta.

• In spermiogenesis: - primary spermatocytes undergo reduction division. - primary spermatocyte gives rise to 4 spermatids. - whole process of spermatogenesis in man takes 6-7 days.

Page 14: 2005-2010(March) Mrcog Part 1 Pastpapers Recall Mrcog Total Guide

- grossly abnormal spermatozoa may be found in fertile semen. - spermatids are haploid.

• In the human male, dihydrotestosterone: - is a precursor of testosterone. - has one-tenth of the ptency of testosterone. - is responsible for involution of the Mullerain system. - is responsible of the male external genitalia. - binds to an intracellular receptor.

• In Human lactation: - estrogens promote development of breast lobules. - estrogen promotes milk producing effect of prolactin on the brest, - human placental lactogen is essential for milk synthesis. - prolactin stimualtes gonadotrophin release. - oxytocin causes milk ejection,

• Concerning testicular hormones: - testosterone reduces plasma LH conc. - Inhibin stimulates LH production. - Estrogen are formed in the testis. - Testosterone is converted to dihyrotestosterone by 5 alpha reductase. - Testosterone in plasma is predominantly bound to albumin.

• Epidermal growth factor: - is mitogenic. - synthesis is stimulated by estradiol. - is a steroid molecule. - is found in endometrium. - binds to a receptor on the nuclear membrane.

• folic acid: - deficiency causes megaloblastic bone marrow.. - is hydroxycobalamin. - is present in green vegetables.

Page 15: 2005-2010(March) Mrcog Part 1 Pastpapers Recall Mrcog Total Guide

- is predominantly absorbed from the large intestine. - is destroyed by boiling water.

• Doppler Ultrasound: - is used to monitor fetal breathing. - is used in fetal HR monitors. - can be used to measure blood velocity in the fetus. - measure proton relaxation times. - requires injection of contrast agents.

• The following are structural aberration of chromosomes: - deletions. - inversions. - aneuploidy. - polyploidy. - translocation.

• Messenger RNA

- synthesis is dependant on RNA polymerase. - is an exact copy of sense DNA. - contains exons. - is measured by western analysis. - translationoccurs in the nucleus.

• In tumors of the bones: - primary malignancy is more common than 2ndry malignancy. - osteoma rarely present in skull bones. - osteosarcoma is associated with Paget's disease of bone. - lymph node metastases are unusual. - simple bone cysts have a strong tendency to recur.

• Concerning the rectus sheath. - Above the costal margin rectus abdominis lies on the costal cartilages.

Page 16: 2005-2010(March) Mrcog Part 1 Pastpapers Recall Mrcog Total Guide

- Below the arcuate line the internal oblique splits to enclose rectus abdominis.

- It contains the musculophrenic artery. - It is innervated by the ilioinguinal nerve - Pyramidalis is innervated by the subcostal nerve.

• The external iliac artery. - Enters the thigh anterior to the inguinal ligament. - at its origin is crossed by the ureter. - at its origin is crossed by the ovarian vessels. - Lies medial to the external iliac vein at its distal end. - gives rise to the deep external pudendal artery.

• Action of Insulin include stimulation of: - glycogenolysis by the liver. - cellular uptake of amino acids. - entry of glucose into neurons. - d)entry of glucose into adipose tissue. - cellular uptake of potassium.

• Metronidazole: - is effective against Giardia lamblia. - is effective when administered per rectum - should not be administered intravenously. - is usually effective against Entamoeba histolytica. - interferes with ethanol metabolism.

• Amniotic fluid: - at term is hyperosmolar compared to fetal plasma. - normally contains maternal and fetal cells. - contains a higher concentration of alpha feto protein at 16 weeks than

at term. - contains bilirubin. - contains phospholipids.

• Chromosomes:

Page 17: 2005-2010(March) Mrcog Part 1 Pastpapers Recall Mrcog Total Guide

- are found in the same numb in all mammalian cells. - can be analysed more quickly from a blood sample than from an

amniotic fluid sample. - can be reliably identified by their length. - the Y chromosome is larger than the X chromosome. - DNA content is doubled during the S phase of the cell cycle.

• In cystic fibrosis, abnormalities are seen in the - pancreas - salivary glands. - brain - kidneys - ileum

• Neutrophil polymorphs at the site of inflammation are capable of the following:

- Phagocytosis. - production of oxygen free radicals. - replication - fusion to form giant cells. - antibody production

• In the statistical analysis of any group of numericals - the mean is always less than the mode - standard deviation is always greater than the standard error - the median value lies at the midpoint of the range - the standard error of the mean is independent of the total

number of observations - there are the same number of observations greater than and

less than the median value

• In a randomised double blind trial comparing a new drug with a placebo

- the patient will be taking either of two drugs - patients can choose their mode of treatment - doctors prescribing treatment decide which patients take the

new drug

Page 18: 2005-2010(March) Mrcog Part 1 Pastpapers Recall Mrcog Total Guide

- a large trial is more likely to give a statistically significant result than a small trial

- half of the patients will take the new drug

• In a trial of oral hypoglycaemic agents 42 patients were given drug A and 38 drug B . blood glucose conc. were measured befor and after a single dose of the drug. Drug B apparently caused greater fall in the blood glucose conc. (P=0.06)

- these results reach an accepted level of statistical significants - Non- parametric statistical analysis bshould be used if datd are

not normally distributed - In biological terms drugs A and B have been shown to be eqully

effective - 6% more patients responded to drug A than drug B - Unequal numbers in the two groups invalidate the trial

• The following statistical statements are correct

- In the normal distribution the value of the mode is 1.73 * that of the median

- In a distribution skewed to the right, the mean lies to the left to the median

- In the series 2;7;5;2;3;2;5;8, the mode is 2 - Student's test is designed to correct for skewed distributions - the Chi-squared test may be used when data are not normally

distributed

• pudendal nerve, nerve for perianal area, down syndrome, patau syndrome, cobalt, technitium, dna virus, HIV virus, syphillis, heptitis b and c virus, PG I2, ionic exchange assosiated with calcium and potassium, positive predictive value, negative predictive value, aspirin,mefanamic acid, insulin secretion and glucagon sec inhibited and stimulated by what substances

• PUDENDAL NERVE ROOT VALUE S1,2,3 ANT PRIMARY RAMI • N SUPPLY OF PERIANAL AREA • CLOSURE OF NEURAL GROOVE ON -WK OF IUL • LENGTH OF UMBILICAL CORD

Page 19: 2005-2010(March) Mrcog Part 1 Pastpapers Recall Mrcog Total Guide

• AUTOSOMAL RECESSIVE TAY SACH'S DISEASE • AUTOSOMAL DOMINANT ACHONDROPLASIA • ANTIINFLAMMATORY MUSCLE RELAXANT PROSTAGLANDIN • STAPH AUREUS COAGULASE POSITIVE, PIGMENTED COLONY,GRAM

POSITIVE • GENTAMYCIN BACTERICIDAL,TOXIC TO 8 TH NERVE • DIURETIC ACTING ON DCT BENDROFLUAZIDE • Successful lactation is - A- maintained by oestrogen F - B-maintained by progesterone F - c -initiated by LH surge F - d- maintained by HPL F - e -inhibited by dopamine T

• The following are required for haemostatic clot formation - a -coversion of prothrombin to thrombin T - b -platelet phospholipid T - c -active conversion of plasminogen to plasmin F - d -fibrin degradation products F - e -antithrombin F

• the following statements about vitamins are true - vit k is water soluble - vit D is poorly absorbed in cases of obstructive jaundice - vit A is a fat soluble vit. - vitamins supply the body wd energy - vit D is bound to transport protein in the circulation.

• ANSWERS - a.F- fat soluble - b.T- also in cases of pancreatic disease where there is pancreatic lipase

deficiency. - c.T- - d.F- vitamins r organic dietry constituents necessary for life haelth n

growth that do not function by supplying the body wd energy. - e- T- physiologically active form of vitamin D is known as calcitriol.

Page 20: 2005-2010(March) Mrcog Part 1 Pastpapers Recall Mrcog Total Guide

- calcitriol is released into the circulation, and by binding to a carrier protein in the plasma, vitamin D binding protein (VDBP), it is transported to various target organs.

• VITAMIN B12 - a fat soluble vit - b-absorption takes place throughout the small intestine - c-is essentail for the metabolism of folic acid in human body. - d-deficiency leads to macrocytic anemia - e-deficiency is common in strict vegetarian.

• ANSWERS - a-F- water soluble - b-F-absorbed mainly in lower ileum aided by gastric intrinsic factor. - C-T - d-T - e-T- source is mainly from animal food stuff, vegetables alone are

inadequate source.

• FOLIC ACID - a-water soluble - b-requires gastric intrinsic factor - c-is necessry for nucleic acid synthesis - d-heat stable - e-involved in krebs cycle

• ANSWERS - a-T - b-F - c-T-tetrahydrofolate is essentaial for both purine and pyramidine

synthesis. - d-F- the normal western diet contains 500-700 microgm/day of wwch

10-100% lost in cooking. - e-F

Page 21: 2005-2010(March) Mrcog Part 1 Pastpapers Recall Mrcog Total Guide

• Folic acid (also known as vitamin B9 or folacin) and folate (the naturally occurring form), as well as pteroyl-L-glutamic acid and pteroyl-L-glutamate, are forms of the water-soluble vitamin B9.

• Folic acid is itself not biologically active, but its biological importance is due to tetrahydrofolate and other derivatives after its conversion to dihydrofolic acid in the liver.

• Vitamin B9 (folic acid and folate inclusive) is essential to numerous bodily functions ranging from nucleotide biosynthesis to the remethylation of homocysteine.

• The human body needs folate to synthesize DNA, repair DNA, and methylate DNA as well as to act as a cofactor in biological reactions involving folate.

• It is especially important during periods of rapid cell division and growth.

• Both children and adults require folic acid to produce healthy red blood cells and prevent anemia.

• Leafy vegetables such as spinach, asparagus, turnip greens, romaine lettuces, dried or fresh beans and peas, fortified grain products (pasta, cereal, bread), sunflower seeds and certain other fruits (orange juice, canned pineapple juice, cantaloupe, honeydew melon, grapefruit juice, banana, raspberry, grapefruit, strawberry) and vegetables (beets, broccoli, corn, tomato juice, vegetable juice, brussels sprouts, bok choy) are rich sources of folate.

Page 22: 2005-2010(March) Mrcog Part 1 Pastpapers Recall Mrcog Total Guide

• Liver and liver products also contain high amounts of folate, as does baker's yeast.

• Some breakfast cereals (ready-to-eat and others) are fortified with 25% to 100% of the recommended dietary allowance (RDA) for folic acid.

• Folic acid naturally found in food is susceptible to high heat, UV, and is soluble in water.

• Folic acid is an important nutrient for women who may become pregnant, because a woman's blood levels of folate fall during pregnancy due to an increased maternal RBC synthesis in the first half of the pregnancy and fetal demands in the second half.

• The first four weeks of pregnancy (when most women do not even realize they are pregnant) require folic acid for proper development of the brain, skull, and spinal cord.

• Serious birth defects like neural tube defects are less likely to occur when women take 0.4 mg of folic acid daily.

• Neural tube defects (NTDs) result in malformations of the spine (spina bifida), skull, and brain (anencephaly).

• The risk of neural tube defects is significantly reduced when supplemental folic acid is consumed in addition to a healthy diet prior to and during the first month following conception.

• Folate deficiency during pregnancy can increase the risk of preterm delivery, infant low birth weight, and fetal growth retardation.

Page 23: 2005-2010(March) Mrcog Part 1 Pastpapers Recall Mrcog Total Guide

• Folate deficiency in the mother increases homocysteine level in the blood which may lead to spontaneous abortion and pregnancy complications such as placental abruption and preeclampsia.Folic acid appears to reduce the risk of stroke.

• Folic acid supplements help relieve hot flushes in postmenopausal women. Just like in estrogen hormone replacement therapy, folic acid interacts with neurotransmitters (norepinephrine, serotonin) in the brain to reduce hot flushes.

• IN SHORT: Folate deficiency may lead to glossitis, diarrhea, depression, confusion, anemia, and fetal neural tube defects and brain defects (during pregnancy).

• VITAMIN C - a-is found only in animal food stuffs. - b-is rapidly destroyed by heating. - c-there are normally large stores in pancreas. - d-impared wound healing is one of the characterstic features of severe

vit c defeciency. - e-exess vit c can lead to formation of oxalate stones in urinary tract.

• ANSWERS - a-F. citrus foods n leafy green veges r rich in vit c while animal sources

contain only traces. - b-T - c-F-eye n adrenal glands contain large quantities of vit c. - d-T - e-T

• Vitamin C or L-ascorbic acid is an essential nutrient for humans, in which it functions as a vitamin.

Page 24: 2005-2010(March) Mrcog Part 1 Pastpapers Recall Mrcog Total Guide

• Ascorbate (an ion of ascorbic acid) is required for a range of essential metabolic reactions in all animals and plants.

• Deficiency in this vitamin causes the disease scurvy in humans.

• Ascorbic acid is absorbed in the body by both active transport and simple diffusion.

• Sodium Dependent Active Transport - Sodium-Ascorbate Co-Transporters (SVCTs) and Hexose transporters (GLUTs) are the two transporters required for absorption.

• Although the body's maximal store of vitamin C is largely determined by the renal threshold for blood, there are many tissues which maintain vitamin C concentrations far higher than in blood.

• Biological tissues that accumulate over 100 times the level in blood plasma of vitamin C are the adrenal glands, pituitary, thymus, corpus luteum, and retina.

• Those with 10 to 50 times the concentration present in blood plasma include the brain, spleen, lung, testicle, lymph nodes, liver, thyroid, small intestinal mucosa, leukocytes, pancreas, kidney and salivary glands.

• Scurvy is an avitaminosis resulting from lack of vitamin C, since without this vitamin, the synthesised collagen is too unstable to perform its function.

• Scurvy leads to the formation of liver spots on the skin, spongy gums, and bleeding from all mucous membranes.

• The spots are most abundant on the pages and legs, and a person with the ailment looks pale, feels depressed, and is partially immobilized.

Page 25: 2005-2010(March) Mrcog Part 1 Pastpapers Recall Mrcog Total Guide

• In advanced scurvy there are open, suppurating wounds and loss of teeth and, eventually, death.

• The human body can store only a certain amount of vitamin C, and so the body soon depletes itself if fresh supplies are not consumed.

• In humans, vitamin C is essential to a healthy diet as well as being a highly effective antioxidant, acting to lessen oxidative stress; a substrate for ascorbate peroxidase;and an enzyme cofactor for the biosynthesis of many important biochemicals.

• Vitamin C acts as an electron donor for important enzymes:[ • Vitamin C acts as an electron donor for eight different enzymes:

• Three participate in collagen hydroxylation. • These reactions add hydroxyl groups to the amino acids proline or

lysine in the collagen molecule via prolyl hydroxylase and lysyl hydroxylase, both requiring vitamin C as a cofactor.

• Hydroxylation allows the collagen molecule to assume its triple helix structure and making vitamin C essential to the development and maintenance of scar tissue, blood vessels, and cartilage.

• are necessary for synthesis of carnitine. • Carnitine is essential for the transport of fatty acids into mitochondria

for ATP generation.

• The remaining three have the following functions in common but do not always do this:

• -dopamine beta hydroxylase participates in the biosynthesis of norepinephrine from dopamine.

Page 26: 2005-2010(March) Mrcog Part 1 Pastpapers Recall Mrcog Total Guide

• -another enzyme adds amide groups to peptide hormones, greatly increasing their stability.

• -one modulates tyrosine metabolism.

• DAILY REQUIREMENTS:

• 75 milligrams per day: the United Kingdom's Food Standards Agency

• 45 milligrams per day: the World Health Organization

• VITAMIN B. - a.B1(thiamin)deficiency leads to impaired collagen formation - b.B1 stores in the body are adequate for upto 9 months - c.B2(riboflavin) concentration is higher in fetus than in mother - d.B6(pyridoxine)requirement in pregnancy is 25mg/day - e.niacin is synthesised in the body from tryptophan.

• ANSWERS - a-T- - b-F- the body contains only 30mg(average adult requirement is 1-1.5) - c-T- as with other water soluble vit, vit B crosses the placenta by active

mechanisms, wch results in higher concentration in fetus - d.F- the true figure is 2.5mg/day an din non pregnant adult is

2mg/day. - e-T

• VITAMIN A - a.requires bile for its absorption. - b.deficiency leads to night blindness.

Page 27: 2005-2010(March) Mrcog Part 1 Pastpapers Recall Mrcog Total Guide

- c.excess leads to xerophthalmia - d.stored in liver - e.daily dietry requirement during pregnancy is 50mg/day

• ANSWERS - a.F - b.T - c.F- xerophthalmia is due to its deficiency. hypervitaminosis A is

charachterised by anorexia , headache, hepatosplenomegaly,patchy loss of hair and hyperostosis.

- d.T - e.F-during pregnancy the requirement is 1000 - microgm/day.

• Vitamin A is a vitamin which is needed by the retina of the eye in the form of a specific metabolite, the light-absorbing molecule retinal.

• This molecule is absolutely necessary for both scotopic and color vision.

• Vitamin A also functions in a very different role, as an irreversibly oxidized form retinoic acid, which is an important hormone-like growth factor for epithelial and other cells.

• Vitamin A can be found in two principal forms in foods:

• -retinol • -carotenes • Vitamin A is found naturally in many foods:

• liver (beef, pork, chicken, turkey, fish)

Page 28: 2005-2010(March) Mrcog Part 1 Pastpapers Recall Mrcog Total Guide

• carrot • broccoli leaf • sweet potato • kale • butter • spinach • pumpkin • collard greens • cantaloupe melon • egg • apricot • papaya • mango • pea • broccoli

• Vitamin A plays a role in a variety of functions throughout the body, such as:

• Vision • Gene transcription • Immune function • Embryonic development and reproduction • Bone metabolism • Haematopoiesis • Skin health • Antioxidant Activity

• DEFICIENCY leads to • -night blindness

• -xerophthalmia. • First there is dryness of the conjunctiva (xerosis) as the normal

lacrimal and mucus secreting epithelium is replaced by a keratinized epithelium.

Page 29: 2005-2010(March) Mrcog Part 1 Pastpapers Recall Mrcog Total Guide

• This is followed by the build-up of keratin debris in small opaque plaques (Bitot's spots) and, eventually, erosion of the roughened corneal surface with softening and destruction of the cornea (keratomalacia) and total blindness.

• Other changes include impaired immunity, hypokeratosis (white lumps at hair follicles), keratosis pilaris and squamous metaplasia of the epithelium lining the upper respiratory passages and urinary bladder to a keratinized epithelium.

• With relations to dentistry, a deficiency in Vitamin A leads to enamel hypoplasia.

• Adequate supply of Vitamin A is especially important for pregnant and breastfeeding women, since deficiencies cannot be compensated by postnatal supplementation.

• However, excess Vitamin A, especially through vitamin supplementation, can cause birth defects and should not exceed recommended daily values.

• VITAMIN E - a.present in animal food stuffs only. - b.deficiency may cause intrauterine fetal death - c.potentiates the action of coumarin anticoagulants - d.used in the treatment of infertility - e.dietry requirement is 10mg/day

• ANSWERS - a.F-present in most foods - b.F-proven only in animals and not in humans - c.T

Page 30: 2005-2010(March) Mrcog Part 1 Pastpapers Recall Mrcog Total Guide

- d.F-there is no evidence that vit e increases virility, or plays any role in treatment of infertility or recurrent abortion.

- e.T

• VIT D - a.water soluble - b.is stored in body fat - c.is absorbed from large intestine - d.deficiency leads to rickets - e.dietry req is 10mg/day

• ANSWERS - a.F - b.T - c.F-it is absorbed from small intestine - d.T - e.T

• VITAMIN K - a.mainly found in green leafy veges - b.in adults no external supplements necessary - c.hypervitaminosis is charactererised by anemia - d.exists in 2 forms k1 n k2 - e.is stored in large quantities in the liver

• ANSWERS - a.T - b.T-except in pregnant pts on anti epilactics who require vit k in last

months of pregnancy and in new borns. - c.T - d.T - e-Falthough accumulates in liver initially but its hepatic concentration

declines rapidly.

• REGARDING metabolism - a.the metabolic rate is the amount of energy liberated per unit of time

Page 31: 2005-2010(March) Mrcog Part 1 Pastpapers Recall Mrcog Total Guide

- b.anabolism is defined as the formation of substances which can store the energy.

- c.basal metabolic rate is defined as the metabolic rate determined at rest in a room at 12-14hrs after the last meal.

- d.the BMR of a man is about 500kcal/day - e.the metabolic rate is decreased after consumption of a meal that is

rich in protein. • ANSWERS - a.T - b.T - c.T - d.F-2000kcal/day - e.F-the metabolic rate is increased after consumption of a meal rich in

protein or fat.

• ANABOLISM - the set of metabolic pathways that construct molecules from smaller

units. - These reactions require energy.

• BASAL METABOLIC RATE • The primary organ responsible for regulating metabolism is the

hypothalamus. • REGARDING METABOLISM - a.oxidation is the comination of a substance with either oxygen or

hydrogen. - b.Co-factors r essential for certain enzyme reactions. - c.A co-enzyme is a protein substance wch acts as a carrier for products

of reaction. - d.Co-enzyme A is a high energy compound wch is formed from

adenine,ribose pentotothenic acid and thioethanol amine. - E.a calorie is defined as the amount of heat energy needed to

raise the temperature of 1gm of water by 1 degree ,from 15 to 16degreesC.

Page 32: 2005-2010(March) Mrcog Part 1 Pastpapers Recall Mrcog Total Guide

• ANSWERS • a.F-oxidation is the combination of a substance with oxygen or loss of

hydrogen or loss of an electron. reduction is the reverse of this. • b.T • c.F-it is an organic non-protein substance. • d.T • e.T

• ENZYMES

- a.are proteins - b.heating usually results in a complete loss of enzyme activity - c.a change in pH has no effect on the activity of an ezyme. - d.are present in all cell organelles. - e.organic solvents will usually destroy an enzymes activity.

• ANSWERS - a.T - b.T - c.F-a change in pH has an effect on enzyme activity. - d.T - e.T

• PROTEIN METABOLISM - a.proteins contain abt 40%nitrogen - b.chains containing>100 amino acid residues are called proteins - c.proteins yield 4 calories /gm absorbed. - d.during pregnancy there is a rise in the plasma concentration of

triglycerides. - e.during pregnancy there is a rise in the plasma concentration of

albumin.

• ANSWERS - a.F - b.T - c.F - d.T

Page 33: 2005-2010(March) Mrcog Part 1 Pastpapers Recall Mrcog Total Guide

- e.F

EMQ: • ant division of ant. remi of S2-4 • ant division of post.rami of S2-4 • genitofemoral nerve • post division of ant. remi of S2-4 • post division of post. remi of S2-4 • sensory supply of perineal nerve • inferior rectal nerve • ischial tuberosity • ischial spine

• A lady is about to deliver and you are about to give her a pudendal block..

• Q1- what is the root value of pudendal nerve? • Q2- what nerve supplies lower part of vagina (I think)? • Q3- why do u give local skin infiltration before episiotomy? • Q4- or what is the nerve supply of peranal area .?

• (2).. If we take the day of fertilization as Day’0’ then… • Day2 • Day4 • Day 8 • Day 10 • Day12 • Day 14 • Day 18 • Day20 • Day 22 • Day 24 • Day 26 • Day 42 [

Page 34: 2005-2010(March) Mrcog Part 1 Pastpapers Recall Mrcog Total Guide

• Day 70 • Q1- Which represents 4 cell stage ? • Q2- conceptus implants completely? • Q3- fetal heart pulse seen on ultrasound? • Q4- vertebra form completely?

• A Cytoplasmic transcription factor receptor • B G protein coupled receptor on cell membrane • C G protein coupled receptor on Golgi complex • D Mega subunit ligand gated ion channel • E Multisubunit antibody receptor on cell membrane • F Multisubunit ligand gated ion channel on cell membrane • G Nuclear protein kinase receptor • H Nuclear transcription factor receptor • I Protein kinase receptor on cell membrane • J Receptor protein complex (intracytoplasmic) • K Transcription factor receptor on cell membrane

• Select the receptor which binds the molecules referred to in the itemS below. Each option may be used once, more than once or not at all.

- insulin . - progesterone . - ostradiol . - prostaglandin .

• (4) • PCR • Fish test • Chromosomal linkage analysis • Telomer analysis • Sum hereditary crosslinkage chromosomal option (dnt remember)

Page 35: 2005-2010(March) Mrcog Part 1 Pastpapers Recall Mrcog Total Guide

• Screen for some known gene mutation • Sweat test • Saliva test

• -A family with one kid presumed to have cystic fibrosis however cftr gene mutation was not detected. Family members willing to give samples if needed.mother wants definite diagnosis as she is 11wks and wants to know diagnosis for her baby.parents are cousins with 1st degree relatives having cystic fibrosis.

• Invasive diagnostics can be applied to reach final results by?

• you have Guthrie spot of their son how will u diagnose him for cystic fibrosis?

• families in UK with cystic fibrosis how will u find f508?

• (5) o Endoplasmic reticulum o Golgi apparatus o Lysosomes o Microtubules o Mitochondria o Nucleolus o Plasma membrane o Ribosomes

• Select the most appropriate organelle that matches the following descriptions

Contains enzymes capable of digesting cells and cellular material

Page 36: 2005-2010(March) Mrcog Part 1 Pastpapers Recall Mrcog Total Guide

'Reads' the mRNA and builds protein . Modification of lipids and proteins with storage of

material prior to export out of the cell.

• (6)[i][b] screening test for following syphilis treponemal antibody test syphilis hemaglutination test syphilis immobilization test HB-electrophorisis mcv mchc MCh serum iron paul-bennel test[/b][/i]

• alpha thalessemia • folic acid deficiency • iron deficiency anemia • primary syphilis in early untreated period

• (7) on taking a large protein meal? • Decrease decrease • Increase increase • Increase decrease • Deacrease increase • Nochange no change • Nochange increase • Nochange decrease

• Insulin & Glucagon excretion what will happened ...?

• (8)

Page 37: 2005-2010(March) Mrcog Part 1 Pastpapers Recall Mrcog Total Guide

o 3-hydroxyisovaleric acid . o 17-alpha hydroxyprogesterone . o 17-delta hydroxyprogesterone . o acetic acid . o cortisol .

• The following level of certain metabolities in amniotic fluid changes significantly of the fetus has an inborn error of the metabolism .select the single metabolite from list of the options above whose level in amniotic fluid is altered by inheretiance disease in item below.

• congentiat adrenal hyperplasia

• (9) Adrenaline Calcitonin

• C Cholecystokinin Cortisol Glucagon Insulin Progesterone Somatostatin

• IT- estosterone

• Select the appropriate hormone from the list for the following structures that produce it. Each answer may be used once, more than once, or not at all.

Adrenal Cortex . Adrenal Medulla . Pancreatic alpha cell . Pancreatic D Cells .

Page 38: 2005-2010(March) Mrcog Part 1 Pastpapers Recall Mrcog Total Guide

• (10) • AAnterior cerebral artery • BAnterior communicating artery • CAnterior inferior cerebellar artery • DBasilar artery • EInternal auditory artery • FInternal carotid artery • GMiddle cerebral artery • HPosterior cerebral artery • IPosterior communicating artery • JSuperior cerebellar artery • KVertebral artery • Complete the diagram of the circle of Willis using the options given: • this question came with diagrame in september 2007 • so you should study circle of weils very well if I get the pictures I will

post it .

• Akeratinising stratified squamous epithelium • Bnon-keratinising stratified squamous epithelium • Cpseudostratified columnar epithelium • Dsimple columnar epithelium • Esimple cuboidal epithelium • Ftransitional epithelium

• Which of the epithelial types described above are present in the anatomical regions below:

ectocervix . endocervical canal . cervical ectropion . transformation zone of cervix .

Page 39: 2005-2010(March) Mrcog Part 1 Pastpapers Recall Mrcog Total Guide

vagina . uterus . Bladder . Trachea . Labia Majora . Bowel .

• 11-Fallopian tube . Vulva . anal canal .

• Autosomal co-dominant • Autosomal dominant • Autosomal recessive • Polygenic • Single gene defect • X linked dominant • X linked recessive

• [Select the most likely mode of inheritance for the following patients’ conditions:

- A 27 -year old female developed gestational diabetes mellitus. Her uncle and grandmother also had diabetes mellitus2- A mothr is concerned regarding her baby who has developed fractures which appear to occur with minimal trauma. He has blue sclera.

- An 18-year old female underwent caries tooth extraction and developed profuse bleeding. On history she revealed menorrhagia. Her mother and her grandfather had the same disease.

Page 40: 2005-2010(March) Mrcog Part 1 Pastpapers Recall Mrcog Total Guide

WARFARIN . HEPARIN . AMPICILLIN . METHYLDOPA . PENCILLIN . CARBIMAZEPINE . METRONIDAZOLE . NON OF THE ABOVE . ALL OF THE ABOVE .

• For each of the following choose the single most appropirate statement from the above list of options .

- Drug contraindicated in breast feeding . - Drug does not cross the placenta .

• A. • Calcitonin

• B. • Cortisol

• C. • Glucagon

• D. • Growth hormone

• E. • Insulin

• F. • Oestradiol 17-¦Â¦Â

• G.

Page 41: 2005-2010(March) Mrcog Part 1 Pastpapers Recall Mrcog Total Guide

• Oxytocin

• H. • Parathyroid hormone

• I. • Prolactin

• J. • Thyroxine

• Instructions: For each action described below, choose the single most likely causative hormone from the above list of options. Each option may be used once, more than once, or not at all.

• Question 3: Stimulates deposition of cartilage at the ends of bones • Question 4: Raises blood glucose levels through the breakdown of fat

and protein • Question 5: Antagonises the effect of parathyroid hormone to minimise

bone density loss • Question 6: Stimulates the release of milk from the breast • Question 7: Stimulates the urinary secretion of calcium

• Options • A. • Common iliac artery • B. • External iliac artery • C. • Inferior epigastric artery • D. • Inferior vesical artery • E.

Page 42: 2005-2010(March) Mrcog Part 1 Pastpapers Recall Mrcog Total Guide

• Internal iliac artery • F. • Middle rectal artery • G. • Ovarian artery • H. • Superior vesical artery • I. • Umbilical artery • J. • Uterine artery • K. • Vaginal artery

• Instructions: After a forceps delivery a 30-year-old primigravida sustains a primary post partum haemorrhage of four litres. Although the uterus appears well contracted the bleeding continues, and a decision is made to identify and treat the bleeding point radiologically. For each question posed below, choose the single most appropriate option from the above list. Each option may be used once, more than once or not at all.

• Question 8: What vessel runs up the broad ligament? • Question 9: Which other vessel arises from the uterine artery? • Question 10: Which other vessel anastomoses with the uterine artery?

Question 11: Which vessel does the uterine artery arise from? • Question 12: Which vessel runs anterior and superior to the ureter?

- vitamin A • B- vitamin E • C- vitamin D

• vitamin B12 • vitamin B6 • vitamin C • folic acid

Page 43: 2005-2010(March) Mrcog Part 1 Pastpapers Recall Mrcog Total Guide

• riblflavine • vitamin K • thiamine

- fat soluble vitamin sysnthesised in the intestinal wall from the beta carotene .

- fat soluble vitamin synthesised by large intestinal bacteria .

- deficiency of this vitamin in childhood causes rickets .

- fat soluble vitamin synthesis by the kidney is regulated by parathyroid hormone .

- deficiency of this vitamins typicaly occurs in women with hyperemesis gravidarum .

- water soluble vitamin with anti-oxidant effects .

- absorption of this vitamin requires the presence of intrinsic factor .

- deficiency of this vitamin causes megaloblastic anaemia and neurological disorders .

- fat soluble vitamin produced in the skin by photo-activiation of 7- dehydrocholesterol .

- fat soluble vitamin with anti-oxdant effects .

Page 44: 2005-2010(March) Mrcog Part 1 Pastpapers Recall Mrcog Total Guide

- fat soluble vitamin deficiency associated with raised serum alkaline phosphatase .

- deficiency of this vitamin associated with osteomalasia .

- maternal intake of this vitamin around the time of coception has been shown to reduce incidance of neural tube defects .

- fat soluble vitamin , deficiency occur within few a days of cessation of bile secretion .

- this vitamin is a cofactor in synthesis of prothrombin . Candida spp Herpes simplex virus Human immunodeficiency virus Gardnerella vaginalis Chlamydia trachomatis Human papilloma virus Treponema pallidum Neisseria gonorrhoeae Trichomonas vaginalis

• Select the micro-organism described in the following cases:

- An obligate intracellular gram negative bacterium ....

- The cause of lymphogranuloma venereum .........

- Neutrophils containing gram negative diplococci .....

Page 45: 2005-2010(March) Mrcog Part 1 Pastpapers Recall Mrcog Total Guide

Double Y Syndrome Down’s Syndrome Edward’s Syndrome Fragile X Syndrome Klinefelter’s Syndrome Patau’s Syndrome Turner’s Syndrome

• Please select the most appropriate syndrome from the options below. You may use each option once, more than once, or not at all.

- An infant girl is born with a webbed neck, hypertension and audible ejection systolic murmur.

- A neonate boy is born with cleft lip and palate, low set ears and polydactyly. The child survives for only 3 weeks.

- An infant boy is born with hypotonia, epicanthic folds and single palmer creases.

Adrenal arteries Celiac trunk Common iliac arteries Gonadal (ovarian or testicular) arteries Inferior mesenteric artery Lumbar arteries Median sacral artery Renal arteries Superior mesenteric artery

• Select the most appropriate option:

- Gives rise to the left gastric, splenic and hepatic arteries.

Page 46: 2005-2010(March) Mrcog Part 1 Pastpapers Recall Mrcog Total Guide

- Gives rise to the intestinal, middle colic and right colic arteries.

- Gives rise to the left colic, sigmoid and superior rectal arteries.

- An unpaired branch that arises from the aortic bifurcation.

- Arises from the aorta between the level of L1 and L2 vertebrae, and forms five segmental arteries that do not anastomose.

• B- Chlamydia trachomatis • C- Herpes simplex virus • D- Human immunodeficiency virus • E- Gardnerella vaginalis • F- Human papilloma virus

Neisseria gonorrhoeae Treponema pallidum Trichomonas vaginalis

• Select the micro-organism described in the following cases:

- A thin motile, gram negative spiral shaped bacterium ...

- An anaerobic flagellated protozoan ...

- A single strand RNA virus .. ACTH CRH Dopamine FSH GnRH Growth Hormone LH Prolactin Somatostatin

Page 47: 2005-2010(March) Mrcog Part 1 Pastpapers Recall Mrcog Total Guide

TSH

• Please select the correct hormone from the options for each of the descriptions below. You may use each option once, more than once, or not at all.

- This hormone acts on cartilage and liver to release IGF-1...

- This hormone promotes iodination of tyrosine residues...

- This hypothalamic hormone inhibits the secretion of growth hormone...

- In males, this hormone facilitates the generation spermatozoa...

- This hormone inhibits galactorrhoea...

Page 48: 2005-2010(March) Mrcog Part 1 Pastpapers Recall Mrcog Total Guide

• Deep inguinal Inferior mesenteric Internal iliac Para-aortic Superficial inguinal

• Lymph from the following structures passes to the above group of lymph nodes first. Please choose the most appropriate answer from the list above.

- Rectal .

- Vulva .

- Hallux .

- Cervix .

- Upper anal canal . Bias Correlation Error False positive False negative Null hypothesis Power Regression Sensitivity Specificity

• Which of the above statistical terms is described by the following statements:

Page 49: 2005-2010(March) Mrcog Part 1 Pastpapers Recall Mrcog Total Guide

- A random source of inaccuracy...

- A type 2 error...

- Ability to exclude a true negative... Clostridium perfringens clostridium difficile Escherichia coli Haemophilus influenzae Staphylococcus aureus streptococcus pyogenes none of the above

• Select the most likely causative organism for the following infections.

- Chronic osteomyelotis after implant surgery ..

- Gas gangrene ..

- Pseudomembranous colitis .. Mean Median Mid-range Mode Range Spread Standard Deviation Standard Error

• Select the appropriate term from the list for the following definitions. Each answer may be used once, more than once, or not at all.

Page 50: 2005-2010(March) Mrcog Part 1 Pastpapers Recall Mrcog Total Guide

- Most frequent value ..

- Spread of estimates of sample means around the true population mean .

- The measure of spread of values around the mean .

- The mid value when all values are listed in ascending order .

- The sum of all the values divided by the number of values . endothelial growth factor human chorionic gonadotrophin human placental lactogen IGF-1 Insulin leptin oestrogen progesterone relaxin

• Select the most appropriate hormone for the following statements.

- The hormone that is the basis of most pregnancy tests.

- Hormone produced by synctiotrophoblast to regulate nutrient storage in the final stages of pregnancy.

- Hormone that is secreted by decidual cells of the placenta .

Page 51: 2005-2010(March) Mrcog Part 1 Pastpapers Recall Mrcog Total Guide

- The hormone that is lactogenic, regulates glucose metabolism and promotes fat breakdown .

• 1) • ant division of ant. remi of S2-4 • ant division of post.rami of S2-4 • b)genitofemoral nerve • post division of ant. remi of S2-4 • post division of post. remi of S2-4 • sensory supply of perineal nerve • inferior rectal nerve • ischial tuberosity • h)ischial spine

• A lady is about to deliver and you are about to give her a pudendal block..

• Q1- what is the root value of pudendal nerve? a • Q2- what nerve supplies lower part of vagina (I think)? e/a • Q3- why do u give local skin infiltration before episiotomy? e • Q4- or what is the nerve supply of peranal area .?

• (2).. If we take the day of fertilization as Day’0’ then…

• Day2 • Day4 • Day 8 • Day 10 • Day12 • Day 14 • Day 18 • Day20 • Day 22

Page 52: 2005-2010(March) Mrcog Part 1 Pastpapers Recall Mrcog Total Guide

• Day 24 • Day 26 • Day 42 [ • Day 70

• Q1- Which represents 4 cell stage ? day 2 • Q2- conceptus implants completely? Day 10/12 • Q3- fetal heart pulse seen on ultrasound? Day 22 • Q4- vertebra form completely? Day 42 • PCR • Fish test • Chromosomal linkage analysis • Telomer analysis • Sum hereditary crosslinkage chromosomal option (dnt remember) • Screen for some known gene mutation • Sweat test • Saliva test

• -A family with one kid presumed to have cystic fibrosis however cftr gene mutation was not detected. Family members willing to give samples if needed.mother wants definite diagnosis as she is 11wks and wants to know diagnosis for her baby.parents are cousins with 1st degree relatives having cystic fibrosis.

- Invasive diagnostics can be applied to reach final results by? ??? - you have Guthrie spot of their son how will u diagnose him for

cystic fibrosis? PCR??? - families in UK with cystic fibrosis how will u find f508? SWEAT

TEST???? • 5)

Endoplasmic reticulum Golgi apparatus Lysosomes Microtubules Mitochondria Nucleolus Plasma membrane Ribosomes

Page 53: 2005-2010(March) Mrcog Part 1 Pastpapers Recall Mrcog Total Guide

• Select the most appropriate organelle that matches the following descriptions

- Contains enzymes capable of digesting cells and cellular material c

- 'Reads' the mRNA and builds protein . h - Modification of lipids and proteins with storage of material prior

to export out of the cell. a

• (6) screening test for following - syphilis treponemal antibody test - syphilis hemaglutination test - syphilis immobilization test - HB-electrophorisis - mcv - mchc - MCh - serum iron - paul-bennel test

alpha thalessemia 4 folic acid deficiency 6 iron deficiency anemia 8 primary syphilis in early untreated period 1

• on taking a large protein meal? Decrease decrease Increase increase Increase decrease Deacrease increase Nochange no change Nochange increase Nochange decrease

Page 54: 2005-2010(March) Mrcog Part 1 Pastpapers Recall Mrcog Total Guide

• Insulin & Glucagon excretion what will happened ...? both will increase in secretions…

• (8) • 3-hydroxyisovaleric acid . • 17-alpha hydroxyprogesterone . • 17-delta hydroxyprogesterone . • acetic acid . • cortisol .

• The following level of certain metabolities in amniotic fluid changes significantly of the fetus has an inborn error of the metabolism .select the single metabolite from list of the options above whose level in amniotic fluid is altered by inheretiance disease in item below.

• congentiat adrenal hyperplasia b

• (9) Adrenaline Calcitonin

• C Cholecystokinin Cortisol Glucagon Insulin Progesterone Somatostatin

• IT- estosterone

• Select the appropriate hormone from the list for the following structures that produce it. Each answer may be used once, more than once, or not at all.

Page 55: 2005-2010(March) Mrcog Part 1 Pastpapers Recall Mrcog Total Guide

o Adrenal Cortex . d o Adrenal Medulla . a o Pancreatic alpha cell . e o Pancreatic D Cells . h

• Akeratinising stratified squamous epithelium • Bnon-keratinising stratified squamous epithelium • Cpseudostratified columnar epithelium • Dsimple columnar epithelium • Esimple cuboidal epithelium • Ftransitional epithelium

• Which of the epithelial types described above are present in the anatomical regions below:

- ectocervix . - endocervical canal . simple columnar - cervical ectropion . - transformation zone of cervix . - vagina . A - uterus . E - Bladder . F - Trachea .F - Labia Majora . - Bowel . simple columnar

• 11-Fallopian tube . ciliated epi - Vulva . - anal canal . simpe columnar

Autosomal co-dominant Autosomal dominant Autosomal recessive

Page 56: 2005-2010(March) Mrcog Part 1 Pastpapers Recall Mrcog Total Guide

Polygenic Single gene defect X linked dominant X linked recessive

• [Select the most likely mode of inheritance for the following patients’ conditions:

- A 27 -year old female developed gestational diabetes mellitus. Her uncle and grandmother also had diabetes mellitus D

- A mother is concerned regarding her baby who has developed fractures which appear to occur with minimal trauma. He has blue sclera. B

- An 18-year old female underwent caries tooth extraction and developed profuse bleeding. On history she revealed menorrhagia. Her mother and her grandfather had the same disease. G

• EMQ

• 1) PGH2

• 2)PGG2 PGE2 PGF2alpha Arachidonic acid Calcium Magnesium

• Q1) Synthesis of prostaglandins by? • Q2) the first Prostaglandin produces? • Q3) Oxytocin uses this as its intermediate precursor?

• 2) Shapes of the pelvic

Page 57: 2005-2010(March) Mrcog Part 1 Pastpapers Recall Mrcog Total Guide

• Round shaped pelvic brim with transverse diameter more than AP

• Round shaped pelvic brim with AP diameter more than the transverse diameter

• C)Oval shaped pelvic brim with transverse diameter more than AP • Oval shaped pelvic brim with AP diameter more

than the transverse diameter • Heart shaped pelvic brim with transverse diameter

more than AP • Heart shaped pelvic brim with AP diameter more

than the transverse diameter

• Q1) Gynecoid • Q2) Android

• 3) DNA and RNA • DNA—Deoxyadenine------Double standed

• Deoxythymine • Deoxycytosine • Deoxyguanine

• DNA—Deoxyadenosine------Single standed • Deoxytyrosine • Deoxycytosine • Deoxyguanine

• RNA—Adenine-----Double stranded • Thymine • Cytosine • Uracil

• ) RNA—Adenine-----Single stranded • Thymine • Cytosine • Uracil

Page 58: 2005-2010(March) Mrcog Part 1 Pastpapers Recall Mrcog Total Guide

• Q1) Messenger RNA? • Q2) DNA? • Q3) Human papilloma virus?

Separated recalled items:

• Hormone increased in Prolactinaemia • Hormone produced by posterior pituatary • Baby born with right sided umbilical hernia • Baby born with stiff limbs and mother had oligohydramniosis • Atomic mass of technetium (99) • Atomic mass of cobalt (58, but the nearest possible on the list 60) • Origins of pudendal nerve- ant s2 s4 • Functions of 21 and 17a hydroxlase (convery what to what) • POstitive predictive value and negative predictive value • Linear regression • Hep B and Hep C- DNA/ RNA viruses • Lidocaine- side effects, shorter that bupivicaine • Anatomy of pelvic bones • Fetal skull • Arterial supply of anus • Lasers- ? red light faster that UV, ? cut with red light • MRI- not radioactive, movement of H ions, resolution • Matching disease with organism- chancre, • Cancers with serum markers • Amyloidosis- ?RA ?intracellular • Bronchodilators • Staph aureus- coagulase, toxin • Time closure of neural tube, time mid gut move from abdo wall • origin of pudendal nerve • nerve supply of the skin of perinium • the structure that can differentiate between body of uterus and cervix

in c/s • struture must be identified during c/s to avoid injury • embryology: by this time if the mid gut not withdrawl to abdomen the

fetus will devolp exomphilus

Page 59: 2005-2010(March) Mrcog Part 1 Pastpapers Recall Mrcog Total Guide

• by this time the an and post neroun was closed ,they give many options

• obstructive jaunduce. • histopathology of endometrium, hyperplasia & adenocarcinoma. • HIV, HBV. • selective COX 2 inhibitors & antiprogesterone. • steroidogenesis pathways & enzymes. • tumour markers. • sarcoidisis. • asbestosis. • PUDENDAL NERVE ROOT VALUE S1,2,3 ANT PRIMARY RAMI • N SUPPLY OF PERIANAL AREA • LENGTH OF UMBILICAL CORD • AUTOSOMAL RECESSIVE TAY SACH'S DISEASE • AUTOSOMAL DOMINANT ACHONDROPLASIA • ANTIINFLAMMATORY MUSCLE RELAXANT PROSTAGLANDIN • STAPH AUREUS COAGULASE POSITIVE, PIGMENTED COLONY,GRAM

POSITIVE • GENTAMYCIN BACTERICIDAL,TOXIC TO 8 TH NERVE

DIURETIC ACTING ON DCT BENDROFLUAZIDE

Page 60: 2005-2010(March) Mrcog Part 1 Pastpapers Recall Mrcog Total Guide

MCQ questions that actually appeared on MRCOG

part 1exams between 2005-2009. 1) The pelvic surface of the sacrum? a) gives origin to the piriform muscles b) gives origin to the levator ani muscles c) is broader in the male than in the female d) trnasmit dorsal rami of sacral nerves e) is in contact with the anal canal 2)Prolactin. a) release is stimulated by TRH b) plasma levels are raised in the first trim of preg. c) release is increased by suckling. d) maybe produced by decidua. e) release is inhibited by metoclopromide. 3) The foll disorders and org are correctly paired. a) opthalmia neonatorum: chlamydia trachomatis b) chancroid: Haemophilus ducreyi. c) sleeping sickness: Leishmania donovani. d) ringworm: Trichenella spiralis. e) non-specific urethritis: Toxoplasma gondii. 4) The heart rate typically increases in response to: a) pain. b) hypoxia. c)ventilatory expiration d) increase in Intracranial pressure e) decrease baroreceptor activity 5) Antibodies. a) are proteins. b) are formed in the fetus before 12 weeks of Intrauterine life. c)have an average molecular weight of around 10 000 daltons. d) of the rhesus type are genetically transmitted. e) are produced by the ribosome of plasma cells.

Page 61: 2005-2010(March) Mrcog Part 1 Pastpapers Recall Mrcog Total Guide

6) the foll disorders have an X linked pattern of inheritance. a) G6PD Deficeincy. b) Kleinfelter syndrome c) adrenogenital syndrome d) haemophilia B e) familial hypercholestroleamia. 7)The foll are derived from the urogenital sinus: a) the bladder trigone. b) the ureters. c) the female urethra. d)greater vestibular glands. e) paraurethral glands.

In the fetal CVS a) the heart arises from endoderm b) the heart is formed by fusion of endocardial tubes. c) Cardiac pulsation is present by the 30th day after fertilization. d) oxygenated blood is tranferred to the left atrium through the foramen ovale. e) the ductus arteriosus closes during the last 4 weeks of pregnancy. 9)Arginine vasopressin a) reduces GFR. b) controls water loss in the Proximal renal tubule. c) is synthesised by the post pituitary gland. d) is released in response to rise in plasma osmolality. e) is released in response to fall in circulating plasma volume. 10) Renin a) is secreted by the zona glomerulosa of the adrenal coretx. b) is a proteolytic enzyme. c) is secreted at an increased rate if the renal perfusion pressure falls. d) acts upon circulating angiotensinogen. e) is released in response to an increase in extracellular fluid volume

Page 62: 2005-2010(March) Mrcog Part 1 Pastpapers Recall Mrcog Total Guide

11) Actinomyces israelii a) is a rickettsia. b) forms yellow granules in pus. c) is a commensal in the mouth. d) is a commensal in the vagina. e) is usually resistant to penicillin. 12) Diseases caused by spirochaetes include. a) Weil's disease. b) lymphogranuloma venereum c) pinta d) Vincent's angina. e) bilharzia 13) The foll are inherited as autosomal recessive conditions: a) tuberous sclerosis. b) phenylketonuria. c) achondroplasia d) sickle cell anaemia. e) Von gierke's disease. 14) Antibodies play an important part in the development of: a) phagocytosis. b) Mantoux responce. c) erythroblastosis fetalis. d) hyperemesis gravidarum e) anaphylaxis 15) chemical mediators concerned in the production of an inflammatory response include: a) 5-hydroxytryptamine b) aldosterone.

Page 63: 2005-2010(March) Mrcog Part 1 Pastpapers Recall Mrcog Total Guide

c) glucocorticoids. d) bradykinin e) leukotreines 16) The parathyroid gland. a) originate from the pharyngeal cleft ectoderm. b) secrete parathyroid hormone via the chief( principal) cells. c) secrete calcitonin via the oxyphil cells. d) may become hyperplastic in the presence of intestinal malabsorption. e) may develop adenomas in association with islet cell tumour of the pancreas 17) In the abdominal wall: a) the rectus abdominis muscle is attched to the crest of the pubis. b) the post border of the external oblique muscle ends in the linea semilunaris. c) the aponeurosis of the external oblique muscle takes part in the formation of the conjoint tendon. d) the inferior epigastric artery is a branch of the internal iliac artery. e) the conjoint tendon blends medially with the anterior layer of the rectus sheath. 1 The urogenital sinus in the female gives rise to the following: a) ureter b) paraurethral glands c) Bartholin's gland d) urachus e)Gartner's duct. 19) Concerning sex hormone: a) the ovary secretes androstenedione. b) The ovary secretes testosterone c) The ovary secretes dihydrotestosterone.

Page 64: 2005-2010(March) Mrcog Part 1 Pastpapers Recall Mrcog Total Guide

d) SHBG conc. are higher in women more than men e) Androgens bound to protein have high biological activity. 20) Features of congenital rubella include: a) excretion of virus by the neonates. b) hepatomegaly c) excessive production of growth hormone. d) cataract e) deafness. 21) Clomifene citrate: a) is an anti-androgens. b) does not stimulate ovulation directly. c) can produce visual disturbance. d) is genereally prescribed throughtout the proliferative phase of the menstrual cycle. e) in the treatment of ovulation increases the risk of multiple pregnancy. 22) The foll are cytotoxic alkylation agents: a) Cyclophosphamide. b) mercaptopurine c) chlorambucil d) fluorouracil e) methotrexate. Most of the statistic questions posted are right. so will post only the ones which are missing from wht i remember. 23) If a distribution of results is markedly skewed to the left: a) the mean is the same as the 50th centile. b) the same number of values lie on either side of the median. c) the mode is equal to the median. d) the student's t test should be used to compare the distribution with

Page 65: 2005-2010(March) Mrcog Part 1 Pastpapers Recall Mrcog Total Guide

another. e) logarithmic transformation of the result will produce a distribution closer to the normal. 24) Corcerning the analysis of clinical trials: a) the 95% confidence interval indicates the range within which 19 out of 20 values will lie. b) The P value illustrates how often the result would be expected to occur by chance. c)b The conventional level of statistical significance is set of P<0.005 d) In a randomised trial, there must be equal numbers of results in each arm of the study. e) A relative risk reduction of 60% is significant irrespective of the value of P. 25) The following substance are normally synthesized in the liver: a) glucagon b) vitamin A c) cholesterol. d) immunoglobulins e) prothrombin 26) The pineal gland: a) is situated at the anterior end of the 3rd ventricles. b) is innervated by the parasymphathetic nervous system. c)produces melatonin. d)maybe calcified in the adults. e) is most active during daylight. 27)In congenital adrenal cortical hyperplasia. a) The commonest deficiency is C18 hydroxylase. b) plasma cortisol concentration is raised. c) urinary excretion of 17 oxysteroids is elevated.

Page 66: 2005-2010(March) Mrcog Part 1 Pastpapers Recall Mrcog Total Guide

d) dexamethasone will suppress the urinary excretion of 17 oxysteroids. e) there is no virilising effects. 28)Foll are RNA containing virus: a) coxsackie b) influneza c) mumps d) herpes simplex. e) cytomegalovirus. 29) Listeria monocytogenes: a) is a gram negative organism. b) is sensitive to ampicillin. c) may cause a transplacental infection. d) is sexually transmitted. e) can be cultured from a high vagina swab. 30) actinomyces israelii. a)is a fungus. b) forms yellow granules in pus. c)is a mouth commensal d) occurs in association with IUCD. e) is resistant to penicillin. 31) The following drugs may cause enlargement of the fetal thyroid gland: a)methyldopa b) thyroxine c) carbimazole d) propranolol e) propylthiouracil 32) The foll statements about anticoagulant are correct:

Page 67: 2005-2010(March) Mrcog Part 1 Pastpapers Recall Mrcog Total Guide

a)Heparin inhibits the action of thrombin b) The action of heparin is antagonised by vitamin K c) Heparin increases antithrombin III activity. d) The effects of anticoagulants are decreased by metronidazole. e) Warfarin is greater than 80% protein bound in plasma 33) The following drugs and side effects are associated: a) methydopa: depression b) paracetamol: thromboembolism c) indomethacin: peptic ulcer d) prednisolone: osteoporosis. e) ritodrine: hypoglycemia 34) Haematopoiesis in the fetus: a) results in nucleated erythrocytes early in development. b) occurs in the yolk sac in the first month. c) does not occur in the bone marrow until term. d) is predominantly hepatic during the 4th month. e) does not require folic acid. 35)Early blood borne dissemination is characterised feature of: a) carcinoma of the endometrium. b) osteosarcoma c) basal cell carcinoma d) carcinoma of the cervix e) choriocarcinoma. 36) The following cells maybe phagocytic: a) neutrophils. b) kupffer cells c) monocytes d) Hofbauer cells.

Page 68: 2005-2010(March) Mrcog Part 1 Pastpapers Recall Mrcog Total Guide

e) plasma cell. 37) The following are premalignant conditions: a) diverticular disease of the large bowel. b) ulcerative colitis. c) pulmonary asbestosis. d) Paget's disease of the bone. e) condylomata of the vulva 3 In the pituitary gland: a) the anterior lobe is smaller than the posterior lobe. b) the posterior lobe is ectodermal in origin. c) the acidophil cells produces oxytocin. d) the basophil cells produce growth hormone. e) the blood supply is derived from the internal carotis artery. 39)The obturator artery: a)branches from the posterior trunk of the internal iliac artery. b) passes through the greater sciatic foramen. c) is crossed by the ureter. d)supplies the hip joint. e) may be replaced by a branch of the superior epigastric artery. 40) In congenital adrenal hyperplasia: a) the commonest cause is a deficiency of 21 hydroxylase. b) the plasma cortisol conc is increased. c) there may be excessive secretion of 17 alpha hydroxyprogesterone. d) sodium retention is characteristic. e)blood cathecholamine conc are increased. 41) In the fetal lung: a)bronchial cartilage formation commences at 18-24 weeks of gestation.

Page 69: 2005-2010(March) Mrcog Part 1 Pastpapers Recall Mrcog Total Guide

b) type II alveolar cells first appear at 16-20 weeks gestation. c) sphingomyelin is the most common phospholipid present at term d) phospholipid release is increased by endogenous adrenaline. e) phospholipid production is decreased by exogenous corticosteroids. 42) In radiotherapy a) 1 gray is equivalent to 1 joule/kg. b) the skin usually receives a greater dose of radiation than the underlying tissues. c) the major effect of radiation energy is to damage the cytoplasm of the cell. d)cells in tissues which are hypoxic are more vulnerable to radiation. e)Radiation induced changes in tissues may take 6 weeks to develop. 43) Concerning the adrenal glands a) cortex is derived from neural crest cells. b) Zona fasiculata secretes aldosterone. c) Cortical adenomas may cause Cushing syndrome. d) Neuroblastoma arise in the medulla. e) Addison's disease may result from autoimmune destruction of the cortex. 44)The following are autosomal recessive: a) neurofibromatosis. b) cystic fibrosis. c) phenylketonuria d) polyposis coli e) sickle cell anaemia 45) Uterine fibroids: a) are defines histologically as fibromyxomas. b) arise from endometrial stroma c) maybe associated with polycythamia.

Page 70: 2005-2010(March) Mrcog Part 1 Pastpapers Recall Mrcog Total Guide

d) predispose to endometrial hyperplasia. e) are liable to sarcomatous change in about 5% of cases. 46) growth of the foll tumors are hormone dependent: a) squamous cell carcinoma of the cervix. b) breast adenocarcinoma. c) uterine leiomyoma. d) prostatic adenocarcinoma e) testicular carcinoma 47)Surfactants: a) is formed mainly in the placenta b) levels in amniotic fluid diminish after 33 weeks of gestation c) formation can be inferred from the lecithin-sphingomyelin ratio in amniotic fluid. d) contains palmitic acid e) decreases the surface tension in pulmonary alveoli. 4 Intracellular fluid differs from Extracellular fluids in that: a) it forms the major proportion of total body water. b) its volume can be measured easily. c) it has a higher concentration of potassium than of sodium. d) its volume is regulated primarily by the kidneys. e) it has a higher phosphate concentration. 49) The Anal canal: a) has an upper part which is innervated by the inferior hypogastric plexus. b) has a lower part which is supplied by the superior rectal artery c) drains lymph to the superficial inguinal nodes from its upperpart. d) has its internal sphincter innervated by the infecrior rectal nerve. e) has a superficial part of its external sphincter attached to the coccyx.

Page 71: 2005-2010(March) Mrcog Part 1 Pastpapers Recall Mrcog Total Guide

50) The right ovarian artery: a) Arises from the abdominal aorta above the renal artery. b) passes posterior to the 3rd (horizontal) part of the duodenum c) passes post to the genitofemoral nerve. d) supplies the right ureter. e) anastomoses with the right uterine artery. 51) Vulva supplied by: a) Internal pudendal artery. b) Inferior rectal artery. c) Genitofemoral artery d) obturator artery e) femoral artery.. This is a question outside past papers..All r false Except internal pudendal artery.. 52) The vagina: a) has an anterior wall longer than the post wall. b) contains mucus secreting glands in its epithelium. c) is related in its lower third to the bladder base. d) during reproductive life has an acid pH. e) is derived from mesonephric duct. 53) The right ureter: a) is approx 50cm in length. b) ia partly covered by duodenum c) crosses the genitofemoral nerve. d) enters the bladder anteromedially. e) receives part of its blood supply from the uterine artery. 54) After birth:

Page 72: 2005-2010(March) Mrcog Part 1 Pastpapers Recall Mrcog Total Guide

a) allantois froms median umblical ligament b) umbilical vein forms medial umbilical ligament. c) umbilical artery forms superior vesical artery. d) ductus venosus forms the ligamentum teres. e) ductus arteriosus forms the arch of the aorta. 55)In spermiogenesis: a) primary spermatocytes undergo reduction division. b) primary spermatocyte gives rise to 4 spermatids. c) whole process of spermatogenesis in man takes 6-7 days. d) grossly abnormal spermatozoa may be found in fertile semen. e) spermatids are haploid. 56) In the human male, dihydrotestosterone: a) is a precursor of testosterone. b) has one-tenth of the ptency of testosterone. c) is responsible for involution of the Mullerain system. d) is responsible of the male external genitalia. e) binds to an intracellular receptor. 57) In Human lactation: a) estrogens promote development of breast lobules. b) estrogen promotes milk producing effect of prolactin on the brest, c) human placental lactogen is essential for milk synthesis. d) prolactin stimualtes gonadotrophin release. e) oxytocin causes milk ejection, 58)Concerning testicular hormones: a) testosterone reduces plasma LH conc. b) Inhibin stimulates LH production. c) Estrogen are formed in the testis. d) Testosterone is converted to dihyrotestosterone by 5 alpha reductase. e) Testosterone in plasma is predominantly bound to albumin.

Page 73: 2005-2010(March) Mrcog Part 1 Pastpapers Recall Mrcog Total Guide

59) Epidermal growth factor: a) is mitogenic. b) synthesis is stimulated by estradiol. c) is a steroid molecule. d) is found in endometrium. e) binds to a receptor on the nuclear membrane. 60) folic acid: a) deficiency causes megaloblastic bone marrow.. b) is hydroxycobalamin. c) is present in green vegetables. d) is predominantly absorbed from the large intestine. e) is destroyed by boiling water. 61) Doppler Ultrasound: a) is used to monitor fetal breathing. b) is used in fetal HR monitors. c) can be used to measure blood velocity in the fetus. d) measure proton relaxation times. e) requires injection of contrast agents. 62) The following are structural abberation of chromosomes: a) deletions. b) inversions. c) aneuploidy. d) polyploidy. e) translocation. 63) Messenger RNA a) synthesis is dependant on RNA polymerase. b) is an exact copy of sense DNA.

Page 74: 2005-2010(March) Mrcog Part 1 Pastpapers Recall Mrcog Total Guide

c) contains exons. d) is measured by western analysis. e) translationoccurs in the nucleus. 64) In tumors of the bones: a) primary malignancy is more common than 2ndry malignancy. b) osteoma rarely present in skull bones. c) osteosarcoma is associated with Paget's disease of bone. d) lymph node metastases are unusual. e) simple bone cysts have a strong tendency to recur.

Questions about

• endometrial hyperplasia • side effect of drug (carboplatin)-(taxal) • arias stella if its specific in pregnancy • chlamydia • varicella • hpv ,wart • hcg concentration in pregnancy • citric acid cycle diagram • hormones diagram levels of, fsh,lh,estrogen,testosterone • by transvaginal u/s yolk sac appear in which week • genetic about pcr, fish test • laser • totipotent cell,tumour like mass • voiding presure • ca requirement in pregnancy • test used to diagnose folic acid deficency • thalasthmia • cervical ectropion cells are columnar cell • mri contraindicated in early pregnancy • stellate instability in dna • transverse abdominus ms attached to lumbar transverse process • cut s1 lead to autonomic bladder

Page 75: 2005-2010(March) Mrcog Part 1 Pastpapers Recall Mrcog Total Guide

• cancer which are hormone dependent -breast -prostate -thyroid

• guthre test • cystic fibrosis and diagnosis

-swet test -karyotyping-fish test • disinfection and sterilization • complication of contraceptive pills • innervation of all pelvic organs. • lining of ureter • erythropoeitin and renin q from john duthie • complications of cisplatin paclitaxel • statistics -std error calculation mean in a normal distribution • occipito frontal diameter • hcg titre at detection by tv u/s 10 at 3 days after fertilization • crl in scan is 60mm what is gestational age ,crl at term ? • parietal suture is between • kuffer cells are –phagocytic • estradiol receptors • progestrone receptors -we know that these are intra nuclear but

there were confusing options with intranuclear , one option had intranuclear kinase .

• anticonvulsant with description of phenytoin • vitamin deficiency causing hyperemesis • macrocytic anemia • xerophtalmia • cystic fibrosis • complement causes • b thalasemia detection • folate def detection • basal cell ca - local malignant i think • tumor like –hamaroma • cystic fibrosis guthic spot test • receptors in apoptosis caspa • arias stella reaction

Page 76: 2005-2010(March) Mrcog Part 1 Pastpapers Recall Mrcog Total Guide

• blood picture in PET and in preg is inflam like • in statistics mostly repeated questions..example given..calculate

sensitivity,specificity,+ve predictive value..in right sided skewed distribution curve median will be on which side of mean?

• microsatellites alleles - cd4 -nk cells

• down syndrome • pulmonary embolism

• apoptosis or endometrial receptors. • anal canal • vagina • ovary • Phenylketonuria • inhibin • passive transport • erythropoietin • renin • hcg levels • syphylis • rubella • hormone dependent malignancies(testicular carc._sertoli leydig-

clear cellcarc.ofkidney-thyroid carc.) • pigd • congenital heart • ring Y chromosome • passive diffussion not depend on . • concentration gradient • molecular size • following tissue are capable of regeneration

spinal cord liver

Page 77: 2005-2010(March) Mrcog Part 1 Pastpapers Recall Mrcog Total Guide

epidermis myocardium bone marrow

• double blind trial • level of HCG in the urine pregnancy test • APOPTOSIS • methods of disinfection and • BOWIE DICK test • progesterone receptors • syphilis , toxoplasma , hpv , immune responses • down syndrome

associated with duodenal atresia true associated with ambig genetalia false only maternal chromosome F only paternal chromosome F

• oogonia • miosis starts at puberty • miosis before mitosis • during S phase chromosomes are doubled. • to calculate sensitivity and specificity. • standard deviation • standard error of the mean. • mean, mode median in normal distribution and skewed data. • T test • MRI... type of radiation,

causes effect in pregnancy women should be placed left tilted during scan.

• dose of radiation . 1 gray = joules/ kg . absorbed dose.

• dopplers USG •

Page 78: 2005-2010(March) Mrcog Part 1 Pastpapers Recall Mrcog Total Guide

EMQ questions that actually appeared on MRCOG

part 1exams between 2007-2009.

EMQ about

• vitamins (hyperemesis), vit B6 and (xerophthalmia).

• paclitaxel carboplatin side effects

• complex partial seiizure gum hypertrophy,acne ,facial

coarsening,vit K TO BE given..?drug used

• oestrogen progesterone receptor site

• vitamins-xerophthalmia..morning sickness..macrocytic anaemia.

• rubella incubation period,specific immunity within 15 days..

• HEPATOCELLULAR carcinoma,..??

• lining epithelium of ureter,..?

• mantoux rection..

• ovulatory DUB

• aneurysm- 10% due 2 inflam.,syphilitic aneu,thoracic

vessels,marfan recessive.

• if a mother has a child with cystic fibrosis ,she is pregnant and will

do amniocentesis

a. which test to be done (every possibility mentioned)

b. all relatives will volunteer to do gutic spot test ,which test you

will carry on the blood .

• crown \heel lenghth at birth

crown/rump length 6 cm =? Weeks

• type of hpv causing benign wart

type of hpv causing cancer cx

Page 79: 2005-2010(March) Mrcog Part 1 Pastpapers Recall Mrcog Total Guide

• .B cell

.plasma cell

.dendritic cell

.mast cells

. t cell

QUESTION

. ANTIBODY PRODUCTION?

. ANTIGEN PRESENTATION?

• Options 1. oval inlet transversal diameter longer than anterposterior . 2. oval inlet anterposterior dia longer than trans. 3. heart shape inlet anterposterior dia. longer than trans. 4. heart shape inlet trans. diam. longer than anterposterior 5. rounded inlet .. T>A. 6. rounded inlet ..A > T Qus. a. Gynaecoid pelvis. b. Android pelvis . ------------------------------------------------------------ a. Adenine monophosphate Guanine monophosphate Cytosine monophosphate duple helix Thymine monophosphate b. Adenine monophosphate Guanine monophosphate Cytosine monophosphate duple helix Uracil monophosphate c. Adenine monophosphate Guanine monophosphate Cytosine monophosphate single helix

Page 80: 2005-2010(March) Mrcog Part 1 Pastpapers Recall Mrcog Total Guide

Uracil monophosphate d. Adenine monophosphate Guanine monophosphate Cytosine monophosphate single helix Thymine monophosphate e. Adenine monophosphate Guanine monophosphate Cytosine monophosphate duple helix Thymine monophosphate f. Diadenine monophosphate Diguanine monophosphate Dicytosine monophosphate duple helix Diuracil monophosphate c. DiAdenine monophosphate DIGuanine monophosphate DICytosine monophosphate single helix DIUracil monophosphate d. DIAdenine monophosphate DIGuanine monophosphate DICytosine monophosphate single helix DIThymine monophosphate e. Adenine monophosphate Guanine monophosphate Cytosine monophosphate duple helix Thymine monophosphate b. Adenine monophosphate Guanine monophosphate Cytosine monophosphate duple helix Uracil monophosphate

Page 81: 2005-2010(March) Mrcog Part 1 Pastpapers Recall Mrcog Total Guide

c. Adenine monophosphate Guanine monophosphate Cytosine monophosphate single helix Uracil monophosphate d. Adenine monophosphate Guanine monophosphate Cytosine monophosphate single helix Thymine monophosphate 1. DNA. 2. mRNA 3. Nucliac acid o HPV.

• Options Somatotroph, acidophil Somatotroph, basophil lactotroph, acidophil lactotroph, basophil corticotroph, acidophil corticotroph, basophil gonadotroph, acidophil gonadotroph, basophil thyrotroph, acidophil thyrotroph, basophil Q1. Growth hormone Q2. Prolactin

number 1: a. pelvic brim oval in shape, transverse diameter more than anteropost diameter. b. pelvic brim oval in shape, anteropost diameter more than transverse diameter. c. pelvic brim heart shaped, transverse diameter more than anteropost

Page 82: 2005-2010(March) Mrcog Part 1 Pastpapers Recall Mrcog Total Guide

diameter. d. pelvic briim heart shaped, anteropost diameter more than transverse diameter. e. pelvic brim circular, transverse diameter more than anteropost diameter. f. pelvic brim circular, anteroposr diameter more than transverse. 1) gynecoid pelvis. 2) android pelvis. number 2: a.double stranded, adenosine monophosphate, guanine monophosphate, cytosine monophosphate, uracil monophosphate. b. double stranded, adenosine monoph, guanine monph, cytosine monoph, thymine monoph. c. single stranded, adenine monoph, guanine monoph, cytosine monoph, thymine monoph. d. single stranded, adenine monoph, guanine monoph, cytosine monoph, uracil monoph. e. single stranded, deoxyadenine monoph, deoxyguanine monoph, deoxycytosine monoph, deoxyuracil monoph. f. single stranded, deoxyadenine monoph, deoxyguanine monoph, deoxycytosine monoph, deoxythymine monoph. g. double stranded, deoxyadenine monoph, deoxyguanine monoph, deoxycytosine monoph, deoxyuracil monoph h. double stranded, deoxyadenine monoph, deoxyguanine monoph, deoxycytosine monoph, deoxythymine monoph, 1) DNA. 2) RNA. 3) genome of HPV. number 3: a. edwards syndrome. b. exomphalus.

Page 83: 2005-2010(March) Mrcog Part 1 Pastpapers Recall Mrcog Total Guide

c. gastroschiasis. d. patau's syndrome. e. down's syndrome. f. kleinfeltr's syndrome. 1) abnormality in the anterior abdominal wall, usually to the right of the umbilicus, other genetic abnormalities rarely associated. 2) abnormality in chromosome 18. number 4: a. azithromycin. b. amoxycillin. c. cefuroxime. d. benzylpenicillin. e. metronidazole. 1) best treatment of acute upper UTI in third trimester. 2)treatment of chlamydia in a non-pregnant woman. number 5: a. L1,2,3 b. L1 c. L2,3,4 d. L1,2 e. L3,4,5 1) ilioinguinal nerve. 2) genitofemoral nerve. number 6: a. streptococus, aerobic, G+ve. b. streptococus, anaerobic, G+ve. c. staphylococus, aerobic, G+ve. d. staphlococus, anaeobic, G-ve.

Page 84: 2005-2010(March) Mrcog Part 1 Pastpapers Recall Mrcog Total Guide

e. pseudomonas, aerobic, G+ve. f. pseudomonas, aerobic G-ve. g.pseudomonas, anaerobic, G-ve. 1) toxic shock syndrome. 2) hospital cross infection. number 7: a. CMV. b. listeria monocytogens. c. treponame pallidum. d. HIV. e. human leucocytic virus. f. staphlococus aureus. g. streptococus. h. varicella zoster. 1) a pregnant woman developed a "flue-like" illness with fever and general malaise, her baby was born with hepatosplenomegaly and jaundice. 2) a woman has a history of 2 previous stillbirths, is now pregnant and at 37 weeks complained of fever and rash, her son was born healthy but at age of one year he had abnormal incisors and later deafness. number 8: a. hepatitis C, DNA. b. hepatitis C, RNA. c. herpes simplex. d. HPV. e. i cant recall the other options. 1) associated with high incidence of hepatocellular carcinoma. 2) papular skin rash. number 9:

Page 85: 2005-2010(March) Mrcog Part 1 Pastpapers Recall Mrcog Total Guide

a. estrogen. b. progesterone. c. DHEA-sulphate. d. cortisol. e. cortisone. 1) hormone maintains uterine quisence throughout pregnancy. 2) hormone produced by the placenta and fetal adrenal gland, promotes fetal lung maturation. number 10: a. allantois. b. cloaca. c. mesonephros. d. yolk sac. 1) rectum develops from the posterior portion if this structure. 2) germ cells arise from it. number 11: a. B cell b. T CD4 cell c. T CD8 cell d. dendritic dells. 1) major antigan presenting cell. 2) could not recall it.

EMQ 1) 1) PGH2 2)PGG2 3) PGE2 4) PGF2alpha

Page 86: 2005-2010(March) Mrcog Part 1 Pastpapers Recall Mrcog Total Guide

5) Arachidonic acid 6) Calcium 7) Magnesium Q1) Synthesis of prostaglandins Q2) the first Prostaglandin produces. Q3) Oxytocin uses this as its intermediate precursor 2) Shapes of the pelvic A) Round shaped pelvic brim with transverse diameter more than AP B) Round shaped pelvic brim with AP diameter more than the transverse diameter C)Oval shaped pelvic brim with transverse diameter more than AP D) Oval shaped pelvic brim with AP diameter more than the transverse diameter E) Heart shaped pelvic brim with transverse diameter more than AP F) Heart shaped pelvic brim with AP diameter more than the transverse diameter Q1) Gynecoid Q2) Android 3) DNA and RNA A) DNA—Deoxyadenine------Double standed Deoxythymine Deoxycytosine Deoxyguanine B) DNA—Deoxyadenosine------Single standed Deoxytyrosine Deoxycytosine Deoxyguanine C) RNA—Adenine-----Double stranded Thymine

Page 87: 2005-2010(March) Mrcog Part 1 Pastpapers Recall Mrcog Total Guide

Cytosine Uracil D) ) RNA—Adenine-----Single stranded Thymine Cytosine Uracil Q1) Messenger RNA Q2) DNA Q3) Human papilloma virus

4) 1) L1 2) L1, L2 3) L1, L2, L3 4) L1, L2, L3, L4 5) L2, L3, L4 6) L3, L4 Q1) Ilioinguinal Q2) Genitofemoral.

) Given in a single dose A) AZITHROMYCIN B) Amoxicillin C) Cephalosporin D) Vancomycin E) Gentamycin F) Benzylpenicillin Q1) A female with a chlamydial infection, and non-pregnant, which is the best medication? Q2) A female presents with acute upper UTI?

6)Infections A) Staph

Page 88: 2005-2010(March) Mrcog Part 1 Pastpapers Recall Mrcog Total Guide

B) Strept C) Pseudomonas aeruginosa Q1) most common cause wound infection in hospital Q2) hospital acquired infection 7) Statistics Another question on calculation for specificity and sensitivity.. People preparing for March 2010, please learn the formulas well.

8-THE FOLLOWING STATEMENTS ABOUT THE ADRENAL GLAND ARE CORRECT a-they lie anterior to the diaphragm ...........True. b-the left adrenal gland lies behind the pancreas.....True. C-lymphatic drainage is to the superficial inguinal nodes.....False d-the adrenal cortex contains chromaffin cells .........False. e-the adrenal medulla is derived from mesoderm ....False the ovary a- is attached to the ant surface of the broad ligament b- lies on the genitofemoral nerve c- lies in the angle between the ureter and the external iliac vessel d- - has visceral afferent fibres from the pelvic splanchnic nerve e- has lymphatic drainage to the superficial inguinal lymph nodes concerning the uterus a- it is formed from the mesonephric duct b- it has a lymphatic drainage in part to the inguinal glands c- the uterine artery passes below the ureter d- the uterine vein communicate with the vesical plexus of veins e-pain sensation from the body of uterus is carried by the pelvic splanchnic nerve about this stem am not sure if it was like that in the exam Obturator artery a- branches from the posterior trunk of the internal iliac artery

Page 89: 2005-2010(March) Mrcog Part 1 Pastpapers Recall Mrcog Total Guide

b- passes through the greater sciatic foramen c- is crossed by the ureter d- supplies the hip joint e- may be replaced by a branch of the superior epigastric artery

The external iliac artery a-enters the thigh anterior to the inguinal ligament b- at its origin is crossed by ureter c- at its origin is crossed by ovarian vessels d-lies medial to the external iliac vein at its distal end e-give rise to the deep external pudendal artery . The pelvic splanchnic nerves.. A- Are derived from the posterior rami of the sacral spinal nerves B- Supply afferent fibres C- Unite with branches of the synpathetic pelvic plexus D- Supply the ascending colon with motor fibres E- Supply the uterus with parasympathetic fibres In the anterior abdominal wall... A- Rectal muscle is intersected tranversely by three bands B- The posterior rectus sheath below the arcuate line consists of transversalis fascia only C- Above the costal margin the posterior rectus sheath is deficient D- The superior epigastric artery arises from the internal thoracic artery. E- The inferior epigastric artery arises from the femoral artery The pelvic surface of the sacrum. A- Gives origin to the piriforms muscle B- Gives origin to the levator ani muscle C- Is broader in the male than in the female D- Transmits the dorsal remi of sacral nerves E- Is in contact with the anal canal.

Page 90: 2005-2010(March) Mrcog Part 1 Pastpapers Recall Mrcog Total Guide

The rectum A- Is supplied in part by the inferior rectal artery B- Is innervated by the inferior rectal nerve C- Is lined by stratified squamous epithelium D- Has its lymphatic drainage to the superficial inguinal nodes E- Possesse a complete layer of longitudinal muscle The Spleen A- has a notched posterior border B- lies in front of the costo-diaphragmatic recess C- is in contact with the body of the pancrease D- lies under the cover of the 9th to the 11th ribs E- is innervated from the renal plexus THE VULVA a-internal pudendal nerve b-anterior cutaneous of thigh c-inferior rectal d-illioinguinal e-obturator the pineal gland a-is situated at the anterior end of the third ventricle b-is innervated by parasympathetic nervous system c-produce melatonin d-may be calcified in the adult e-is most active during daylight

screening test for following 1- syphilis treponemal antibody test 2- syphilis hemaglutination test 3- syphilis immobilization test 4- HB-electrophorisis 5- mcv

Page 91: 2005-2010(March) Mrcog Part 1 Pastpapers Recall Mrcog Total Guide

6- mchc 7- MCh 8- serum iron 9- paul-bennel test Q1) Thalassemia Q2) Syphilis

Sep. 2009 Options 1. oval inlet transversal diameter longer than anterposterior . 2. oval inlet anterposterior dia longer than trans. 3. heart shape inlet anterposterior dia. longer than trans. 4. heart shape inlet trans. diam. longer than anterposterior 5. rounded inlet .. T>A. 6. rounded inlet ..A > T Qus. a. Gynaecoid pelvis. b. Android pelvis . ------------------------------------------------------------ a. Adenine monophosphate Guanine monophosphate Cytosine monophosphate duple helix Thymine monophosphate b. Adenine monophosphate Guanine monophosphate Cytosine monophosphate duple helix Uracil monophosphate c. Adenine monophosphate Guanine monophosphate Cytosine monophosphate single helix Uracil monophosphate d. Adenine monophosphate

Page 92: 2005-2010(March) Mrcog Part 1 Pastpapers Recall Mrcog Total Guide

Guanine monophosphate Cytosine monophosphate single helix Thymine monophosphate e. Diadenine monophosphate Diguanine monophosphate Dicytosine monophosphate duple helix Diuracil monophosphate f. Diadenine monophosphate Diguanine monophosphate Dicytosine monophosphate duple helix Dithymine monophosphate g. DiAdenine monophosphate DIGuanine monophosphate DICytosine monophosphate single helix DIUracil monophosphate h. DIAdenine monophosphate DIGuanine monophosphate DICytosine monophosphate single helix DIThymine monophosphate 1. DNA. 2. mRNA 3. Nucliac acid o HPV.

STRUCTURE OF BASES AND NOT THE NUCLEOTIDE OR NUCLEOSIDES

gynecoid and android pelvis and the DNA

1) a) ant division of ant. remi of S2-4

Page 93: 2005-2010(March) Mrcog Part 1 Pastpapers Recall Mrcog Total Guide

b) ant division of post.rami of S2-4 b)genitofemoral nerve c) post division of ant. remi of S2-4 d) post division of post. remi of S2-4 e) sensory supply of perineal nerve f) inferior rectal nerve g) ischial tuberosity h)ischial spine A lady is about to deliver and you are about to give her a pudendal block.. Q1- what is the root value of pudendal nerve? Q2- what nerve supplies lower part of vagina (I think)? Q3- why do u give local skin infiltration before episiotomy? Q4- or what is the nerve supply of peranal area .? (2).. If we take the day of fertilization as Day’0’ then… Day2 Day4 Day 8 Day 10 Day12 Day 14 Day 18 Day20 Day 22 Day 24 Day 26 Day 42 [ Day 70 Q1- Which represents 4 cell stage ?

Page 94: 2005-2010(March) Mrcog Part 1 Pastpapers Recall Mrcog Total Guide

Q2- conceptus implants completely? Q3- fetal heart pulse seen on ultrasound? Q4- vertebra form completely? (3) A Cytoplasmic transcription factor receptor B G protein coupled receptor on cell membrane C G protein coupled receptor on Golgi complex D Mega subunit ligand gated ion channel E Multisubunit antibody receptor on cell membrane F Multisubunit ligand gated ion channel on cell membrane G Nuclear protein kinase receptor H Nuclear transcription factor receptor I Protein kinase receptor on cell membrane J Receptor protein complex (intracytoplasmic) K Transcription factor receptor on cell membrane Select the receptor which binds the molecules referred to in the itemS below. Each option may be used once, more than once or not at all. 1- insulin . 2- progesterone . 3- ostradiol . 4- prostaglandin . (4) PCR Fish test Chromosomal linkage analysis Telomer analysis Sum hereditary crosslinkage chromosomal option (dnt remember) Screen for some known gene mutation Sweat test

Page 95: 2005-2010(March) Mrcog Part 1 Pastpapers Recall Mrcog Total Guide

Saliva test -A family with one kid presumed to have cystic fibrosis however cftr gene mutation was not detected. Family members willing to give samples if needed.mother wants definite diagnosis as she is 11wks and wants to know diagnosis for her baby.parents are cousins with 1st degree relatives having cystic fibrosis. 1- Invasive diagnostics can be applied to reach final results by? 2- you have Guthrie spot of their son how will u diagnose him for cystic fibrosis? 3- families in UK with cystic fibrosis how will u find f508? (5) A- Endoplasmic reticulum B- Golgi apparatus C- Lysosomes D- Microtubules E- Mitochondria F- Nucleolus G- Plasma membrane H- Ribosomes Select the most appropriate organelle that matches the following descriptions 1- Contains enzymes capable of digesting cells and cellular material 2- 'Reads' the mRNA and builds protein . 3- Modification of lipids and proteins with storage of material prior to export out of the cell. (6)[i][b] screening test for following 1- syphilis treponemal antibody test

Page 96: 2005-2010(March) Mrcog Part 1 Pastpapers Recall Mrcog Total Guide

2- syphilis hemaglutination test 3- syphilis immobilization test 4- HB-electrophorisis 5- mcv 6- mchc 7- MCh 8- serum iron 9- paul-bennel test[/b][/i] A- alpha thalessemia B- folic acid deficiency C- iron deficiency anemia D- primary syphilis in early untreated period (7) on taking a large protein meal? A- Decrease decrease B- Increase increase C- Increase decrease D- Deacrease increase E- Nochange no change F- Nochange increase G- Nochange decrease Insulin & Glucagon excretion what will happened ...? (8) a- 3-hydroxyisovaleric acid . b- 17-alpha hydroxyprogesterone . c- 17-delta hydroxyprogesterone . d- acetic acid . e- cortisol . The following level of certain metabolities in amniotic fluid changes significantly of the fetus has an inborn error of the metabolism .select the single metabolite from list of the options above whose level in amniotic fluid is altered by inheretiance

Page 97: 2005-2010(March) Mrcog Part 1 Pastpapers Recall Mrcog Total Guide

disease in item below. congentiat adrenal hyperplasia (9) A- Adrenaline B- Calcitonin C Cholecystokinin D- Cortisol E- Glucagon F- Insulin G- Progesterone H- Somatostatin IT- estosterone Select the appropriate hormone from the list for the following structures that produce it. Each answer may be used once, more than once, or not at all. 1- Adrenal Cortex . 2- Adrenal Medulla . 3- Pancreatic alpha cell . 4- Pancreatic D Cells . (10) AAnterior cerebral artery BAnterior communicating artery CAnterior inferior cerebellar artery DBasilar artery EInternal auditory artery FInternal carotid artery GMiddle cerebral artery HPosterior cerebral artery IPosterior communicating artery

Page 98: 2005-2010(March) Mrcog Part 1 Pastpapers Recall Mrcog Total Guide

JSuperior cerebellar artery KVertebral artery Complete the diagram of the circle of Willis using the options given: this question came with diagrame in september 2007 so you should study circle of weils very well if I get the pictures I will post it .

Akeratinising stratified squamous epithelium Bnon-keratinising stratified squamous epithelium Cpseudostratified columnar epithelium Dsimple columnar epithelium Esimple cuboidal epithelium Ftransitional epithelium Which of the epithelial types described above are present in the anatomical regions below: 1- ectocervix . 2- endocervical canal . 3- cervical ectropion . 4- transformation zone of cervix . 5- vagina . 6- uterus . 7- Bladder . 8- Trachea . 9- Labia Majora . 10- Bowel . 11-Fallopian tube . 12- Vulva . 13- anal canal . A- Autosomal co-dominant B- Autosomal dominant

Page 99: 2005-2010(March) Mrcog Part 1 Pastpapers Recall Mrcog Total Guide

C- Autosomal recessive D- Polygenic E- Single gene defect F- X linked dominant G- X linked recessive [Select the most likely mode of inheritance for the following patients’ conditions: 1- A 27 -year old female developed gestational diabetes mellitus. Her uncle and grandmother also had diabetes mellitus2- A mothr is concerned regarding her baby who has developed fractures which appear to occur with minimal trauma. He has blue sclera. 3- An 18-year old female underwent caries tooth extraction and developed profuse bleeding. On history she revealed menorrhagia. Her mother and her grandfather had the same disease. A- WARFARIN . B- HEPARIN . C- AMPICILLIN . D- METHYLDOPA . E- PENCILLIN . F- CARBIMAZEPINE . G- METRONIDAZOLE . H- NON OF THE ABOVE . I- ALL OF THE ABOVE . For each of the following choose the single most appropirate statement from the above list of options . 1- Drug contraindicated in breast feeding . 2- Drug does not cross the placenta .

A. Calcitonin

Page 100: 2005-2010(March) Mrcog Part 1 Pastpapers Recall Mrcog Total Guide

B. Cortisol C. Glucagon D. Growth hormone E. Insulin F. Oestradiol 17-¦Â¦Â G. Oxytocin H. Parathyroid hormone I. Prolactin J. Thyroxine Instructions: For each action described below, choose the single most likely causative hormone from the above list of options. Each option may be used once, more than once, or not at all. Question 3: Stimulates deposition of cartilage at the ends of bones Question 4: Raises blood glucose levels through the breakdown of fat and protein Question 5: Antagonises the effect of parathyroid hormone to

Page 101: 2005-2010(March) Mrcog Part 1 Pastpapers Recall Mrcog Total Guide

minimise bone density loss Question 6: Stimulates the release of milk from the breast Question 7: Stimulates the urinary secretion of calcium

Options A. Common iliac artery B. External iliac artery C. Inferior epigastric artery D. Inferior vesical artery E. Internal iliac artery F. Middle rectal artery G. Ovarian artery H. Superior vesical artery I. Umbilical artery J. Uterine artery K. Vaginal artery Instructions: After a forceps delivery a 30-year-old primigravida sustains a primary post partum haemorrhage of four litres. Although the uterus appears well contracted the bleeding continues, and a decision is made to identify and treat the bleeding point radiologically. For each question posed below, choose the single most appropriate option from the above list. Each option may be used once, more than once or not at all.

Page 102: 2005-2010(March) Mrcog Part 1 Pastpapers Recall Mrcog Total Guide

Question 8: What vessel runs up the broad ligament? Question 9: Which other vessel arises from the uterine artery? Question 10: Which other vessel anastomoses with the uterine artery? Question 11: Which vessel does the uterine artery arise from? Question 12: Which vessel runs anterior and superior to the ureter?

A- vitamin A B- vitamin E C- vitamin D D- vitamin B12 E- vitamin B6 F- vitamin C G- folic acid H- riblflavine I- vitamin K J- thiamine 1- fat soluble vitamin sysnthesised in the intestinal wall from the beta carotene . 2- fat soluble vitamin synthesised by large intestinal bacteria . 3- deficiency of this vitamin in childhood causes rickets . 4- fat soluble vitamin synthesis by the kidney is regulated by parathyroid hormone . 5- deficiency of this vitamins typicaly occurs in women with hyperemesis gravidarum . 6- water soluble vitamin with anti-oxidant effects . 7- absorption of this vitamin requires the presence of

Page 103: 2005-2010(March) Mrcog Part 1 Pastpapers Recall Mrcog Total Guide

intrinsic factor . 8- deficiency of this vitamin causes megaloblastic anaemia and neurological disorders . 9- fat soluble vitamin produced in the skin by photo-activiation of 7- dehydrocholesterol . 10- fat soluble vitamin with anti-oxdant effects . 11- fat soluble vitamin deficiency associated with raised serum alkaline phosphatase . 12- deficiency of this vitamin associated with osteomalasia . 13- maternal intake of this vitamin around the time of coception has been shown to reduce incidance of neural tube defects . 14- fat soluble vitamin , deficiency occur within few a days of cessation of bile secretion . 15- this vitamin is a cofactor in synthesis of prothrombin .

A- Candida spp B- Herpes simplex virus C- Human immunodeficiency virus D- Gardnerella vaginalis E- Chlamydia trachomatis F- Human papilloma virus G- Treponema pallidum H- Neisseria gonorrhoeae I- Trichomonas vaginalis Select the micro-organism described in the following cases:

Page 104: 2005-2010(March) Mrcog Part 1 Pastpapers Recall Mrcog Total Guide

1- An obligate intracellular gram negative bacterium .... 2- The cause of lymphogranuloma venereum ......... 3- Neutrophils containing gram negative diplococci .....

A- Double Y Syndrome B- Down’s Syndrome C- Edward’s Syndrome D- Fragile X Syndrome E- Klinefelter’s Syndrome F- Patau’s Syndrome G- Turner’s Syndrome Please select the most appropriate syndrome from the options below. You may use each option once, more than once, or not at all. 1- An infant girl is born with a webbed neck, hypertension and audible ejection systolic murmur. 2- A neonate boy is born with cleft lip and palate, low set ears and polydactyly. The child survives for only 3 weeks. 3- An infant boy is born with hypotonia, epicanthic folds and single palmer creases.

A- Adrenal arteries B- Celiac trunk C- Common iliac arteries D- Gonadal (ovarian or testicular) arteries E- Inferior mesenteric artery F- Lumbar arteries

Page 105: 2005-2010(March) Mrcog Part 1 Pastpapers Recall Mrcog Total Guide

G- Median sacral artery H- Renal arteries I- Superior mesenteric artery Select the most appropriate option: 1- Gives rise to the left gastric, splenic and hepatic arteries. 2- Gives rise to the intestinal, middle colic and right colic arteries. 3- Gives rise to the left colic, sigmoid and superior rectal arteries. 4- An unpaired branch that arises from the aortic bifurcation. 5- Arises from the aorta between the level of L1 and L2 vertebrae, and forms five segmental arteries that do not anastomose.

A- Candida spp B- Chlamydia trachomatis C- Herpes simplex virus D- Human immunodeficiency virus E- Gardnerella vaginalis F- Human papilloma virus G- Neisseria gonorrhoeae H- Treponema pallidum I- Trichomonas vaginalis Select the micro-organism described in the following cases: 1- A thin motile, gram negative spiral shaped bacterium ...

Page 106: 2005-2010(March) Mrcog Part 1 Pastpapers Recall Mrcog Total Guide

2- An anaerobic flagellated protozoan ... 3- A single strand RNA virus ..

A- ACTH B- CRH C- Dopamine D- FSH E- GnRH F- Growth Hormone G- LH H- Prolactin I- Somatostatin J- TSH Please select the correct hormone from the options for each of the descriptions below. You may use each option once, more than once, or not at all. 1- This hormone acts on cartilage and liver to release IGF-1... 2- This hormone promotes iodination of tyrosine residues... 3- This hypothalamic hormone inhibits the secretion of growth hormone... 4- In males, this hormone facilitates the generation spermatozoa... 5- This hormone inhibits galactorrhoea...

B- A- Deep inguinal B- Inferior mesenteric C- Internal iliac D- Para-aortic E- Superficial inguinal Lymph from the following structures passes to the above group of lymph nodes first. Please choose the most appropriate answer

Page 107: 2005-2010(March) Mrcog Part 1 Pastpapers Recall Mrcog Total Guide

from the list above. 1- Rectal . 2- Vulva . 3- Hallux . 4- Cervix . 5- Upper anal canal .

A- Bias B- Correlation C- Error D- False positive E- False negative F- Null hypothesis G- Power H- Regression I- Sensitivity J- Specificity Which of the above statistical terms is described by the following statements: 1- A random source of inaccuracy... 2- A type 2 error... 3- Ability to exclude a true negative...

A- Clostridium perfringens B- clostridium difficile C- Escherichia coli D- Haemophilus influenzae E- Staphylococcus aureus F- streptococcus pyogenes G- none of the above

Page 108: 2005-2010(March) Mrcog Part 1 Pastpapers Recall Mrcog Total Guide

Select the most likely causative organism for the following infections. 1- Chronic osteomyelotis after implant surgery .. 2- Gas gangrene .. 3- Pseudomembranous colitis ..

A- Mean B- Median C- Mid-range D- Mode E- Range F- Spread G- Standard Deviation H- Standard Error Select the appropriate term from the list for the following definitions. Each answer may be used once, more than once, or not at all. 1- Most frequent value .. 2- Spread of estimates of sample means around the true population mean . 3- The measure of spread of values around the mean . 4- The mid value when all values are listed in ascending order . 5- The sum of all the values divided by the number of values .

A- endothelial growth factor B- human chorionic gonadotrophin C- human placental lactogen D- IGF-1

Page 109: 2005-2010(March) Mrcog Part 1 Pastpapers Recall Mrcog Total Guide

E- Insulin F- leptin G- oestrogen H- progesterone I- relaxin Select the most appropriate hormone for the following statements. 1- The hormone that is the basis of most pregnancy tests. 2- Hormone produced by synctiotrophoblast to regulate nutrient storage in the final stages of pregnancy. 3- Hormone that is secreted by decidual cells of the placenta . 4- The hormone that is lactogenic, regulates glucose metabolism and promotes fat breakdown .

PCR Fish test Chromosomal linkage analysis Telomer analysis Sum hereditary crosslinkage chromosomal option (dnt remember) Screen for some known gene mutation Sweat test Saliva test -A family with one kid presumed to have cystic fibrosis however cftr gene mutation was not detected. Family members willing to give samples if needed.mother wants definite diagnosis as she is 11wks and wants to know diagnosis for her baby.parents are cousins with 1st degree relatives having cystic fibrosis.

Page 110: 2005-2010(March) Mrcog Part 1 Pastpapers Recall Mrcog Total Guide

1- Invasive diagnostics can be applied to reach final results by? ??? 2- you have Guthrie spot of their son how will u diagnose him for cystic fibrosis? PCR??? 3- families in UK with cystic fibrosis how will u find f508? SWEAT TEST????

5) A- Endoplasmic reticulum B- Golgi apparatus C- Lysosomes D- Microtubules E- Mitochondria F- Nucleolus G- Plasma membrane H- Ribosomes Select the most appropriate organelle that matches the following descriptions 1- Contains enzymes capable of digesting cells and cellular material c 2- 'Reads' the mRNA and builds protein . h 3- Modification of lipids and proteins with storage of material prior to export out of the cell. a (6) screening test for following 1- syphilis treponemal antibody test 2- syphilis hemaglutination test 3- syphilis immobilization test 4- HB-electrophorisis 5- mcv 6- mchc 7- MCh 8- serum iron 9- paul-bennel test

Page 111: 2005-2010(March) Mrcog Part 1 Pastpapers Recall Mrcog Total Guide

A- alpha thalessemia 4 B- folic acid deficiency 6 C- iron deficiency anemia 8 D- primary syphilis in early untreated period 1

(7) on taking a large protein meal? A- Decrease decrease B- Increase increase C- Increase decrease D- Deacrease increase E- Nochange no change F- Nochange increase G- Nochange decrease Insulin & Glucagon excretion what will happened ...? both will increase in secretions… (8) a- 3-hydroxyisovaleric acid . b- 17-alpha hydroxyprogesterone . c- 17-delta hydroxyprogesterone . d- acetic acid . e- cortisol . The following level of certain metabolities in amniotic fluid changes significantly of the fetus has an inborn error of the metabolism .select the single metabolite from list of the options above whose level in amniotic fluid is altered by inheretiance disease in item below. congentiat adrenal hyperplasia b (9) A- Adrenaline B- Calcitonin

Page 112: 2005-2010(March) Mrcog Part 1 Pastpapers Recall Mrcog Total Guide

C Cholecystokinin D- Cortisol E- Glucagon F- Insulin G- Progesterone H- Somatostatin IT- estosterone Select the appropriate hormone from the list for the following structures that produce it. Each answer may be used once, more than once, or not at all. 1- Adrenal Cortex . d 2- Adrenal Medulla . a 3- Pancreatic alpha cell . e 4- Pancreatic D Cells . h

A- WARFARIN . B- HEPARIN . C- AMPICILLIN . D- METHYLDOPA . E- PENCILLIN . F- CARBIMAZEPINE . G- METRONIDAZOLE . H- NON OF THE ABOVE . I- ALL OF THE ABOVE . For each of the following choose the single most appropirate statement from the above list of options . 1- Drug contraindicated in breast feeding . H 2- Drug does not cross the placenta B A. Calcitonin B. Cortisol

Page 113: 2005-2010(March) Mrcog Part 1 Pastpapers Recall Mrcog Total Guide

C. Glucagon D. Growth hormone E. Insulin F. Oestradiol 17-¦Â¦Â G. Oxytocin H. Parathyroid hormone I. Prolactin J. Thyroxine Instructions: For each action described below, choose the single most likely causative hormone from the above list of options. Each option may be used once, more than once, or not at all. Question 3: Stimulates deposition of cartilage at the ends of bones GH Question 4: Raises blood glucose levels through the breakdown of fat and protein Insulin Question 5: Antagonizes the effect of parathyroid hormone to minimize bone density loss calcitonin

Page 114: 2005-2010(March) Mrcog Part 1 Pastpapers Recall Mrcog Total Guide

Question 6: Stimulates the release of milk from the breast Oxytocin Question 7: Stimulates the urinary secretion of calcium calcitonin..? B. Options A. Common iliac artery B. External iliac artery C. Inferior epigastric artery D. Inferior vesical artery E. Internal iliac artery F. Middle rectal artery G. Ovarian artery H. Superior vesical artery I. Umbilical artery J. Uterine artery K. Vaginal artery Instructions: After a forceps delivery a 30-year-old primigravida sustains a primary post partum haemorrhage of four litres. Although the uterus appears well contracted the bleeding continues, and a decision is made to identify and treat the bleeding point radiologically. For each question posed below, choose the single most appropriate

Page 115: 2005-2010(March) Mrcog Part 1 Pastpapers Recall Mrcog Total Guide

option from the above list. Each option may be used once, more than once or not at all. Question 8: What vessel runs up the broad ligament? J Question 9: Which other vessel arises from the uterine artery? K Question 10: Which other vessel anastomoses with the uterine artery? G Question 11: Which vessel does the uterine artery arise from? E Question 12: Which vessel runs anterior and superior to the ureter? J C. Options : A- vitamin A B- vitamin E C- vitamin D D- vitamin B12 E- vitamin B6 F- vitamin C G- folic acid H- riblflavine I- vitamin K J- thiamine 1- fat soluble vitamin sysnthesised in the intestinal wall from the beta carotene . VIT A 2- fat soluble vitamin synthesised by large intestinal bacteria . VIT K 3- deficiency of this vitamin in childhood causes rickets . VIT D 4- fat soluble vitamin synthesis by the kidney is regulated

Page 116: 2005-2010(March) Mrcog Part 1 Pastpapers Recall Mrcog Total Guide

by parathyroid hormone . VIT D 5- deficiency of this vitamins typicaly occurs in women with hyperemesis gravidarum . THIAMINE 6- water soluble vitamin with anti-oxidant effects . VIT C 7- absorption of this vitamin requires the presence of intrinsic factor . VITB 12 8- deficiency of this vitamin causes megaloblastic anaemia and neurological disorders . B 12 9- fat soluble vitamin produced in the skin by photo-activiation of 7- dehydrocholesterol . VIT D 10- fat soluble vitamin with anti-oxdant effects . VIT E 11- fat soluble vitamin deficiency associated with raised serum alkaline phosphatase . ??? 12- deficiency of this vitamin associated with osteomalasia VIT D. 13- maternal intake of this vitamin around the time of coception has been shown to reduce incidance of neural tube defects . FOLIC ACID 14- fat soluble vitamin , deficiency occur within few a days of cessation of bile secretion . VIT K 15- this vitamin is a cofactor in synthesis of prothrombin VIT K

D. A- Candida spp B- Herpes simplex virus C- Human immunodeficiency virus D- Gardnerella vaginalis

Page 117: 2005-2010(March) Mrcog Part 1 Pastpapers Recall Mrcog Total Guide

E- Chlamydia trachomatis F- Human papilloma virus G- Treponema pallidum H- Neisseria gonorrhoeae I- Trichomonas vaginalis Select the micro-organism described in the following cases: 1- An obligate intracellular gram negative bacterium ...T VAGINALIS 2- The cause of lymphogranuloma venereum ........C TRACHOMATIS. 3- Neutrophils containing gram negative diplococci NESISSRIA - Double Y Syndrome B- Down’s Syndrome C- Edward’s Syndrome D- Fragile X Syndrome E- Klinefelter’s Syndrome F- Patau’s Syndrome G- Turner’s Syndrome Please select the most appropriate syndrome from the options below. You may use each option once, more than once, or not at all. 1- An infant girl is born with a webbed neck, hypertension and audible ejection systolic murmur. TURNERS 2- A neonate boy is born with cleft lip and palate, low set ears and polydactyly. The child survives for only 3 weeks. EDWRDS 3- An infant boy is born with hypotonia, epicanthic folds and single palmer creases DOWN - A- Adrenal arteries B- Celiac trunk C- Common iliac arteries

Page 118: 2005-2010(March) Mrcog Part 1 Pastpapers Recall Mrcog Total Guide

D- Gonadal (ovarian or testicular) arteries E- Inferior mesenteric artery F- Lumbar arteries G- Median sacral artery H- Renal arteries I- Superior mesenteric artery Select the most appropriate option: 1- Gives rise to the left gastric, splenic and hepatic arteries. COELAIC 2- Gives rise to the intestinal, middle colic and right colic arteries. SUP MESENTERIC 3- Gives rise to the left colic, sigmoid and superior rectal arteries. INF MESESNTERIC 4- An unpaired branch that arises from the aortic bifurcation. MEDIAN SACRAL 5- Arises from the aorta between the level of L1 and L2 vertebrae, and forms five segmental arteries that do not anastomoses LUMBAR

For the pregnancies at risk of the following conditions (1-8) , select the most appropriate prenatat diagnostic investigation from the alternatives (a-e). conditions: ---------------- 1- spina bifida. 2- duchenne muscular dystrophy. 3- cystic fibrosis. 4- congenital heart disease. 5- sever osteogenesis imperfecta. 6- fetal rubella . 7- anonychia .

Page 119: 2005-2010(March) Mrcog Part 1 Pastpapers Recall Mrcog Total Guide

8- down syndrome Alternatives: -------------------- a- karyotyping of the fetal amniocytes . b- fetoscopy . c- DNA analysis of chorionvillus sample . d- ultrasonography . e- fetal blood sampling .

Which of the following viruses are associated with the following conditions 1- Herpes simplex type 2 . 2- Human papillomavirus types 16 and 18 . 3- Hepatitis E . 4- Echoviruses . 5- Cytomegalovirus . a- cervical intraepithelial neoplasis (CIN) . b- severe and often fatal maternal infection in pregnancy in developing counteries . c- outbreaks of infection in nurseries for newborn infants . d- neonatal infection acquired via fetal scalp monitoring . e- persistent and generalised fetal infection .

For each complication (a-d) select the drug (i-v) most likely to be the cause : a- Cystitis . b- Cardiac toxicity . c- Pulmonary fibrosis .

Page 120: 2005-2010(March) Mrcog Part 1 Pastpapers Recall Mrcog Total Guide

d- renal toxicity .[/i] [i]i- Vincristine . ii- Bleomycin . iii- cis platinum . iv- Cyclophosphamide . v- Doxorubicin .

For each drug (A-B) select the appropriate description(i-v): A- 5-Fluorouracil (5FU) . B- Cyclophosphamide . i- Pyridine of purine analogue . ii- Alkylating agent . iii- Folic acid analogue . iv- Plant alkaloid . v- Antibiotic .

Match the following (A - E) to five of the statements below.. DURING THE MENSTRUAL CYCLE ; A- follicle stimulating hormone . B- oestradiol . C- progesterone . D- the first meitotic division . 1- is completed following the LH surge . 2- is produced by the adrenal gland . 3- is completed during the neonatal period . 4- is inhibited by oestradiol . 5- decereases mid - cycle . 6- is a precursor of oestradiol .

Page 121: 2005-2010(March) Mrcog Part 1 Pastpapers Recall Mrcog Total Guide

7- is inhibited by GnRH . 8- is produced throughout the cycle . 9- is produced in the secretory phase . 10- is secreted by the hypthalamus.

Match five of the following hormones / steriods to the statements below : a- Dihydroepiandrostenedione . b- FSH . c- GnRH . d- Insulin . e- LH . f- Oestradiol . g- Progesterone . h- Prolactin . I- Testosterone . j- Thyroid stimulating hormone (TSH) . 1- is produced in a pulsatile fashion from the hypothalamus . 2- is increased in the postpartum period . 3- is produced predominantly by the thecal cells in women . 4- is the pituitary hormone that is stimulated by low oestradiol concentrations . 5- is produced predominantly by the adrenal gland in women

Match the following (A - E) to five of the statements below.. DURING THE MENSTRUAL CYCLE ; A- follicle stimulating hormone .-------------------------------------------4 B- oestradiol .---------------------------------------------------------------8 C- progesterone .-----------------------------------------------------------9 D- the first meitotic division .----------------------------------------------1 E- testosterone .------------------------------------------------------------6

Page 122: 2005-2010(March) Mrcog Part 1 Pastpapers Recall Mrcog Total Guide

1- is completed following the LH surge . 2- is produced by the adrenal gland . 3- is completed during the neonatal period . 4- is inhibited by oestradiol . 5- decereases mid - cycle . 6- is a precursor of oestradiol . 7- is inhibited by GnRH . 8- is produced throughout the cycle . 9- is produced in the secretory phase . 10- is secreted by the hypthalamus. Testosterone is produced by the thecal cells and converted to ostradiol by aromatase. Oestradiol is secreted throughout the menstrual cycle initially by granulosa cells in the developing follicle and then by the corpus luteum ,Progesterone is produced by the corpus luteum , changing the endometrium from proliferative to secretory . The LH surge triggers the final stage of the first meiotic division whilst fertilisation causes the second meiotic division with the extrusion of the polar body by uneven division of the cytoplasm GnRH is secreted by the hypothalamus in a pulsatile manner and stimulates the production and release of LH and FSH in the anterior pitutary gland . Androgen precursors are secreted by the adrenal but oestradiol is only produced by the overy ....

Sep. 2009 Options 1. oval inlet transversal diameter longer than anterposterior . 2. oval inlet anterposterior dia longer than trans. 3. heart shape inlet anterposterior dia. longer than trans. 4. heart shape inlet trans. diam. longer than anterposterior 5. rounded inlet .. T>A. 6. rounded inlet ..A > T Qus. a. Gynaecoid pelvis. b. Android pelvis .

Page 123: 2005-2010(March) Mrcog Part 1 Pastpapers Recall Mrcog Total Guide

------------------------------------------------------------ a. Adenine monophosphate Guanine monophosphate Cytosine monophosphate duple helix Thymine monophosphate b. Adenine monophosphate Guanine monophosphate Cytosine monophosphate duple helix Uracil monophosphate c. Adenine monophosphate Guanine monophosphate Cytosine monophosphate single helix Uracil monophosphate d. Adenine monophosphate Guanine monophosphate Cytosine monophosphate single helix Thymine monophosphate e. Diadenine monophosphate Diguanine monophosphate Dicytosine monophosphate duple helix Diuracil monophosphate f. Diadenine monophosphate Diguanine monophosphate Dicytosine monophosphate duple helix Dithymine monophosphate g. DiAdenine monophosphate DIGuanine monophosphate DICytosine monophosphate single helix DIUracil monophosphate

Page 124: 2005-2010(March) Mrcog Part 1 Pastpapers Recall Mrcog Total Guide

h. DIAdenine monophosphate DIGuanine monophosphate DICytosine monophosphate single helix DIThymine monophosphate 1. DNA. 2. mRNA 3. Nucliac acid o HPV.

Page 125: 2005-2010(March) Mrcog Part 1 Pastpapers Recall Mrcog Total Guide

MRCOG part 1 RECALLS

SEPTEMBER 2008

Paper 1 & 2 EMQs In a country wide survey, 10 general practices were picked at random and 5% of patients were selected at random from each practice: Ta) the sample of patients is a true random sample Tb) all practices had an equal chance of selection Tc) all patients had an equal chance of selection Fd) two siblings could not have been selected Te) inferences about all patients in the country may be drawn from the sample A lady is about to deliver and you are about to give her a pudendal block..

a) ant division of ant. remi of S2-4 b) ant division of post.rami of S2-4 b)genitofemoral nerve c) post division of ant. remi of S2-4 d) post division of post. remi of S2-4 e) sensory supply of perineal nerve f) inferior rectal nerve g) ischial tuberosity h)ischial spine

Q1 -wht is the root value of pudendal nerve? Q2- wht nerve supplies lower part of vagina (I think)? Q3 –why do u give local skin infiltration before episiotomy?

If we take the day of fertilization as Day’0’ then…

Day2 Day4 Day 8 Day 10 Day12 Day 14

Page 126: 2005-2010(March) Mrcog Part 1 Pastpapers Recall Mrcog Total Guide

Day 18 Day20 Day 22 Day 24 Day 26 Day 42 Day 56 Day 70

Q4-Which represents 4 cell stage ? Q5- conceptus implants completely? Q6-feable hrt pulse seen on ultrasound? Q7- vertebra form completely?

Table: on different values given for deaths due to maternal mortality with direct and indirect causes as well as # of days from delivery to the cause of death

Deaths due to placenta previa and hypovolemia x @ e.g 20 days,10 days,8days etc suicide(with previous hx of psychiatric illness) crash victim diabetic neuropathy two more causes given with values

Different numbers were given in options

Q8-the number of late maternal deaths due to indirect causes? Q9-number of maternal deaths due to direct causes?

Calculate Numbers needed to treat from a drug trial on osteoporosis: (Q frm EMQ book)

Options

Page 127: 2005-2010(March) Mrcog Part 1 Pastpapers Recall Mrcog Total Guide

25 50 100 250 300 400 1000 others

Q10-N1=10,n2=5,n3=9990,n4=9995 Q11-N1=50,N2=25,n3=9950,N4=9975

FSH Oxytocin GH Prolactin Rennin ACTH etc

Q12-which is a protein with alpha and beta subunits ? Q13 – which stimulates mineralocorticoid activity/relsease? Q14- which hormone is produced by posterior pituatory?

Folic acid Magnesium Calcium Vit b1-B12 given

Q15- which vitamin is required in 400mcgs at preconception? Q16-which vitamins absorption is hampered by oxalic acid & phytates? Insulin & Glucagon excretion

Decrease decrease Increase increase Increase decrease Deacrease increase

Page 128: 2005-2010(March) Mrcog Part 1 Pastpapers Recall Mrcog Total Guide

Nochange no change Nochange increase Nochange decrease Etc

Q-17-on taking a large protein meal?

Folowing are risk factors for cancer

Q-18- HCC – ans Hep B

Q19- Skin cancer-ans UV radiation

Q20- Cervical/uterine cancer-ans HPV

Q21- Breast cancer-ans BRCA1 mutation

(Repeats from march 2008)

Hormone receptors for following are located in

a) protein kinase nuclease receptor b) nucleae transcription receptor c) G-linked cell membrane receptor d) G-linked golgi apparatus receptor e) Protein complex intra cytoplasmic receptor f) Multiligand receptor in cell membrane etc g) Protein Tyrosine kinase activity receptor on cell membrane

Q22- Insulin Q23-PG E2 Q24-Estrogen Q25-Progesterone

Q26-A family with one kid presumed to have cystic fibrosis however cftr gene mutation was not detected. Family members willing to give samples

Page 129: 2005-2010(March) Mrcog Part 1 Pastpapers Recall Mrcog Total Guide

if needed.mother wants definite diagnosis as she is 11wks and wants to know diagnosis for her baby.parents are cousins with 1st degree relatives having cystic fibrosis. Invasive diagnostics can be applied to reach final results by? Q27-you have Guthrie spot of their son how will u diagnose him for cystic fibrosis? Q28- families in uk with cystic fibrosis how will u find f508? Options:

Pcr Fish test Chromosomal linkage analysis Telomer analysis Sum hereditary crosslinkage chromosomal option (dnt remember) Screen for some known gene mutation Sweat test Saliva test

screening test for following

Q29-alpha thalessemia Q30-folic acid deficiency Q31-iron deficiency anemia Q32-(one more i think!)

options

hbelectrophorisis mcv mchc MCh serum iron paul-bennel test etc

Q33-which is the commonest cause of non-dysjunction at meiosis? trisomy 21

Page 130: 2005-2010(March) Mrcog Part 1 Pastpapers Recall Mrcog Total Guide

Q-34 which defect is associated with 60-70% cleft palate and cleft palate? trisomy 13 or 18 or KF syndrome

all about cystic fibrosis

screening tests for a-thallesemia trait iron deficiency amemia,folic acid anemia and....EMQ

RECEPTORS FOR ESTROGEN,PROGESTERONE,PROSTAGLANDIN,INSULIN ....EMQ

NERVE SUPPLYING PERIANAL SKIN,PUDENDAL NERVE ORIGIN,LA DURING EPISIOTOMY IS GIVEN FOR?????......EMQ

EMQ receptors for insulin prostaglandins estogen progesteron

primary syphilis in early untreated period

options syphilis treponemal antibody test syphilis hemaglutination test syphilis immobilization test

emq traetment of chlamedia ..................bacterial vaginosis

emq pre malignant conditions emq statistics calculate n to be traetad emq diagnosis of iron defecincy anaemia folic acid .................. early untrated syphylis emq congenital adrenal h icreased ? SUBSTANCE

Page 131: 2005-2010(March) Mrcog Part 1 Pastpapers Recall Mrcog Total Guide

emq embryology 4 cells ? day vertebra formation

mcq about DIGITALIS, cyclosporine .mitochondrial dna ,cd8 cells , the uterer(2 mcq) ,3 repeated statistics mcqs, EMQ 36 WS PRIMI GRAVIDA had flu like sympt , the baby was born jaundiced , hepatosplenomegaly ,skIn rash , brady cardia ,,,,choose the cuase (all viruses ,bacteria,listeria,plasmodium )??????

EMQ ...why we do local infiltration + pudendal block

EMQ ...cause of post operative infection ??pseudomonas

EMQ .....WHICH vit or mineral absorbtion is affected by phytate (no iron in the choices

MCQ

citric acid cycle fatty acid oxidation ???? genetic imprinting ?????stylomastoid foramen spinal cord in smooth muscle cells /regeneration possible /actin &myosin present/lengh of fibers repeated 2 mcqs u/s and radiation effect apoptosis 2 mcqs about hyperplasia pelvic splanchnic nerve

Paper 1 & 2 EMQs A lady is about to deliver and you are about to give her a pudendal block.. a) ant division of ant. remi of S2-4 b) ant division of post.rami of S2-4

Page 132: 2005-2010(March) Mrcog Part 1 Pastpapers Recall Mrcog Total Guide

b)genitofemoral nerve c) post division of ant. remi of S2-4 d) post division of post. remi of S2-4 e) sensory supply of perineal nerve f) inferior rectal nerve g) ischial tuberosity h)ischial spine Q1 -wht is the root value of pudendal nerve? Q2- wht nerve supplies lower part of vagina (I think)? Q3 –why do u give local skin infiltration before episiotomy? If we take the day of fertilization as Day’0’ then… Day2 Day4 Day 8 Day 10 Day12 Day 14 Day 18 Day20 Day 22 Day 24 Day 26 Day 42 Day 56 Day 70 Q4-Which represents 4 cell stage ? Q5- conceptus implants completely? Q6-feable hrt pulse seen on ultrasound? Q7- vertebra form completely? Table: on different values given for deaths due to maternal mortality with direct and indirect causes as well as # of days from delivery to the cause of death Deaths due to placenta previa and hypovolemia x @ e.g 20 days,10 days,8days etc suicide(with previous hx of psychiatric illness) crash victim diabetic neuropathy

Page 133: 2005-2010(March) Mrcog Part 1 Pastpapers Recall Mrcog Total Guide

two more causes given with values Different numbers were given in options Q8-the number of late maternal deaths due to indirect causes? Q9-number of maternal deaths due to direct causes? Calculate Numbers needed to treat from a drug trial on osteoporosis: (Q frm EMQ book) Options 25 50 100 250 300 400 1000 others Q10-N1=10,n2=5,n3=9990,n4=9995 Q11-N1=50,N2=25,n3=9950,N4=9975 FSH Oxytocin GH Prolactin Rennin ACTH etc Q12-which is a protein with alpha and beta subunits ? Q13 – which stimulates mineralocorticoid activity/relsease? Q14- which hormone is produced by posterior pituatory? Folic acid Magnesium Calcium Vit b1-B12 given

Page 134: 2005-2010(March) Mrcog Part 1 Pastpapers Recall Mrcog Total Guide

Q15- which vitamin is required in 400mcgs at preconception? Q16-which vitamins absorption is hampered by oxalic acid & phytates? Insulin & Glucagon excretion Decrease decrease Increase increase Increase decrease Deacrease increase Nochange no change Nochange increase Nochange decrease Etc Q-17-on taking a large protein meal? Folowing are risk factors for cancer Q-18- HCC – ans Hep B Q19- Skin cancer-ans UV radiation Q20- Cervical/uterine cancer-ans HPV Q21- Breast cancer-ans BRCA1 mutation (Repeats from march 2008) Hormone receptors for following are located in a) protein kinase nuclease receptor b) nucleae transcription receptor c) G-linked cell membrane receptor d) G-linked golgi apparatus receptor e) Protein complex intra cytoplasmic receptor f) Multiligand receptor in cell membrane etc g) Protein Tyrosine kinase activity receptor on cell membrane Q22- Insulin Q23-PG E2 Q24-Estrogen Q25-Progesterone

Page 135: 2005-2010(March) Mrcog Part 1 Pastpapers Recall Mrcog Total Guide

Q26-A family with one kid presumed to have cystic fibrosis however cftr gene mutation was not detected. Family members willing to give samples if needed.mother wants definite diagnosis as she is 11wks and wants to know diagnosis for her baby.parents are cousins with 1st degree relatives having cystic fibrosis. Invasive diagnostics can be applied to reach final results by? Q27-you have Guthrie spot of their son how will u diagnose him for cystic fibrosis? Q28- families in uk with cystic fibrosis how will u find f508? Options: Pcr Fish test Chromosomal linkage analysis Telomer analysis Sum hereditary crosslinkage chromosomal option (dnt remember) Screen for some known gene mutation Sweat test Saliva test EMQ 1 ST PAPER WHICH NERVE SUPPLIES PERIANAL SKIN EMQ 2 CYSTIC FIBROSIS a - invasive testing will be done ( she is preg 11 ws) which test b- their child blood is ready for guathri test ,which test you will do c- the relatives are willing to be tested , which test will reveal the 405 mutation for families in great braiten any one have suggetions

more options were given for these qs karyotyping comparative genomic hybridization flourecent ???? MCQ

Page 136: 2005-2010(March) Mrcog Part 1 Pastpapers Recall Mrcog Total Guide

propranolol smooth endoplasmic reticulum anal canal smooth muscles (2 mcq ,paper1 ,2 ) primitive streak fetal circulation ??microtubules digitalis mcq ; the following are bacteria cryptococcus ,candida,histoplasmosis, mcq : the embryological origin of peritoneum,.... mcq mitosis miosis mcq : the follwing are structral abnormalities of chromosomes mcq : ?..................increase with preg mcq : drug interaction methotrexate , sulphonamides

mcq :arterial and mean blood pressure mcq : causes of alkalosis mcq : the folloing are types of cells with their secretion ? endothelial cells heparin ?mast cells igE ? plasma celle (i am not sure of the exact coupling)

mcq at 20 ws gestation rbcs are made in liver lymph nodes spleen bone marrow mcq about pudendal nerve mcq ischiorectal fossa mcq ureter in abdomen mcq ureter in pelvis mcq foramen of winslow (in detailes)

Page 137: 2005-2010(March) Mrcog Part 1 Pastpapers Recall Mrcog Total Guide

mcq kallman syndrome in men mcq hcg mcq trophoplasts mcq ? all types of cells in islets of langerhans and their products mcq the following are premalignant conditions: paget disease ...(repeat mcq endotoxins or exotoxins ( i am not sure) MCQ CORPUS LUTEUM mianly endothelial clls secerets inhibin function decrease after 12 ws

MCQ INHIBIN

MARCH 2008

cystic fibrosis guthic spot test receptors in apoptosis caspa arias stella reaction blood picture in PET and in preg is inflam like

1.EMQ paclitaxel carboplatin side effects 2.EMQ complex partial seiizure gum hypertrophy,acne ,facial coarsening,vit K TO BE given..?drug used 3.EMQ oestrogen progesterone receptor site 4.EMQ on vitamins-xerophthalmia..morning sickness..macrocytic anaemia. 5.MCQ rubella incubation period,specific immunity within 15 days.. 6.EMQ HEPATOCELLULAR carcinoma,..?? 7.EMQ lining epithelium of ureter,..?? 8. EMQ montoux rection.. 9.MCQ ovulatory DUB 10.MCQ concerning aneurysm- 10% due 2 inflam.,syphilitic aneu,thoracic vessels,marfan recessive

Page 138: 2005-2010(March) Mrcog Part 1 Pastpapers Recall Mrcog Total Guide

if a mother has a child with cystic fibrosis ,she is preg and will do amniocentesis a. which test to be done (every possibility mentioned) b. all relatives will volunteer to do gutic spot test ,whichtest you will carry on the blood

-graph with hormones of the endometrial cycle - -citric acid cycle (graph)

microsatellites alleles - cd4 -nk cells -down syndrome -pulmonary embolism anal canal -vagina -ovary

phenylketonuria inhibin passive transport erythropoietin renin hcg levels syphylis rubella chlamydia hormone dependent malignancies(testicular carc._sertoli leydig-clear cellcarc.ofkidney-thyroid carc.) pigd

missed abortion MCQ congenital heart emq crown \heel lenghth at birth crown/rump length 6 cm =? weeks

Page 139: 2005-2010(March) Mrcog Part 1 Pastpapers Recall Mrcog Total Guide

about 10 questions statistics plrese does anyone remember the q about ring Y chromosome q about sattellite alleles ??

passive diffussion not depend on . concenteration gradiant molecular size 2following tissue are capable of regeneration spinal cord liver epedermis myocardium bone marrow 3 double blind trial delliosion of HCG in the urine pregnancy test

about the q APOPTOSIS and capsase ,q methods of disinfection and BOWIE DICK test where is source to study these subjects? q progesterone receptors q about progesterone receptors

syphilis , toxoplasma , hpv , immune responses EMQ type of hpv causing benign wart type of hpv causing cancer cx MCQ down syndrome associated with duodenal atresia true associated with ambig genetalia false only maternal chromosome F

Page 140: 2005-2010(March) Mrcog Part 1 Pastpapers Recall Mrcog Total Guide

only paternal chromosome F MCQ oogonia miosis starts at puberty miosis before mitosis during S phase chromosomes are doubled the most diagnostic test to be done on amniocentesis gautic spot test what test you do with the blood sample

emq about vitamins (hyperemesis) i wrote vit B6 and (xerophthalmia) q about endometrial hyperplasia q about side effect of drug (carboplatin)-(taxal) q about arias stella if its specific in pregnancy q about chlamydia q about varicella q about hpv ,wart answers 16 or 18 or 6,11 q about hcg conc in pregnancy citric acid cycle diagram hormones diagram fsh,lh,estrogen,testosteron q about:by transvaginal u/s yolk sac apear in which week many questions in genetic about pcr,fish test q about laser q in pathology about totipotent cell,tumour like mass q about voiding presure q about ca requirement in pregnancy q about test used to diagnose folic acid deficency q about thalasthmia q about cervical ectropion cells i choose columner cell - mri contraindicated in early pregnancy q about stallete instability in dna - transverse abdominus ms attached to lumbar transverse process -cut s1 lead to autonomic bladder

Page 141: 2005-2010(March) Mrcog Part 1 Pastpapers Recall Mrcog Total Guide

q about cancer which are hormone dependent -breast -prostate -thyroid q about guthre test q about cystic fibrosis and diagnosis -swet test -karyotyping-fish test q about disinfection and sterelization q about complication of contraceptive pills

emqs 1lining of ureter 2.erythropoeitin and renin q from john duthie 3complications of cisplatin 4 paclitaxel 5statistics -std error calculation 7 mean in a normal distribution 8occipito frontal diametr 9hcg titre at detection by tvs 10 at 3 days aftr fertilization 11crl in scan is 60mm what is gestational age 12crl at term ? 13 parietal sutur is between 14kuffer cells are -phagocytic 15 estradiol receptors 16 progestrone receptors -we know that thes are intra nuclear but there were 2 confusin options with intranuclear plz refer this in depth , one option had intranuclear kinase or somethin 16anticonvulsant with description of phenytoin 17vitamin deficiency causin hyperemesis 18 - macrocytic anemia 19 -xerophtalmia 20 cystic fibrosis 3 emqs 24 citric acid cyle diagrm , enzymes were options complement causes 26 b thalasemia detection 27 folate def detection

Page 142: 2005-2010(March) Mrcog Part 1 Pastpapers Recall Mrcog Total Guide

29 basal cell ca - local malignant i think 30tumor like –hamaroma March 2007

Copper IUD mode of action: sperm motility microthrombi MRI: ionising radiation? no adverse effect on fetus? detect fetal cardiac abn? USG: probe: array of magnets? doppler to detect fetal heart movement? facial hair increased by: oestrogen? testerone? spirolactone? characterics: pass thro' origin? slope = 0? pass thro the mean? unaffected by changing scale? unaffected by changing dependent variable?

Copper containing IUDs: F should be changed every year F have a high incidence of actinomycoces colonization than plastic devices T Cause a relative increased in ectopic pregnancy T have been implicated as a cause of fatal infection in pregnancies F do not cause menorrhagia Comments: Inert devices can be left in place until the menopause, but copper devices need renewal every 3-5 years, depending on the make, because of the gradual absorption of copper. Copper IUCDs produce local

Page 143: 2005-2010(March) Mrcog Part 1 Pastpapers Recall Mrcog Total Guide

concentrations of copper salts which apparently give some protection against bacterial contamination. Pelvic infection with actinomyces organisms is most likely with a plastic device that has been in situ for some years. While the rate of intrauterine pregnancy is reduced, that of ectopics is not. Hence, there is a relative increase in ectopic pregnancy after IUCD insertion. If an IUCD is left in place there is a slight risk of intrauterine infection, preterm labour and antepartum haemorrhage, but most pregnancies are uncomplicated and the device is delivered with the placenta. Increased menstrual loss may be caused by increased fibrinolytic activity which occurs round the IUCD. The progestogen intrauterine system (IUS) reduces menstrual flow and often dysmenorrhoea. Which of the following is/are true concerning MRI? A It involves ionizing radiation (False) B Has no recognised side effects on the foetus (True) C The pregnant mother should be turned to her left side during scanning (True) D Tissue with high hydrogen concentrations are difficult to distinguish False) E Blood vessels appear white on scanning (False) Comments: Water is a molecule composed of hydrogen and oxygen atoms. The nuclei of the hydrogen atoms are able to act as microscopic compass needles. When the body is exposed to a strong magnetic field, the nuclei of the hydrogen atoms are directed into order - stand "to attention". When submitted to pulses of radio waves, the energy content of the nuclei changes. After the pulse, a resonance wave is emitted when the nuclei return to their previous state. The small differences in the oscillations of the nuclei are detected. By advanced computer processing, it is possible to build up a three-dimensional image that reflects the chemical structure of the tissue, including differences in the water content and in movements of the water molecules. This results in a very detailed image of tissues and organs in the investigated area of the body. In this manner, pathological changes can be documented.[/color:febed2ebf4][/b:febed2ebf4]---------------------------------------------------------------------- Many stats about sensitivity, positive predictive value, accuracy, even the linear regression Acute UTI in pregnancy

Page 144: 2005-2010(March) Mrcog Part 1 Pastpapers Recall Mrcog Total Guide

rarely symptomatic ?F Copper IUD mode of action: ?T sperm motility ?T microthrombi ?T intervent ovary movement MRI: ionising radiation F no adverse effect on fetus T detect fetal cardiac abn ?F ( I think it will be very small) USG: probe: array of magnets? ?F doppler to detect fetal heart movement T facial hair increased by: oestrogen?F testerone?T spirolactone F ( I think it is used for treating hirsutism) Mycobacteria ?T strict ?? *(nor sure which word they use ) aerobic ------------------------------------------------------------------- 48---if a distribution of results is markedly skewed to the left(sep2000) a-the mean is same as 50th centile— f b-the same no of values lie on either side of the median- t c-the mode is equal to the median- f d-the students test should be used to compare this distribution with another- f e-logrithmic transformation of the results will produce a distribution close to normal-t -concerning the ablity of a test to predict disease a-sensitivity is the ablity to predict those with disease correctly T b--sensitivity is the same as positive predictive value F c-the confidence interval must cross 1 to prove significance t (NOT A MUST as CI at 1 is perfect ) d-an odds ratio of 1;3 implies the risk of 33% f e--an odds ratio of 2 indicates a halving of risk F

Page 145: 2005-2010(March) Mrcog Part 1 Pastpapers Recall Mrcog Total Guide

------------------------------------------------------------------------------------ 1-SUCCESSFUL LACTATION IS a-maintained by estrogen -- b-maintained by progesterone -- c-initiated by prolactin surge-- d-maintained by human placental lactogen e-inhibited by dopamine – 2-THE RELEASE OF CATECHOLAMINES FROM THE ADRENAL MEDULLA INCREASES a-during sleep— b-when the nerves to adrenal glands are stimulated— c-when the blood sugar rises— d- immediately following a myocardial infarction— e- in the presence of pheochromocytoma—T F-DURING ACUTE HAEMORRHAGE— 3- In the pathogenesis of thrombosis a-prostacyclin induces platelet aggregation— b-plateletes synthesis thromboxane A2-- c-thromboxane A2 induces vasoconstriction-- d-contact with subendothelial collagen causes platelete aggregation-- e-thrombin inhibits platelete aggregation— --4- CONVERSION OF GLUCOSE TO LACTIC ACID a-occurs in single enzymatic reaction— b-is the only pathway for synthesis of ATP in red blood cells— C-is a reversible process in skeletal muscle d- is inhibited by high cellular conc of ATP- e-occurs in skeletal muscle when availability of oxygen is limited- 5-GLUCOCORTICOIDS a- promotes hepatic gluconeogenesis- b- suppress uptake of glucose by muscle- c—promotes protein breakdown- d—promotes fat breakdown- e-increase glycolysis in adipose tissue- 6-Actinomycete israelii

Page 146: 2005-2010(March) Mrcog Part 1 Pastpapers Recall Mrcog Total Guide

1-Is a fungus— 2- forms yellow granules in pus – 3-is a commensal in mouth-- 4- is a commensal in vagina-- 5- is usually resistant to penicillin— 7-Following substances increase the serum uric acid concentration( a-colchicine-- b-chlorothiazide-- c-allopurinol-- d-probenecid-- e-phenylbutazone— 7--THE FOLLOWING ARE INHERITED AS AUTOSOMAL RECESSIVE CONDITIONS A. tuberous sclerosis. b-phenylketonuria-- C. ahondroplasia.-- D. sickle cell anaemia- E. Von Giek l disease.— 8-. Genes on sex chromosomes are responsible for the inheritance of ( A. glucose-&pbosphate dehydrogenase defiency. B. defective colour vision.— C. hairy ear rims. – D. homocystinuria. E. Hurler syndrome. 9-in human a haploid no of chromosomes is found in) a-red blood cells- b-blastocysts- c-primary oocytes-. d-the first polar body- e-spermatozoa- 10-osteoprosis is associated with a-an increase in uncalcified bone matrix(osteoid tissue b-prolonged oestrogen therapy—

Page 147: 2005-2010(March) Mrcog Part 1 Pastpapers Recall Mrcog Total Guide

c-a normal histological bone structure- d-bone fracture— e-irregularity of epiphyseal plates— 11-THE FOLLOWING HAVE an anti-emetic effect a-hyoscine bromide— b-morphine sulphate— c-chlorpropamide-- d-promethazine hydrochloride— e-perphenazine— 12CLOMIFENE CITRATE a- is an antiandrogen— b-does not stimulate ovulation directly— c-can produce visual disturbances-- d-is generally prescribed throughout the proliferative phase of the menstrual cycle— e-in the treatment of anovulation increases the risk of multiple pregnancy— 13--if a distribution of results is markedly skewed to the left a-the mean is same as 50th centile b-the same no of values lie on either siDe of the median c-the mode is equal to the median- d-the students test should be used to compare this distribution with another- e-logrithmic transformation of the results will produce a distribution close to normal- 15. when a man has hemophilia a. 50% of his daughters would not expected to be carriers – b. 25% of his sons expected to be carriers c. Good medical control of blood definition reduce the risk of this condition in his children d. His new born child is likely to require an urgent blood transfusion - e. His sister has 50% probability of being a carrier 16- THE INTERSTITIAL CELL (LEYDIG) OF THE TESTIS a--secrete seminal fluid— b-are stimulated by LH – c-secrete androgen binding protein—

Page 148: 2005-2010(March) Mrcog Part 1 Pastpapers Recall Mrcog Total Guide

d-secrete fructose – e-produce testosterone T in response to LH -- 17--CONCERNING VIRUSES a-the core of every virus contain RNA— b-they usually produce intracellelur toxin causing cell death c-antibodies are directed against capsular protein— d- they can be grown in intact cells-- e-interferone are synthetic antiviral substances— 18-THE FOLLOWING ARE CYTOTOXIC ALKYLATING AGENTS( a-cyclophosphamide— b-mercaptopurine-- c-chlorambucil-- d-fluorouracil-- e-methotrexate- 19-2,3 DIPHOSPHOGLYCERATE a-is present in higher conc in maternal erythrocytes than fetal erythrocytes-- b-binds more avidly to haemoglobin A than to haemoglobin E— c-increases the affinity of haemoglobin for oxygen— d-is a phospholipid e- is synthesised by the pentose phosphate pathway 18-HYPERKALEMIA IS A CHARACTERSTIC FINDING IN TINDALL a-primary aldosteronism b-treatment with Spironolactone- c-hyperparathyroidism- d-ACTH secreting tumours of bronchus- e-renal failure- 19- CONCERNING INHERITABLE DISEASES A-huntigdon ,s chorea is transmitted by a dominant gene— b-phenylketonuria is transmitted by a recessive gene— c-haemophillia ia an autosomal dominant condition-- d- Von Willebrand disease is a sex linked condition— e-cystic fibrosis is transmitted by an x linked recessive gene--

Page 149: 2005-2010(March) Mrcog Part 1 Pastpapers Recall Mrcog Total Guide

20-Early blood borne dissemination is a characteristic feature of: a) carcinoma of the endometrium – b) osteosarcoma – c) basal cell carcinoma – d) carcinoma of the cervix- e) choriocarcinoma- 21- in tissue pigmentation the following are associated a-kernicterus and conjugated billirubin b-addison disease and increased cutaneous melanin c-melanosis coli and bile pigments- d—wilson disease and copper deposition in the cornea- e-corpus leuteum and carotenoids- 22-acquired diverticular disease of the colon a-is present in at least 15% of Caucasian over the age of 50 years- b-is due to congenital abnormality of the bowel wall c-is associated with increased intraluminal pressure— d-is associated with muscular thickening— e-may result in intestinal obstruction— 23 stored blood which is to be used for transfusion— a-kept at -4 degree b-must be used in 1 week – c-is tested for compliment content before transfusion -- d-may be used for platelet replacement – e-contains an acid anticoagulant – 24- in uncomplicated homozygous beta thalasemia there is a-hypochromasia-- b-a reduction in haemoglobin A— c-an increase in haemoglobin F— d- RED CELL SICKLING e-presence of megaloblasts in bone marrow- 32-The following are capable of cellular regeneration a)spinal cord b)liver parenchyma – c)gut epithelium – d)kidney ???

Page 150: 2005-2010(March) Mrcog Part 1 Pastpapers Recall Mrcog Total Guide

e)bone marrow— 33-during the development of female reproductive system a-primordial germ cells arise in yolk sac-- b-ovarian development is dependent upon oestrogen activity-- c-the paramesonephric duct give rise to the cervix-- d-the greater vestibular glands arise from the urogenital sinus-- e-differentiation of external genitalia is dependent upon ovarian activity-- 36- IN CONGENITAL ADRENAL CORTICAL HYPERPLASIA a-commonest deficiency is C21 b- plasma cortisol conc is raised – c-there may be excessive secretion of 17 hydroxy progesterone-- d- sodium retension is characterstic – e-blood catecholamine conc are increased – 37-- SCHISTOSOMA HAEMATOBIUM a- is a snail – b- is prevalent in china— c- infestation may affect the uterine cervix d- give rise to chronic granulomatous lesions-- e- infestation predisposes to carcinoma— 38- KETONE BODIES . a-can be utilised by theadult brain—. B-Include acetone - c-are water soluble -- d- are synthesised in skeletal muscles— e-can be utilised during starvation— 39-IRON IONS a-diffuse passively into erythropoitic cells- b-bind to transferin- c-are taken up by hepatocytes- d- are necessary for cytochrome synthesis- e-are absorbed predominantly by the ileum- 40-- --the conjugation of billirubin

Page 151: 2005-2010(March) Mrcog Part 1 Pastpapers Recall Mrcog Total Guide

a-takes place in hepatocytes- b-is catalysed by UDP glucuronyl transferase- c-is inhibited by phenobarbitone- d-renders it water soluble- e-is impaired in acute billiary obstruction- 41- in radiotherapy( a-1Gray is eq to 1joule per kg b- the skin usually recieve a greater dose of radiation than underlying tissue- c-the major effect of radiation energy is to damage the cytoplasm of the cell- d-cells in tissue which are hypoxic are more vulnerable to radiation- e-radiation induced changes in tissues may take 6 weeks to develop- -42--osteomalacia is characterised by a-mineralization of the periostium – b-deposition of uncalcifiedbone matrix -- c-normal osteobastic activity -- d-increase capillary fragility - e-normal calcification of bone— 43-progesterone a-is a C21 compound b-synthesised by the ovary before ovulation- c-increases ventilation- d- raises BMR- e-binds to corticotrophin –binding globulin- 44-EPIDERMAL GROWTH FACTOR A- is mitogenic-. b- synthesis is stimulated by oestradiol- c-is a steroid molecule-F ... d- is found in endometrium-- e- binds to receptor on the nuclear membrane- 45-the following are structural aberrations of chromosomes: a. deletions -- b. inversions—

Page 152: 2005-2010(March) Mrcog Part 1 Pastpapers Recall Mrcog Total Guide

c. aneuploidy – d. . polyploidy- e translocations— 46-- Messenger RNA a-Synthesis is dependent on RNA polymerase – b-is an exact copy of sense DNA— c-Contains exons – d-Is measured by Western blotting – e-translation occurs in the nucleus 47-- BETA SYMPATHOMIMETIC DRUGS MAY— A- caue bronchospasm— b-reduce frequency of uterine contraction— c-cause heart block— d-reduce diastolic blood pressure— e- increase blood glucose conc 48- The following statistical statements are correct- a-in the normal distribution,the value of mode is 1.73 times the median –b-in a distribution skew to the right,the mean lies to the left of median- c-in the series- 2;7;5;2;3;2;5;8, the mode is 2 - d-student’s ‘t’ test is designed to correct for skew distribution – e-the chi squared testmay be used when data are not normally distributed-- 50-Contraindicated in breast feeding— A-POPs B-Bromocriptine- C-chloroqunine- d-?? e-warfarine—F 51-Contraindicated with kidney impairment A-Dopamine - B-Gentamycine – C-Cephahexin D-Cis platin 52- the following are characters of hypoxic cell death:

Page 153: 2005-2010(March) Mrcog Part 1 Pastpapers Recall Mrcog Total Guide

A-Apoptosis… -- B-Phagocytosis—… C-Pyknosis… -- D-Poikilocytosis… E-Release of phospholipids— 52- the following are characters of hypoxic cell death: A-Apoptosis… -- B-endocytosis C-Pyknosis… -- D-Poikilocytosis… E-koliosytosis prostagladins- a-lipooxygenase pathway b-thromboxane causes vasoconstriction c-increased in the myometruim d-? e-? linear regression: a-starts from the origin b-passes thru the mean- c-values vary d-ranking is done??? following test r used to compare 2 drugs a and b a-student t test b-meta analysis c-wiloxone paired test withney u paired test e? toxic shock syndrome associated with the use of tampoons b-due to toxigenic starinsof strep c-infrequently reported outside n america d-a consequence of previous antibioitic therapy e-confined to seualy active women the following can regenerate

Page 154: 2005-2010(March) Mrcog Part 1 Pastpapers Recall Mrcog Total Guide

a-kidneyepitheluim? b-pheriheral axons- c-liver parenchyme d-bone marrow e-bone radiation sensitive a-bone b-bone marrow c-epith of gut d-skin e-? Mycobacteruim a- are alchochol acid fast b-does not form spores c-d-a facultative anaerobes d-responsible for leprosy e-pathogenic inhumans all strains the germination of tetanus spores in a wound is inhibited by a-tissue trauma b-oxygen c-inject of toxoid d-injection of antitoxin e-removal of devitalised tissue Aldosterone a-reduces Na resorption in PCT b-reduces Na absorption in descen loop of henle c-Increase Na absortion in DCT Increase K loss from the tubule e increases Na absortion in collecting tubules the following r conclusive evidence of pregnancy in uterine cureetings???? can, t remmber a-decidua compacta- b-Arias -stella changes in endometrail glad- c-spiral arterioles d-plasma cell infiltration e chorionic villi

Page 155: 2005-2010(March) Mrcog Part 1 Pastpapers Recall Mrcog Total Guide

White cell migration from the bld vessels in areas of inflammation involves--- march 1997/2 21 a-call migration between the endothelial cells b-a pssive loss of fluid bld elements c-cell migration independeant of endothelial cell motion d-initail emigration of polymorhs neutrophils e-more polymorphs than monocytes after 2 day= the vulva is supplied by a-ant cutaneu erve of the thigh b-femoral br/o genito femoral nerve c-? d? e? glucocorticoids a-promote hepatic gluconeogenesis b-suppress uptake of glucose by muscles c-promote protein break down d-promote fat reakdown e-increase glycolysis halothane(refer sept 1997)42 a-cardic arrythmias b-explosive mixtures with air c-liver damage 53- U/S: A-Pulstile B-Increase body temp 1 degree after 15 min scanning.. C-High frequency penetrates deeper TISSUE,- D-Can distinguish between 2 points closer than 0.5 mm E-Best echoes are produded by beam at right angle to the structures. 54-Disinfectant solutions may become contaminated with: 1-Enterobacter species – 2-Streptococcus species - 3-Escherichia coli - 4-Pseudomonas aeruginosa - 5-Staphyloccus pyogenes-

Page 156: 2005-2010(March) Mrcog Part 1 Pastpapers Recall Mrcog Total Guide

55-Nitric oxide a) is synthesised in the endothelium – b) has a short half life -- c) causes smooth muscle contraction d) increases during pregnancy - e) combines with oxygen to produce L-arginine— but few question got modifications eg; nitrous oxide synthesised by macrophages naloxone antiemitic effect drugs contraindicated in breast feeding drugs causing renal impairment ultasound mri statistics quit difficult all definitions rna pre implantation diagnosis

1( FIBRINOGEN 2) HEPATOGLOGIN 3) FERRITIN 4) ...

predominantly produced by UG system - is a kind of lukotriene - is phospholipid

which reactant protein will increase in injury

--------------------------------------------------------------------------------

1) FIBRINOGEN 2) HEPATOGLOGIN 3) FERRITIN 4) albumin 5) ?

Cell cycle -prophase I and II

Page 157: 2005-2010(March) Mrcog Part 1 Pastpapers Recall Mrcog Total Guide

-mitosis I and II

Vulva nerve supply

Urinary bladder - lining/embrology

vagina - embrology

Fetal testis

IL I

MCH I

Function of Neutrophil polymorphs

Fetal HbF

PG

Umblical cord – embrology

prostaglandins half lifa 30 min f il1 produced by macrophages whartons jelly originate from extra embryonic endodem detrusor muscle is of mesodermal origin acute phase proteins are transferrin alfa feto protein preimplantation diagnosis

Page 158: 2005-2010(March) Mrcog Part 1 Pastpapers Recall Mrcog Total Guide

possible 2 detect the sex of the embryo uses pcr 4 chromosomal deletions can be used 4 detection of single gene disoderes facial hair causes testosterone mestranol spironolactone real time bias length time bias negative predictive value fibrocystic disease of thr breast...in paper 2 , breast anatomy and breast from endocrine function (in paper 1..)...ie 3 qs we about the breast levator ani its attachnment and its coverings endometrial carcinoma adenomyosis about measuring the arterial blood pressure and its methods interleukin 2 prenatal implantation genetic diagnosis menstural cycle in paper 1 and 2

MARCH 2006

The germination of tetanus spores in a wound is inhibited by a)tissue trauma b)oxygen c)injection of anti-toxin d)injection of toxoid e)removal of devitalised tissue The following values fall within the normal range for the adult female bladder a)residual volume of 100ml b)voiding capacity of 250ml c)bladder capacity of 900ml d)intravesical pressure rise of less than 10cm H2O during early filling e)maximum urine flow rate of 60ml per second

Page 159: 2005-2010(March) Mrcog Part 1 Pastpapers Recall Mrcog Total Guide

In the small intestine, the following substances are absorbed by active processes a)water b)sodium c)vitamin K d)amino acids e)chloride White cell migration from blood vessels in areas of inflammation involves a)cell migration occurring between endothelial cells b)a passive loss of fluid blood elements c)cell migration independent of endothelial cell motion d)initial emigration of polymorphonuclear neutrophils e)more polymorphs that monocytes after 2 days Antibodies (!!) a)are soluble proteins b)are formed in the fetus before 12 weeks of intrauterine life c)have an average molecular weight of around 10000 daltons d)of the rhesus type are genetically transmitted e)are produced by the ribosomes of plasma cells In DNA a)a codon is a sequence of three bases b)all codons have an identified function c)there is a greater variety of amino acids than there are different codons d)replication can be initiated at several different points along a chromosome e)complementary pairing precedes messenger mRNA synthesis The inferior vena cava a)is formed at the level of the fifth lumbar vertebra b)commences posterior to the right external iliac artery c)receives the left ovarian vein d)receives the right ovarian vein e)pierces the central tendon of the diaphragm In the normal human pelvis a)the promontory of the sacrum is in the upper anterior border of the first sacral vertebra b)the anterior surface of the sacrum has five paired foramina

Page 160: 2005-2010(March) Mrcog Part 1 Pastpapers Recall Mrcog Total Guide

c)the joint between the two pubic bones is a synovial joint d)the acetabular fossa is wholly formed from parts of the pubic and ischial bones e)the transverse diameter of the brim is greater than the anteroposterior diameter Arginine vasopressin a)reduces the glomerular filtration rate b)controls water loss in the proximal renal tubule c)is synthesised by the posterior pituitary gland d)is released in response to a rise in plasma osmolality e)is released in response to a fall in circulating plasma volume Concerning ovarian function a)progesterone is the major steroid of the developing follicle b)granulosa cells secrete oestradiol c)oestradiol is derived from androgen precursors d)insulin-like growth factor (IGF-1) is not secreted by the ovary e)circulating inhibin concentrations are a marker of granulosa cell function Actinomyces israelii a)is a rickettsia b)forms yellow granules in pus c)is a commensal in the mouth d)is a commensal in the vagina e)is usually resistant to penicillin Halothane produces a)cardiac arrhythmias b)explosive mixtures with air c)liver damage if given repeatedly d)myometrial relaxation e)bronchial irritation The therapeutic effect of the first drug is enhanced by the second drug a)phenytoin: ethinyloestradiol b)bromocryptine: metoclopramide c)penicillin: probenicid d)ritodrine: dexamethasone e)warfarin: phenobarbitone Potassium

Page 161: 2005-2010(March) Mrcog Part 1 Pastpapers Recall Mrcog Total Guide

a)is mainly intracellular b)plasma levels vary in proportion to intracellular levels c)plasma levels are decreased in Addison’s disease d)plasma levels are increased in diabetic ketoacidosis e)deficiency occurs with prolonged vomiting Concerning carbohydrates a)sucrose is a disaccharide of glucose and fructose b)cereal grains contain less than 40% starch c)cellulose is a fructose polysaccharide d)a normal diet contains less than 60g of carbohydrate daily e)dietary carbohydrate is oxidised in the body to carbon dioxide and water Steroid hormones a)all contain 20 carbon atoms b)can be produced by structures of urogenital ridge origin c)are mostly activated in the liver d)are predominantly excreted unchanged in the urine e)mainly circulate unbound to carrier proteins Ventilation is increased due to stimulation of central receptors by a)nikethamide b)hypoxia c)doxapram d)phenobarbitone e)salbutamol Tetrahydrofolic acid a)is involved in purine synthesis b)is a precursor of folic acid c)is a coenzyme in amino acid synthesis d)catalyses the conversion of glucose to glucose-6-phosphate e)activity is inhibited by Methotrexate Mitochondrial DNA a)is located in the nucleus b)inheritance is patrilineal c)is present in two copies per cell d)mutation causes cystic fibrosis e)is involved in the control of oxidative phosphorylation The following conditions may lead to hydronephrosis

Page 162: 2005-2010(March) Mrcog Part 1 Pastpapers Recall Mrcog Total Guide

a)mercury poisoning b)cervical carcinoma c)renal calculi d)renal vein thrombosis e)posterior urethral valves In uncomplicated homozygous beta thalassaemia there is a)hypochromasia b)a reduction in haemoglobin A2 c)an increase in haemoglobin F d)no depletion of iron stores e)the presence of megaloblasts in bone marrow The following statements relate to lung function in normal pregnancy a)vital capacity is increased by about 50% b)tidal volume is increased c)the subcostal angle increases d)the residual volume is reduced e)the respiratory rate is increased In normal pregnancy, uterine blood flow a)is about 50ml/minute at term b)is maintained throughout the cardiac cycle to the choriodecidual space c)is reduced by prostacyclin d)is increased during uterine contractions e)represents about 10% of the cardiac output by the end of the first trimester During normal pregnancy a)arterial pCO2 decreases b)the blood hydrogen ion concentration decreases c)plasma bicarbonate concentrations decrease d)urine pH falls e)lactic acid production is increased The anal canal a)has an upper part which is innervated by the inferior hypogastric plexus b)has a lower part which is supplied by the superior rectal artery c)drains lymph to the superficial inguinal nodes from its upper part d)has its internal sphincter innervated by the inferior rectal nerve e)has a superficial part of its external sphincter attached to the coccyx. Human placental lactogen

Page 163: 2005-2010(March) Mrcog Part 1 Pastpapers Recall Mrcog Total Guide

a)is a single chain polypeptide b)reaches the same concentration in fetal and maternal blood at term c)may be secreted by the decidua d)is detectable only after the 25th week of pregnancy e)is an insulin antagonist Human chorionic gonadotrophin !! a)is a glycoprotein b)is detectable 48 hours after fertilisation occurs c)secretion peaks at 20 weeks of gestation d)is synthesised by the corpus luteum of pregnancy e)binds to luteinizing hormone receptors Cholecalciferol (vitamin D) a)promotes the absorption b)is 25-hydroxylated in the liver c)is synthesised in the SKIN d)is 1-hydroxylated in the kidney e)is most active in the 1,25-dihydroxyl form Unconjugated bilirubin !! a)is normally present in the plasma in lower concentration than conjugated bilirubin b)circulates in the plasma bound to albumin c)is not excreted in the urine d)does not cross the blood-brain barrier e)crosses the placenta The following are capable of cellular regeneration: a)spinal cord b)liver c)epidermis d)myocardium e)bone marrow The Leydig cells of the testis !! a)secrete seminal fluid b)are stimulated by luteinising hormone c)are active in intrauterine life d)secrete fructose e)produce androstenedione Angiotensin II !!

Page 164: 2005-2010(March) Mrcog Part 1 Pastpapers Recall Mrcog Total Guide

a)is the most potent vasoconstrictor b)reduces aldosterone production c)is mainly found in the lungs d)is a decapeptide e)is produced when the extracellular fluid volume is reduced Cyproterone acetate a)is an oestrogen b)is used for the treatment of amenorrhoea c)binds to androgen receptors d)increases libido e)inhibits spermatogenesis Lignocaine used as a local anaesthetic !! a)causes tachycardia if given as a systemic injection b)has a longer lasting action than bupivicaine c)is used in combination with adrenaline for ring block d)causes vasoconstriction e)is a weak base Plasma osmolarity in the human a)is normally about 290 milliosmoles per kg in the nonpregnant state b)increases during the first trimester of pregnancy c)is closely controlled by plasma protein concentration d)is regulated by arginine vasopressin e)regulates the sensation of thirst In the statistical analysis of any group of numerical observations a)the mean is always less than the mode b)the median value always lies at the mid-point of the range c)standard deviation is always greater than the standard error of the mean d)the standard error of the mean is independent of the total number of observations e)there are the same number of observations greater than and less than the median value The pelvic splanchnic nerves a)are derived from the posterior rami of the sacral spinal nerves b)supply afferent fibres c)mix with branches of the sympathetic pelvic plexus d)supply the ascending colon with motor fibres e)supply the uterus with parasympathetic fibres

Page 165: 2005-2010(March) Mrcog Part 1 Pastpapers Recall Mrcog Total Guide

Exotoxins a)are derived from gram-negative bacteria b)have specific action c)are more toxic than endotoxins d)are neutralised by their homologous antitoxin e)can be converted to toxoid Candida albicans a)is gram positive b)is an anaerobic organism c)is associated with diabetes mellitus d)is motile e)is inhibited by oral tetracycline therapy Transferrin a)is one third saturated with iron b)is increased in pregnancy c)binds to 10mg of iron per gram d)levels in the neonate are low e)is actively transported In Crohn’s disease there is a) non casesous granuloma formation b) formation of deep fissures c) formation of crypt abscesses d) the presence of mucosal polyps e) a recognised association with occurence in the vulva The rectum a) usually commences at the level of S3 b) has a mesentery in its proximal third c) drains lymph to the pre-aortic nodes d) has a parasympathetic nerve supply derived entirely form the S3 spinal segment e) is about 25cm in length The female urethra (!!) a) traverses the perineal membrane b) is lined throughout by urothelium c) has a muscle layer continuous with that of the bladder d) has an external sphincter supplied by the obturator nerve

Page 166: 2005-2010(March) Mrcog Part 1 Pastpapers Recall Mrcog Total Guide

e) corresponds developmentally to the membranous urethra in the male During menstruation, endometrial haemostasis depends upon a) a platelet and fibrin plug formation after the first 20 hours of bleeding b) oestrogen mediated vasoconstriction c) prostaglandin mediated spiral vessel constriction d) production of plasminogen activating factor by the glandular epithelial cells In HIV, seroconversion illness after a needlestick injury a)occurs within 1 to 4 weeks following exposure b)infectivity decreases after seroconversion c)the CD4 count is a useful adjunct in diagnosis d)measurement of HIV RNA viral load is most useful in diagnosis The following tumours arise in the ovary: a) nephroblastoma b) cystadenoma c) granulosa cell tumour d) neuroblastoma e) teratoma Early blood borne dissemination is a characteristic feature of: a) carcinoma of the endometrium b) osteosarcoma c) basal cell carcinoma d) carcinoma of the cervix e) choriocarcinoma Breast milk compared to cow's milk (!!) a) has more protein b) has more sodium c) contains more carbohydrate d) has more casein e) contains more calories The following are consequences of pulmonary embolism: a) pulmonary infarction b) fibrinous pleurisy c) right ventricular hypertrophy d) sudden death e) haemoptysis

Page 167: 2005-2010(March) Mrcog Part 1 Pastpapers Recall Mrcog Total Guide

In a country wide survey, 10 general practices were picked at random and 5% of patients were selected at random from each practice: a) the sample of patients is a true random sample b) all practices had an equal chance of selection c) all patients had an equal chance of selection d) two siblings could not have been selected e) inferences about all patients in the country may be drawn from the sample Nitric oxide a) is synthesised in the endothelium b) has a short half life c) causes smooth muscle contraction d) increases during pregnancy e) combines with oxygen to produce L-arginine Carbohydrates a) yield 17kj/g of energy b) are a major source of energy for the brain c) are a a major component of the diet d) give a higher energy yield compared to fats Head mesenchyme gives rise to the following skull bones a)Parietal b)Mandible c)Maxillary d)Frontal e)Ethmoid Pre-implantation genetic diagnosis a) can be used to identify single gene defects b) is used in ICSI only c) testing for trisomy 21 should be offered in a case of ovum donation with a 45 year old recipient and 23 year old donor d) is routinely used in IVF in selection of embryos e) is governed by the regulations of the Human, Embryology and Fertilisation Authority The following are gram positive organisms a)Brucella b)Listeria c)Staphylococcus d)Vibrio cholera

Page 168: 2005-2010(March) Mrcog Part 1 Pastpapers Recall Mrcog Total Guide

Endometriotic deposits a) only occur in the pelvis b) consist of deposits of endometrial stromal tissue without glands c) do not occur in postmenopausal women d) are hormone sensitive Amyloidosis a)Can be inherited b)Is associated with bronchiectasis c)Is a cause of cardiomyopathy Phenylketonuria a)can only be diagnosed in the adult b)is diagnosed by high levels of phenylpyruvate in the urine c)treatment is with diet restriction DUB (Dysfunctional uterine bleeding) a) is common at the extremes of life b) may be caused by coagulation disorders Chylomicrons a) are not normally present in the fasting state b) after hydrolysis of lipoproteins, mainly consist of phospholipid Other questions that I cannot remember in such detail were about: Insulin (*half life of 30min) Pituitary gland Anatomy (*contains pars tuberalis) Many Pathology questions (equivalents ie *ovarian dysgerminoma = seminoma?, *endodermal sinus tumour = yolk sac tumour?) Embryology (notochord, primitive streak, mesonephros derivatives) Ionising radiation (radiosensitivity ie *liver /intestinal epithelium - which is more radiosensitive; *is radiation exposure of dental x-ray equivalent to 1/5 that from a transatlantic flight) difference b/w human milk and cows milk casien lactose sodium ascorbic acid water phenyl ketoneuria

Page 169: 2005-2010(March) Mrcog Part 1 Pastpapers Recall Mrcog Total Guide

inherited disorder cause by defficiency of phenyl alanine dietry restriction of tyrosine helpful diagnosed by measuring phenyl ketone level in urine essential amino acids are not synthesized by body sufficiently all ketogenic can produce energy dont remember exact wording radiation mostly harmful in 8-12 wks gestation xrays r ionizing radiation contraindicated in pregnancy transatlantic flight hv higher dose of radiation than xray MRI repeat q from sept paper chylomicrins responsible for turbulence of plsma?dont remem metabolized in adipose tissue -true 80% 20% in liver after hydrolysis by lipoprotien mainly phospholipids r absorbed form intestinal luminal cells to ----? ovarian tumors dysgerminoma and endodermal sinus tumor hv same origin choriocharcinoma can arise cocp increase risk of ovarian cancers -this qs repeated again in other qs with different wording b cells arise from plasma cells in pregnancy humoral immunity is depressed early in pregnany tumor necrotic factor alpha raised in parturition pulmonary embolism cause s sudden death fibrinous plurisy rt ventricular hypertrohy haemoptysis repeated from sept 05 transferrin - endo metrium hiv seroconversion carbohydrate preimplantation diagnosis repeat from march 2005 progesteron following rise in leuteal phase

Page 170: 2005-2010(March) Mrcog Part 1 Pastpapers Recall Mrcog Total Guide

basal body temp progesterone LH--PLZ SOME ONE ADD I DONT REMEMBER NOTOCHORD R SOLID CELLS GIVE RISE TO brain --? SKULL MESENCHYME GIVE RISE TEMPORAL BONE FRONTAL PARIETAL Q-Disinfectant solutions may become contaminated with: 1-Enterobacter species 2-Streptococcus species 3-Escherichia coli 4-Pseudomonas aeruginosa 5-Staphyloccus pyogenes Q-Examples of active transport across a membrane include the passage of: 1-Potassium ions into the neurons 2-Water into the proximal renal tubular cell 3-Glucose from the proximal renal tubular cell 4-Fatty acids into the mucosal cells of the intestine 5-Hydrogen ions from the gastric oxyntic cells. ENDOMETRIOSIS has endometrial tissue with no glands not occurs after menopause

The following values fall within the normal range for the adult female bladder Fa)residual volume of 100ml Tb)voiding capacity of 250ml Fc)bladder capacity of 900ml Td)intravesical pressure rise of less than 10cm H2O during early filling Fe)maximum urine flow rate of 60ml per second

Page 171: 2005-2010(March) Mrcog Part 1 Pastpapers Recall Mrcog Total Guide

The germination of tetanus spores in a wound is inhibited by Fa)tissue trauma Tb)oxygen Fc)injection of anti-toxin Td)injection of toxoid Te)removal of devitalised tissue In DNA Ta)a codon is a sequence of three bases Tb)all codons have an identified function Fc)there is a greater variety of amino acids than there are different codons Td)replication can be initiated at several different points along a chromosome Fe)complementary pairing precedes messenger mRNA synthesis

Concerning ovarian function Fa)progesterone is the major steroid of the developing follicle Tb)granulosa cells secrete oestradiol Tc)oestradiol is derived from androgen precursors Td)insulin-like growth factor (IGF-1) is not secreted by the ovary Fe)circulating inhibin concentrations are a marker of granulosa cell function Actinomyces israelii Fa)is a rickettsia Tb)forms yellow granules in pus Tc)is a commensal in the mouth Fd)is a commensal in the vagina Fe)is usually resistant to penicillin In the small intestine, the following substances are absorbed by active processes F a)water T b)sodium F c)vitamin K T d)amino acids T e)chloride White cell migration from blood vessels in areas of inflammation involves T a)cell migration occurring between endothelial cells F b)a passive loss of fluid blood elements

Page 172: 2005-2010(March) Mrcog Part 1 Pastpapers Recall Mrcog Total Guide

T c)cell migration independent of endothelial cell motion T d)initial emigration of polymorphonuclear neutrophils F e)more polymorphs that monocytes after 2 days Which are formed from intramembranous ossification? T Temporal , occipital, parietal F. Sphenoid, Ethmoid

Antibodies (!!) Ta)are soluble proteins -- Ab are glycoprotein. Are they soluble? not sure [color=red:c2901df9c1][b:c2901df9c1]F[/b:c2901df9c1][/color:c2901df9c1]b)are formed in the fetus before 12 weeks of intrauterine life Fc)have an average molecular weight of around 10000 daltons Fd)of the rhesus type are genetically transmitted Te)are produced by the ribosomes of plasma cells

Correction The germination of tetanus spores in a wound is inhibited by Fa)tissue trauma Tb)oxygen [color=red:1e03073b8c][b:1e03073b8c]T[/b:1e03073b8c][/color:1e03073b8c]c)injection of anti-toxin Td)injection of toxoid Te)removal of devitalised tissue

Insulin (*half life of 30min) F Pituitary gland Anatomy (*contains pars tuberalis)F Q-Disinfectant solutions may become contaminated with: T1-Enterobacter species F2-Streptococcus species F3-Escherichia coli T4-Pseudomonas aeruginosa F5-Staphyloccus pyogenes

The inferior vena cava Ta)is formed at the level of the fifth lumbar vertebra Fb)commences posterior to the right external iliac artery Fc)receives the left ovarian vein

Page 173: 2005-2010(March) Mrcog Part 1 Pastpapers Recall Mrcog Total Guide

Td)receives the right ovarian vein Te)pierces the central tendon of the diaphragm

In the normal human pelvis Ta)the promontory of the sacrum is in the upper anterior border of the first sacral vertebra Fb)the anterior surface of the sacrum has five paired foramina Fc)the joint between the two pubic bones is a synovial joint [b:715c81bcd5]d)the acetabular fossa is wholly formed from parts of the pubic and ischial bones [/b:715c81bcd5] Te)the transverse diameter of the brim is greater than the anteroposterior diameter

Arginine vasopressin Fa)reduces the glomerular filtration rate Fb)controls water loss in the proximal renal tubule Fc)is synthesised by the posterior pituitary gland Td)is released in response to a rise in plasma osmolality Te)is released in response to a fall in circulating plasma volume

Concerning ovarian function Fa)progesterone is the major steroid of the developing follicle Tb)granulosa cells secrete oestradiol Tc)oestradiol is derived from androgen precursors Td)insulin-like growth factor (IGF-1) is not secreted by the ovary Fe)circulating inhibin concentrations are a marker of granulosa cell function

Actinomyces israelii Fa)is a rickettsia Tb)forms yellow granules in pus Tc)is a commensal in the mouth Fd)is a commensal in the vagina Fe)is usually resistant to penicillin

Halothane produces Ta)cardiac arrhythmias

Page 174: 2005-2010(March) Mrcog Part 1 Pastpapers Recall Mrcog Total Guide

Fb)explosive mixtures with air Tc)liver damage if given repeatedly Td)myometrial relaxation Fe)bronchial irritation

The therapeutic effect of the first drug is enhanced by the second drug Fa)phenytoin: ethinyloestradiol Fb)bromocryptine: metoclopramide Tc)penicillin: probenicid Td)ritodrine: dexamethasone Fe)warfarin: phenobarbitone

Potassium Ta)is mainly intracellular Fb)plasma levels vary in proportion to intracellular levels Fc)plasma levels are decreased in Addison’s disease Td)plasma levels are increased in diabetic ketoacidosis Te)deficiency occurs with prolonged vomiting

Concerning carbohydrates Ta)sucrose is a disaccharide of glucose and fructose [color=red:715c81bcd5][b:715c81bcd5]F[/b:715c81bcd5][/color:715c81bcd5]b)cereal grains contain less than 40% starch [color=red:715c81bcd5][b:715c81bcd5]F[/b:715c81bcd5][/color:715c81bcd5]c)cellulose is a fructose polysaccharide Fd)a normal diet contains less than 60g of carbohydrate daily Te)dietary carbohydrate is oxidised in the body to carbon dioxide and water

Steroid hormones Fa)all contain 20 carbon atoms Tb)can be produced by structures of urogenital ridge origin Fc)are mostly activated in the liver Fd)are predominantly excreted unchanged in the urine Fe)mainly circulate unbound to carrier proteins

Ventilation is increased due to stimulation of central receptors by

Page 175: 2005-2010(March) Mrcog Part 1 Pastpapers Recall Mrcog Total Guide

Ta)nikethamide Tb)hypoxia Tc)doxapram Fd)phenobarbitone Fe)salbutamol

Tetrahydrofolic acid Ta)is involved in purine synthesis Fb)is a precursor of folic acid Fc)is a coenzyme in amino acid synthesis Fd)catalyses the conversion of glucose to glucose-6-phosphate Te)activity is inhibited by Methotrexate

Mitochondrial DNA Ta)is located in the nucleus Fb)inheritance is patrilineal Fc)is present in two copies per cell Fd)mutation causes cystic fibrosis Te)is involved in the control of oxidative phosphorylation

The following conditions may lead to hydronephrosis ?Fa)mercury poisoning Tb)cervical carcinoma Tc)renal calculi ?Fd)renal vein thrombosis Te)posterior urethral valves

In uncomplicated homozygous beta thalassaemia there is Ta)hypochromasia Fb)a reduction in haemoglobin A2 Tc)an increase in haemoglobin F Td)no depletion of iron stores Fe)the presence of megaloblasts in bone marrow

The following statements relate to lung function in normal pregnancy Fa)vital capacity is increased by about 50% (ans from busyspr) Tb)tidal volume is increased

Page 176: 2005-2010(March) Mrcog Part 1 Pastpapers Recall Mrcog Total Guide

Tc)the subcostal angle increases Td)the residual volume is reduced Fe)the respiratory rate is increased

[color=darkblue:715c81bcd5]In normal pregnancy, uterine blood flow Fa)is about 50ml/minute at term Tb)is maintained throughout the cardiac cycle to the choriodecidual space

Fc)is reduced by prostacyclin Fd)is increased during uterine contractions Te)represents about 10% of the cardiac output by the end of the first trimester [/color:715c81bcd5]

P.200 sep 2001 paper 2 Q 46 During normal pregnancy Ta)arterial pCO2 decreases Fb)the blood hydrogen ion concentration decreases Tc)plasma bicarbonate concentrations decrease Fd)urine pH falls ?Te)lactic acid production is increased

The anal canal Ta)has an upper part which is innervated by the inferior hypogastric plexus Fb)has a lower part which is supplied by the superior rectal artery Fc)drains lymph to the superficial inguinal nodes from its upper part Fd)has its internal sphincter innervated by the inferior rectal nerve Te)has a superficial part of its external sphincter attached to the coccyx.

Human placental lactogen ?T[b:715c81bcd5]a)is a single chain polypeptide [/b:715c81bcd5] Fb)reaches the same concentration in fetal and maternal blood at term Tc)may be secreted by the decidua ?F[b:715c81bcd5]d)is detectable only after the 25th week of pregnancy[/b:715c81bcd5] ?T[b:715c81bcd5]Te)is an insulin antagonist [/b:715c81bcd5]

Page 177: 2005-2010(March) Mrcog Part 1 Pastpapers Recall Mrcog Total Guide

Human chorionic gonadotrophin !! Ta)is a glycoprotein ?T[b:715c81bcd5]b)is detectable 48 hours after fertilisation occurs [/b:715c81bcd5] Fc)secretion peaks at 20 weeks of gestation Fd)is synthesised by the corpus luteum of pregnancy Te)binds to luteinizing hormone receptors

Cholecalciferol (vitamin D) Ta)promotes the absorption Tb)is 25-hydroxylated in the liver Tc)is synthesised in the SKIN Td)is 1-hydroxylated in the kidney Te)is most active in the 1,25-dihydroxyl form

Unconjugated bilirubin !! ?Fa)is normally present in the plasma in lower concentration than conjugated bilirubin ?Tb)circulates in the plasma bound to albumin Fc)is not excreted in the urine Fd)does not cross the blood-brain barrier ?Te)crosses the placenta

The following are capable of cellular regeneration: Fa)spinal cord Tb)liver Tc)epidermis Fd)myocardium Te)bone marrow

The Leydig cells of the testis !! Fa)secrete seminal fluid Tb)are stimulated by luteinising hormone Tc)are active in intrauterine life Fd)secrete fructose ?Fe)produce androstenedione

Page 178: 2005-2010(March) Mrcog Part 1 Pastpapers Recall Mrcog Total Guide

Angiotensin II !! Ta)is the most potent vasoconstrictor Fb)reduces aldosterone production Tc)is mainly found in the lungs Fd)is a decapeptide -- angiotensin I is a decapeptide Te)is produced when the extracellular fluid volume is reduced

Cyproterone acetate Fa)is an oestrogen [color=blue:715c81bcd5]--is an antiandrogenic progesterone[/color:715c81bcd5] Fb)is used for the treatment of amenorrhoea [color=blue:715c81bcd5]-- treat hirsutism [/color:715c81bcd5] Tc)binds to androgen receptors Fd)increases libido Te)inhibits spermatogenesis

Lignocaine used as a local anaesthetic Ta)causes tachycardia if given as a systemic injection Fb)has a longer lasting action than bupivicaine Fc)is used in combination with adrenaline for ring block Fd)causes vasoconstriction Te)is a weak base

Plasma osmolarity in the human Ta)is normally about 290 milliosmoles per kg in the nonpregnant state Fb)increases during the first trimester of pregnancy ?Fc)is closely controlled by plasma protein concentration Td)is regulated by arginine vasopressin Te)regulates the sensation of thirst

In the statistical analysis of any group of numerical observations Fa)the mean is always less than the mode Tb)the median value always lies at the mid-point of the range Tc)standard deviation is always greater than the standard error of the mean Fd)the standard error of the mean is independent of the total number of observations

Page 179: 2005-2010(March) Mrcog Part 1 Pastpapers Recall Mrcog Total Guide

Te)there are the same number of observations greater than and less than the median value

The pelvic splanchnic nerves Fa)are derived from the posterior rami of the sacral spinal nerves Fb)supply afferent fibres Tc)mix with branches of the sympathetic pelvic plexus Fd)supply the ascending colon with motor fibres Te)supply the uterus with parasympathetic fibres

Exotoxins Ta)are derived from gram-negative bacteria [color=red:715c81bcd5][b:715c81bcd5]T[/b:715c81bcd5][/color:715c81bcd5]b)have specific action Tc)are more toxic than endotoxins Td)are neutralised by their homologous antitoxin Te)can be converted to toxoid

Candida albicans Ta)is gram positive Fb)is an anaerobic organism Tc)is associated with diabetes mellitus Fd)is motile Fe)is inhibited by oral tetracycline therapy T treat with miconazole T commonsal in intestine

Transferrin Ta)is one third saturated with iron c)binds to 10mg of iron per gram d)levels in the neonate are low e)is actively transported Increased in pregnancy t

Page 180: 2005-2010(March) Mrcog Part 1 Pastpapers Recall Mrcog Total Guide

In Crohn’s disease there is ?Ta) non casesous granuloma formation Tb) formation of deep fissures Tc) formation of crypt abscesses Td) the presence of mucosal polyps e) a recognised association with occurence in the vulva

The rectum Ta) usually commences at the level of S3 Fb) has a mesentery in its proximal third [b:715c81bcd5][color=red:715c81bcd5]T[/color:715c81bcd5][/b:715c81bcd5]c) drains lymph to the pre-aortic nodes [color=red:715c81bcd5][b:715c81bcd5]T[/b:715c81bcd5][/color:715c81bcd5]d) has a parasympathetic nerve supply derived entirely form the S3 spinal segment [color=red:715c81bcd5][b:715c81bcd5]F[/b:715c81bcd5][/color:715c81bcd5]e) is about 25cm in length

[b:715c81bcd5]The female urethra (!!) a) traverses the perineal membrane b) is lined throughout by urothelium c) has a muscle layer continuous with that of the bladder d) has an external sphincter supplied by the obturator nerve e) corresponds developmentally to the membranous urethra in the male [/b:715c81bcd5]

[b:715c81bcd5]During menstruation, endometrial haemostasis depends upon a) a platelet and fibrin plug formation after the first 20 hours of bleeding b) oestrogen mediated vasoconstriction Tc) prostaglandin mediated spiral vessel constriction d) production of plasminogen activating factor by the glandular epithelial cells [/b:715c81bcd5] During menstruation endometrial hemostasis depends upon

Platelet and fibrin plug formation after first 20 hrs of bleeding Production of PAF by glandular epithelial cells

Page 181: 2005-2010(March) Mrcog Part 1 Pastpapers Recall Mrcog Total Guide

Estrogen mediated hemostasis PG mediated hemostasis

In HIV, seroconversion illness after a needlestick injury Ta)occurs within 1 to 4 weeks following exposure [color=darkblue:715c81bcd5][b:715c81bcd5]?T[/b:715c81bcd5][/color:715c81bcd5]b)infectivity decreases after seroconversion Tc)the CD4 count is a useful adjunct in diagnosis Td)measurement of HIV RNA viral load is most useful in diagnosis HIV seroconversion Illness occurs witin 1-4 wks......T...........SEROCONVERSION OCCURS IN FIRST FEW WEEKS......FOLLOWED BY SIGNS OF ILLNESS After illness infectivity decreases .........[b:715c81bcd5][color=darkblue:715c81bcd5].F.........[/color:715c81bcd5][/b:715c81bcd5]I THINK SO BECAUSE THOUGH ITS A LATENT PHASE AND NO OVERT SYMPTOMS OCCUR WHILE THE VIRUS MULTIPLIES, I THINK INFECTIVITY REMAINS JUST THE SAME......... The best available diagnostic test is HIV viral load assay........[color=red:715c81bcd5][b:715c81bcd5]T[/b:715c81bcd5][/color:715c81bcd5].........FROM AN OBSTETRIC POINT OF VIEW.....VIRAL LOAD IS THE ONLY USEFUL WAY OF PREDICTING DISEASE......BECAUSE SEROLOGY IS UNRELIABLE IN ITS RESULTS DUE TO MATERNAL ANTIBODIES......WHICH MIGHT TAKE UPTO 18MONTHS TO DIE DOWN... Without any intervention during pregnancy the mother to child transmission rate is 80% .........[color=red:715c81bcd5][b:715c81bcd5]F[/b:715c81bcd5][/color:715c81bcd5].....ITS 15-45%......LATTER IN UNTREATED CASES...... Mode of delivery by LSCS decreases the transmission to baby......[color=red:715c81bcd5][b:715c81bcd5]T[/b:715c81bcd5][/color:715c81bcd5].....CAN BRING THE TRANSMISS DOWN TO 2% Short course of anti viral Rx during labor decreases transmission to baby.........[b:715c81bcd5][color=red:715c81bcd5]T[/color:715c81bcd5][/b:715c81bcd5]

Page 182: 2005-2010(March) Mrcog Part 1 Pastpapers Recall Mrcog Total Guide

BF is safe ........[b:715c81bcd5][color=red:715c81bcd5]F[/color:715c81bcd5][/b:715c81bcd5].......MOST IMP METHOD OF TRASMISS IF OTHER PREVENTIVE MEASURES HAVE BEEN UNDERTAKEN....

The following tumours arise in the ovary: Fa) nephroblastoma Tb) cystadenoma Tc) granulosa cell tumour Fd) neuroblastoma Te) teratoma

Early blood borne dissemination is a characteristic feature of: Fa) carcinoma of the endometrium [color=red:715c81bcd5]T[/color:715c81bcd5]b) osteosarcoma Fc) basal cell carcinoma Fd) carcinoma of the cervix Te) choriocarcinoma

Breast milk compared to cow's milk (!!) a) has more protein Tb) has more sodium c) contains more carbohydrate d) has more casein e) contains more calories difference b/w human milk and cows milk Tlactose Tsodium ascorbic acid Twater

PS:human milk contains more water

The following are consequences of pulmonary embolism: Ta) pulmonary infarction Tb) fibrinous pleurisy

Page 183: 2005-2010(March) Mrcog Part 1 Pastpapers Recall Mrcog Total Guide

?Fc) right ventricular hypertrophy Td) sudden death Te) haemoptysis

[color=blue:715c81bcd5]In a country wide survey, 10 general practices were picked at random and 5% of patients were selected at random from each practice: Ta) the sample of patients is a true random sample Tb) all practices had an equal chance of selection Tc) all patients had an equal chance of selection Fd) two siblings could not have been selected Te) inferences about all patients in the country may be drawn from the sample [/color:715c81bcd5] ans from onexamination

Nitric oxide Ta) is synthesised in the endothelium Tb) has a short half life Fc) causes smooth muscle contraction Td)production is increases during pregnancy [b:715c81bcd5]e) combines with oxygen to produce L-arginine [/b:715c81bcd5] T generated by arginine (from onexamination)

Carbohydrates a) yield 17kj/g of energy b) are a major source of energy for the brain ?Tc) are a a major component of the diet Fd) give a higher energy yield compared to fats

Head mesenchyme gives rise to the following skull bones a)Parietal b)Mandible c)Maxillary d)Frontal e)Ethmoid

Pre-implantation genetic diagnosis

Page 184: 2005-2010(March) Mrcog Part 1 Pastpapers Recall Mrcog Total Guide

Ta) can be used to identify single gene defects Fb) is used in ICSI only Fc) testing for trisomy 21 should be offered in a case of ovum donation with a 45 year old recipient and 23 year old donor Fd) is routinely used in IVF in selection of embryos ?T[b:715c81bcd5]e) is governed by the regulations of the Human, Embryology and Fertilisation Authority[/b:715c81bcd5]

The following are gram positive organisms ?Fa)Brucella Tb)Listeria Tc)Staphylococcus ?Fd)Vibrio cholera

Endometriotic deposits Fa) only occur in the pelvis ?Tb) consist of deposits of endometrial stromal tissue without glands ?Fc) do not occur in postmenopausal women ?Td) are hormone sensitive

Amyloidosis Ta)Can be inherited Tb)Is associated with bronchiectasis Tc)Is a cause of cardiomyopathy (ref: Familial Amyloidosis

Familial amyloidosis, or ATTR, is a rare form of inherited amyloidosis. The amyloid deposits in familial amyloidosis are composed of the protein transthyretin, or TTR, which is made in the liver. Familial amyloidosis is inherited an [color=darkblue:715c81bcd5]autosomal dominant [/color:715c81bcd5]in genetics terminology) http://www.ncbi.nlm.nih.gov/entrez/query.fcgi?cmd=Retrieve&db=PubMed&list_uids=12472203&dopt=Abstract http://www.emedicine.com/med/topic3365.htm When the heart is involved heavily but the nerves are not, the disease is called familial amyloid cardiomyopathy (FAC).

Page 185: 2005-2010(March) Mrcog Part 1 Pastpapers Recall Mrcog Total Guide

Phenylketonuria Fa)can only be diagnosed in the adult Tb)is diagnosed by high levels of phenylpyruvate in the urine ?Tc)treatment is with diet restriction ?Tinherited disorder Fcause by defficiency of phenyl alanine ( defficiency of phenylalanine hydroxylase) Fdietry restriction of tyrosine helpful (restriction of phenylalanine) ?Fdiagnosed by measuring phenyl ketone level in urine

DUB (Dysfunctional uterine bleeding) Ta) is common at the extremes of life Tb) may be caused by coagulation disorders

Insulin (*half life of 30min) T Pituitary gland Anatomy (*contains pars tuberalis) F

Many Pathology questions (equivalents ie *ovarian dysgerminoma = seminoma?, *endodermal sinus tumour = yolk sac tumour?)

Embryology (notochord, primitive streak, mesonephros derivatives)

NOTOCHORD R SOLID structure as it forms --T GIVE RISE TO brain -F F is formed from primitive streak (ans from mrcogexam) [color=blue:715c81bcd5][b:715c81bcd5]F fuses temporally with the endoblast of yolk sac[/b:715c81bcd5][/color:715c81bcd5]

Ionising radiation (?T radiosensitivity ie *liver /intestinal epithelium - which is more radiosensitive; *is radiation exposure of dental x-ray equivalent to 1/5 that from a transatlantic flight)

Page 186: 2005-2010(March) Mrcog Part 1 Pastpapers Recall Mrcog Total Guide

essential amino acids Fare not synthesized by body sufficiently ?Tall ketogenic ?Tcan produce energy dont remember exact wording [b:715c81bcd5]Are ONLY used for the synthesis or important body proteins. Canot be catabolised for energy.[/b:715c81bcd5]

radiation ?Tmostly harmful in 8-12 wks gestation Txrays r ionizing radiation Tcontraindicated in pregnancy [b:715c81bcd5]transatlantic flight hv higher dose of radiation than xray [/b:715c81bcd5]

Biophysics: Radiation effect more in hypoxic tissue..false Bone, peripheral nerve and muscle are senstive to radiation..false Intestine and bone marrow are sestive to radiation..true

chylomicrins Ta) are not normally present in the fasting state ?T[b:715c81bcd5]responsible for turbulence of plsma?[/b:715c81bcd5]dont remem [color=darkblue:715c81bcd5]metabolized in adipose tissue -[b:715c81bcd5]true [/b:715c81bcd5]80% 20% in liver [/color:715c81bcd5] Fafter hydrolysis by lipoprotien mainly phospholipids ?F r absorbed form intestinal luminal cells to ----? (ref: Monoglycerides and free fatty acids associate with bile salts and lecithin to form micelles * The core of the micelle is composed of cholesterol and fat soluble vitamins * Monoglycerides, cholesterol and free fatty acids are absorbed by passive diffusion across the duodenal and jejunal mucosa while the bile salts

Page 187: 2005-2010(March) Mrcog Part 1 Pastpapers Recall Mrcog Total Guide

remain in the lumen and are absorbed in the terminal ileum Chylomicrons have a core of triglycerides and cholesterol and a coat of protein and phospholipids

ovarian tumors Tchoriocharcinoma can arise

cocp F increase risk of ovarian cancers

Pill failure: tetracycline…T Rifmpicin…true digoxin....F phenobarbitone....T

Contraindicated in breast feeding: cascara.....T phenophthalien...F norethisterone.....F chloramphenicol...T sulphonamides......T

b cells Farise from plasma cells in pregnancy humoral immunity is depressed early in pregnany Ftumor necrotic factor

alpha raised in parturition

pulmonary embolism cause s : Tsudden death Tfibrinous plurisy

Page 188: 2005-2010(March) Mrcog Part 1 Pastpapers Recall Mrcog Total Guide

Frt ventricular hypertrohy Thaemoptysis

following rise in leuteal phase Tbasal body temp Tprogesterone

SKULL MESENCHYME GIVE RISE TEMPORAL BONE FRONTAL PARIETAL

Q-Examples of active transport across a membrane include the passage of: ?T1-Potassium ions into the neurons F2-Water into the proximal renal tubular cell --F ?T3-Glucose from the proximal renal tubular cell [color=red:715c81bcd5][b:715c81bcd5]F[/b:715c81bcd5][/color:715c81bcd5]4-Fatty acids into the mucosal cells of the intestine 5-Hydrogen ions from the gastric oxyntic cells. (ref: glucose absorption in small intestine n renal tubules is active,all other places is facilitated using GLUT. also Na/K pump is active. Na absorption is active in renal tubules. Monoglycerides, cholesterol and free fatty acids are absorbed by passive diffusion across the duodenal and jejunal mucosa while the bile salts remain in the lumen and are absorbed in the terminal ileum )

ENDOMETRIOSIS has endometrial tissue with no glands ?Tnot occurs after menopause[b:715c81bcd5][/b:715c81bcd5]

Regarding data

Page 189: 2005-2010(March) Mrcog Part 1 Pastpapers Recall Mrcog Total Guide

a The coefficient of variation expresses the Standard deviation as a proportion of the mean

b Standard error of mean indicates how close a sample is to the population mean

c A probability of one means that an event has one in ten chance of happening

d Logarithmictransfoormation is a method of normalizing data that are not normally distributed

e ANOVA is used for normally distributed data

Indomethacin can block ovulaion

Pulsatile release of LH occurs in first weel of life

RBCs possess the antigens of ABO system in fetus

MRI involves the administration of radio labelled compound

Doppler effect is used to detect movement

Proton is injected to improve contrast during MRI

ventilation and stimulation of central chemoreceptors...someone said true for hypoxia. From my knowledge, the central chemoreceptors respond to pH and CO2, not O2, rather the PERIPHERAL aortic and carotid bodies respond to O2 and thus hypoxia.

sept.2005

Page 190: 2005-2010(March) Mrcog Part 1 Pastpapers Recall Mrcog Total Guide

Sigmoid colon Attached to pelvic wall Lies lateral to psoas major Ends at S3

Rectum Starts at S3 No appendices epiploae Middle rectal artery is the major artery suuplying it 12 cm long

T pallidum Spiral shape Causes yaws in children Stained with silver impregnation

HIV seroconversion Illness occurs witin 1-4 wks After illness infectivity decreases The best available diagnostic test is HIV viral load assay Without any intervention during pregnancy the mother to child transmission rate is 80% Mode of delivery by LSCS decreases the transmission to baby Short course of anti viral Rx during labor decreases transmission to baby BF is safe

Percutaneous mode of infection H simplex Rabies Hepatitis B Epstein barr virus

Hep A mode of infection is through Blood transfusion Shared needles Faeco oral route Insect bites Air borne

Page 191: 2005-2010(March) Mrcog Part 1 Pastpapers Recall Mrcog Total Guide

Match the disease with their respective causative org Bone marrow suppression – myco bacterium avium Red cell aplasia – parvovirus

The foll are pre malignant CIN 3 Vulval lichen sclerosis Endometrioses Fibro epithelial polyp Atypical endometrial hyperplasia

Bacterial vaginosis Microscopic finding “clue cells” pH less than 4.5 best Rx with ampicillin sensitive to clindamycin may be due to gardnerella vaginalis

vaginal infecton incidence of n gonorrhoea resistance to penicillin is 1% incidence of n gonorrhoea resistance to 3 rd gen ceph 5% azithromycin DOC for trichomoniasis crytococcus neoformans causes meningitis

C trachomatis Strains L123 cause LGV Reticulate body is the infectious form

Transferrin Increased in pregnancy One third saturated with Fe Binds to 10 mg of iron per gm Levels in neonate are low

Antibodies

Page 192: 2005-2010(March) Mrcog Part 1 Pastpapers Recall Mrcog Total Guide

Are soluble proteins Formed before 12 wks in fetus ABO antigens are present in fetus

Foll Hb chains are present in intrauterine life Alpha 2 beta2 Alpha 2 gamma 2 Gamma 4 Alpha 2 epsilon 2 Zeta 2 epsilon2

Pulm embolism Spiral CT scan is indicated in suspected cases DVT always causes pulm embolism X ray angiography is diagnostic Spiral CT involves dignoses based on the perfusion copared to air in lungs ECG is reliable for diagnosis

MRI Involves administerind of radio contrast agents Protons give better imaging Damage to fetus is due to tissue heating

Xray Contraindicated in pregnancy Low rays are used for breast imaging

Conventional USG Increased freq gives higher resolution Increased freq gives better attenuation TVS uses higher freq compared to conventional USG Doppler USG measures movement Gives accurate measurement of bld flow in fetal vessels Doppler effect based on blood flow within vessels

Page 193: 2005-2010(March) Mrcog Part 1 Pastpapers Recall Mrcog Total Guide

Calcification occurs in Renal calculi Hyaline degeneration of fibroid Parathyroid adenomas Secondary bone deposits in prostratic CA

Foll originate from endothelial cells Nitric oxide Protein C Fibronectin Plasminogen activator

Thyroid 100-200 micro gms are trapped daily increases in size in pregnancy trapsinorganic iodine colloid is stored outside epithelial cells TBG is increased n pregnancy 5% is bound to CBG

ketone bodies formed from acetone formed from acetyl CoA not formed during starvation not utilised by brain

Mgso4 Potentiates action of non depolarising agents Check respiratory status Causes brisk patellar reflexes In toxicity Rx IV calcium sulphate

CIN CIN 3 does not involve breach in basement epithelial layer Arias stella reaction mimicks clear cell CA Tissue biopsy is required for diagnosis

Page 194: 2005-2010(March) Mrcog Part 1 Pastpapers Recall Mrcog Total Guide

Incidence is decreased in renal transplant patients

GH Stmulated by glucose infusion Increased in sleep Opposes insulin action Causes positive nitrogen balance Decreased in pregnancy

Foll form boundaries of ovarian fossa Ureter Ext iliac vn Int iliac art Int pudendal art Obliterated umb art

Penem antibiotics Active in beta lactam ring Drug of choice for ps aeroginoa

Vagina Derived from mullerian ducts Seperated from anal canal by anal body

Bladder Pelvic splanchnic stimulation contracts the trigone Ext urethral sphincter innervated by pudendal nv Mucosa of urethra is innervated by pudendal nv Transaction of spinal cord above S2 causes autonomous bladder Afferent innervation of glans of clitoris is by ilioinguinal nv

Hypo k+ is found in Prolonged vomiting In diabetis ketoacidosis Addison’s disease

Foll are below inguinal ligament

Page 195: 2005-2010(March) Mrcog Part 1 Pastpapers Recall Mrcog Total Guide

Ilio inguinal nv Lateral branch of genito femoral nv Subcostal nv Femoral art

Inf hypogastric (pelvic) plexus Supplies pain fibres to body of ut Continues as vesical plexus Lies at base of braod lig

Sciatic nv S3&4 contribute towards it Lies behind quadratus femoris Tibial branch supplies short head of biceps femoris Injection on the outer & upper quadrant of the buttock may cause damage to the nv

Undescended testis More common in premature than term infants Commonly assoc with gonadal neoplasia Asoc with 1% of absent testis More common on left side

Embryo Parameso duct lies medial to mesonephros Mesonephros forms urine TVS detects fetal heart beat 21 days after implantation Erthropoiesis is entirely medullary

UG sinus Divides the cloaca coronally Urachus connects bladder to umbilicus

Corpus luteum Is major source of relaxin Contains endothelial cells

Page 196: 2005-2010(March) Mrcog Part 1 Pastpapers Recall Mrcog Total Guide

Hysterectomy prolonges the life period 17 hydroxy prog is the main prog secreted steroids prevent recruitment of new follicles

fertilisation acrosome fuses with zona pellucida polar body has same karyotype as ovum fetilised egg moves slower than unfertilised egg in fallopian tube inner ell mass lysis deciduas during implantation

histo pathologic examination of tissue of conception can diagnose CIN 3 Cx incompetence Ectopic preg Gest Trophblastic Di

Ring inversion of Y chromosome found the foll will be done Maternal chromosome is tested Paternal chromosome is tested Scan to look for male genitalia Anomaly scan and amniocentesis done for fetus Do karyotype of all previous children of couple

Right ureter Below sigmoid coon Supplied by vaginal art Lined throughout by urothelium Contain glands

Oral absorption of drug depends More availablibility in ionised form than non ionised Increased lipid solubility increases penetration in CNS Enters intestinal cells by simle diffusion Protein binding increases excretion Lipid form increases uptake

Fetal circulation

Page 197: 2005-2010(March) Mrcog Part 1 Pastpapers Recall Mrcog Total Guide

Portal vn drains in IVC Ductus arteriosus takes blood to pulm art R atria and L atria communicate thro’ Foramen Ovale

Voiding in females Occurs above rate of 20 ml/sec When vesical press is more than 45 mm H2O Initial contraction of trigone

Enzyme activity is modified by Dephosphorylation Allosteric esters Binding to plasma prot Proteolytic cleaving of inactive precursors Gene transcription

Functional cyst occurs in Stein levinthal synd PID Multiple sclerosis Gest tropho di Rx with clomiphene

Amniotic fluid Hypotonic compared to fetal plasma Absored in lungs Unrelated to swallowing Increased until term in fetus Contains more HCO3 compared to fetal blood

Action of complement Increase in permeability Migration of polymorphs

Genetics South blot detects DNA DNA is exact copy of RNA

Page 198: 2005-2010(March) Mrcog Part 1 Pastpapers Recall Mrcog Total Guide

DNA is transcribed by RNA polymerase Ribosomes contain RNA Transcription occurs at any direction along the chain DNA replication is called transformation DNA has transcription restriction enzyme Histones do not contain DNA

Insulin Binding to tyrosine receptor kinase increases activity Causes glucose transporter movement to plasma membrane Increases glycogen synthetase activity Increases fat deposition Increase glucose utilisation by CNS

Heparin T ½ is 1.5 hrs LMW heparin is administered 4 hrly Given in pregnancy causes intracranial haemorrhage in fetus Longer duration of action in S/C compared to IV administration

Foll have imp natural reservoir other than human Listeria Saolmonella typhimurium Brucella

Crohn’s disease Non caseous granulomatous infection Forms deep fissures Forms crypt abscesses Assoc with mucosal polyps Recognised to occur in vulva Assoc with intestinal wall thickening

Absorption in kdny Glucose occurs in loop of henle Water is actively absorbed in proximal conv tubules

Page 199: 2005-2010(March) Mrcog Part 1 Pastpapers Recall Mrcog Total Guide

Estrogen 17 beta estradiol is formed by aromatisation of testos increases LH receptors secreted by theca cells granulosa cells secrete inhibin humoral immunity is depressed in pregnancy T & B cells are derived from bone marrow

teratoma usually benign can occur is assoc with germ cell tumors secretes hormones

during menstruation platelet and fibrin plug is formed after 1st 20 hrs of bleeding hemostasis is thro’ estr mediated vaso constriction hemostasis is thro PG mediated spiral vessel constriction

During ovulation Basal vacuoles occur in mid secretory phase Gland stromal mitosis maximum in mid secretory phase PG in follicle fluid during ovulation Increase of estr and prog receptors in proliferaive phase LDL receptors are increased in proliferative phase Indomethacin inhibits ovulation Matrixmetalloproteinases(MMC) is inhibited by progestrones Cloiphene can cause deficient secretory phase

Glycogen Synthesis occurs in muscle Is stored along with water

Carbohydrates Yield energy ?17 J/Kg Major source of energy for brain Major component of diet

Page 200: 2005-2010(March) Mrcog Part 1 Pastpapers Recall Mrcog Total Guide

Higher energy compared with fats

Foll decrease the efficacy of OCP’s Isoniazid Carbamaepine Phenytoin Rifampicin

Female pelvis compared to male Sacrum more curved Greater sc notch is larger Sub pubic angle is more Distance betw pubic sym to acetabulum is less than the diam of acetabulum

Vascular derived epithelial growth factor Causes vasodilatation of placental vessels Causes angiogenesis in placenta Is an angigogenic factor Increases permeability of endothelial cells

And always save the best for last ……STATISTICS

Definition LBW – less than 1,500 gms PMR – deaths in first 1 week per 1,000 births Perinatal mortality – deaths after 22wks ( more than 154 days) until 1 wk after birth Maternal mortality rate

Tests used ANOVA Std error of mean means diffce betw sample mean from population mean Probablity of 1 means 1 in 10 chance for event to occur

Examples of ?continuous variables are Bld glucose

Page 201: 2005-2010(March) Mrcog Part 1 Pastpapers Recall Mrcog Total Guide

Haemoglobin Gender Height Blood pressure

Sigmoid colon Attached to pelvic wall......T Lies lateral to psoas major ........F Ends at S3 .........T

SIGMOID COLON...STARTS AT PELVIC BRIM.....ENDS AT S3.......IS 45 CMS LONG.....LIES ON THE PERITONEAL SURFACE OF BLADDER AND UTERUS.....SO THE PSOAS IS A DEEPER STRUCTURE.....

Rectum Starts at S3 ........T No appendices epiploae.......T Middle rectal artery is the major artery suuplying it........F......ITS THE SUPERIOR RECTAL ARTERY 12 cm long .............T

T pallidum Spiral shape........T Causes yaws in children..........T Stained with silver impregnation.........F....... THE ONLY TESTS FOR MICROSCOPY ARE DARK GROUND

Page 202: 2005-2010(March) Mrcog Part 1 Pastpapers Recall Mrcog Total Guide

IMMUNOFLOURSCENCE................OTHERWISE SEROLOGY IS THE OTHER DIAGNOSTIC AID....

HIV seroconversion Illness occurs witin 1-4 wks......T...........SEROCONVERSION OCCURS IN FIRST FEW WEEKS......FOLLOWED BY SIGNS OF ILLNESS After illness infectivity decreases ..........F.........I THINK SO BECAUSE THOUGH ITS A LATENT PHASE AND NO OVERT SYMPTOMS OCCUR WHILE THE VIRUS MULTIPLIES, I THINK INFECTIVITY REMAINS JUST THE SAME......... The best available diagnostic test is HIV viral load assay........T.........FROM AN OBSTETRIC POINT OF VIEW.....VIRAL LOAD IS THE ONLY USEFUL WAY OF PREDICTING DISEASE......BECAUSE SEROLOGY IS UNRELIABLE IN ITS RESULTS DUE TO MATERNAL ANTIBODIES......WHICH MIGHT TAKE UPTO 18MONTHS TO DIE DOWN... Without any intervention during pregnancy the mother to child transmission rate is 80% .........F.....ITS 15-45%......LATTER IN UNTREATED CASES...... Mode of delivery by LSCS decreases the transmission to baby......T.....CAN BRING THE TRANSMISS DOWN TO 2% Short course of anti viral Rx during labor decreases transmission to baby.........T BF is safe ........F.......MOST IMP METHOD OF TRASMISS IF OTHER PREVENTIVE MEASURES HAVE BEEN UNDERTAKEN....

Percutaneous mode of infection H simplex........T Rabies .......F.....BITE WITH SALIVA Hepatitis B ........F...... Epstein barr virus .......T I THINK.......ECAUSE IT SAYS YOU CAN

Page 203: 2005-2010(March) Mrcog Part 1 Pastpapers Recall Mrcog Total Guide

GET IT FROM PATIENTS THROUGH "CASUAL CONTACT"......

Hep A mode of infection is through Blood transfusion.....F Shared needles .....F Faeco oral route ......T Insect bites ......F Air borne ......F

Match the disease with their respective causative org Bone marrow suppression – myco bacterium avium Red cell aplasia – parvovirus

The foll are pre malignant CIN 3 ........T Vulval lichen sclerosis.......F BUT NOT SURE... Endometrioses .......F Fibro epithelial polyp ............T I THINK... Atypical endometrial hyperplasia ??T

Bacterial vaginosis Microscopic finding “clue cells”.........T pH less than 4.5 .......F........HAS TO BE MORE THAN 5 FOR BV TO OCCUR best Rx with ampicillin ............F sensitive to clindamycin ...........F.......I DIDNT FIND IT ANYWHERE.....SO I WILL GO FOR F

Page 204: 2005-2010(March) Mrcog Part 1 Pastpapers Recall Mrcog Total Guide

may be due to gardnerella vaginalis........T.....

vaginal infecton incidence of n gonorrhoea resistance to penicillin is 1% ......T BUT NOT SURE 100% incidence of n gonorrhoea resistance to 3 rd gen ceph 5% .....? FALSE azithromycin DOC for trichomoniasis ,.......F....METRO... crytococcus neoformans causes meningitis .........T.....ESP IN AIDS PTS

C trachomatis Strains L123 cause LGV.........T Reticulate body is the infectious form .......F.....RETICULATE I THE VEGETATIVE NON INF INTRACELL. FORM....

Transferrin Increased in pregnancy ........T One third saturated with Fe .........T Binds to 10 mg of iron per gm ???.....DUNNO....DIDNT FIND THIS KIND OF DATA ANYWHERE.... Levels in neonate are low........FALSE.....

Antibodies Are soluble proteins........I THINK ITS TRUE....THEY HAVE TO BE.....BUT NO BOOK CONFIRMATION YET..... Formed before 12 wks in fetus .........TRUE.....IGM FORMS AT 11 WEEKS.... ABO antigens are present in fetus........TRUE

Page 205: 2005-2010(March) Mrcog Part 1 Pastpapers Recall Mrcog Total Guide

Foll Hb chains are present in intrauterine life Alpha 2 beta2 .......TRUE......HB A......PRESENT 8TH MONTH ONWARS Alpha 2 gamma 2 ........T......HB F......ALL THROUGH PREG AND LATER TOO... Gamma 4 ......F Alpha 2 epsilon 2........F Zeta 2 epsilon2 ........T.....HBGOWER 1.......PRESENT I THE FIRST FEW WEEKS..... ALSO THERE IS HBA2....ALPHA2 AND DELTA 2 UNITS...

Pulm embolism Spiral CT scan is indicated in suspected cases.........F DVT always causes pulm embolism ............F X ray angiography is diagnostic ...............T Spiral CT involves dignoses based on the perfusion copared to air in lungs ...............F ECG is reliable for diagnosis............F IS SPIRAL CT AND VQ SCAN THE SAME THING????SORRY FOR ASKING SUCH A BASICALLY DUMB QUESTION....

MRI Involves administerind of radio contrast agents........T I THINK.... Protons give better imaging ?????T I THINK.........BECAUSE IT DEENDS ON THE OSCILLATION OF CHARGED PARTICLES......EPS THE HYDROGEN ATOM.... Damage to fetus is due to tissue heating.......F I THINK.....BECAUSE MRI IS NOT AN IONISING RADIATION.....SO I DONT THINK THERE IS ANY HEATING.....

Page 206: 2005-2010(March) Mrcog Part 1 Pastpapers Recall Mrcog Total Guide

Xray Contraindicated in pregnancy...............F Low rays are used for breast imaging ............????......LOW COMPARED TO WHAT??7 TO 10 MRADS ARE USED..........WHICH IS MORE THAN EG THE 2-3 MRADS FOR CHEST X RAY...........I DONT THINK THATS LOW.......I WOULD SAY FALSE.....

Conventional USG Increased freq gives higher resolution Increased freq gives better attenuation TVS uses higher freq compared to conventional USG Doppler USG measures movement ........T Gives accurate measurement of bld flow in fetal vessels........T Doppler effect based on blood flow within vessels ...........T......THE SOUND OF BLOOD FLOW WITHIN VESSELS....TO BE EXACT.... FOR THE FIRST THREE STEMS......I COULDNT FIND THE ANSWER......BUT IT WAS SOMEWHERE IN THIS FORUM.........I WILL POST IT AS SOON AS I COME ACROSS IT AGAIN.......

Calcification occurs in Renal calculi ...........T Hyaline degeneration of fibroid.........T.....CALCIFICATION OCURS IN FIBROID........I DONT WHAT THE SIGNIFICANCE OF HYALINE DEGENERATION IS IN TH QUESTION Parathyroid adenomas ............T Secondary bone deposits in prostratic CA ..........T

Page 207: 2005-2010(March) Mrcog Part 1 Pastpapers Recall Mrcog Total Guide

Foll originate from endothelial cells Nitric oxide......T Protein C ............F Fibronectin ........T Plasminogen activator................T

Thyroid 100-200 micro gms are trapped daily.......T increases in size in pregnancy .........T I THINK.....ACTOVITY INCREASES........SO DOES THE SIZE......BUT IM NOT SURE.... trapsinorganic iodine .......T colloid is stored outside epithelial cells.........T TBG is increased n pregnancy ...........T. 5% is bound to CBG ...........................TO TBG I THINK WAS MEANT....F......T3 AND T4 ARE BOUND 75 AND 85 %

ketone bodies formed from acetone..............T formed from acetyl CoA ..........T ormed during starvation .....T not utilised by <a href="http://www.[obscene]a.com/tutorial/?q=brain &s=0">brain </a> T

Mgso4 Potentiates action of non depolarising agents T I THINK Check respiratory status T......PARALYSIS OF RESP MUSCLE IS THE

Page 208: 2005-2010(March) Mrcog Part 1 Pastpapers Recall Mrcog Total Guide

FIRST SIGN OF OVERDOSE Causes brisk patellar reflexes T.........ANOTHER SIGN OF MG EXCESS.... In toxicity Rx IV calcium sulphate .....F.....CA GLUCONATE...

CIN CIN 3 does not involve breach in basement epithelial layer F....THATS EXACTLY TH DISTINCTION Arias stella reaction mimicks clear cell CA..........THIS ONE IS JUT SOMEWHERE IN MY brain CELLS......DONT KNOW NOW.... Tissue biopsy is required for diagnosis ......FALSE....SMEAR CAN ALSO SHOW CIN III CHNGES....... Incidence is decreased in renal transplant patients?????......F I THINK.....COULDNT FIND INFO ANYWHERE.......BUT MY GUESS IS THAT IMMUNOSUPRESSION DURING RENAL TRANSPLANT WOULD EXACERBATE MALIGNANCY RATHER THAN SUPPRESS IT.....

GH Stmulated by glucose infusion.........FALSE.... Increased in sleep ......T Opposes insulin action F Causes positive nitrogen balance F Decreased in pregnancy F GH STIM BY LOW ENERGY CONDN HYPOGLYCaenia EXCERCISE FASTING

Page 209: 2005-2010(March) Mrcog Part 1 Pastpapers Recall Mrcog Total Guide

HIGH AMINO ACIDS HIGH PROT MEAL ARGININE INFUSION STRESS STIM GLUCAGO SLEEP

GH INHIB BY GLUCOSE CORTISOL FFA MEDROXYPROGEST GH

ANABOLIC HORMONE......WANTS TO BULD AND CONSTRUCT.....SO I THINK N2 BALANCE WOULD BE NEGATIVE...... AND SHOULD BE HIGH IN PREG I THINK......

Foll form boundaries of ovarian fossa Ureter T LATERAL TO OF Ext iliac vn T ABOVE OF Int iliac art T ABOVE THEN POST TO OF Int pudendal art F.....WAY OFF BELOW THE LEVATOR ANI....NOWHERE CLOSE... Obliterated umb art..T LAT TO OVARY..

Page 210: 2005-2010(March) Mrcog Part 1 Pastpapers Recall Mrcog Total Guide

Penem antibiotics Active in beta lactam ring T Drug of choice for ps aeroginoa T

Vagina Derived from mullerian ducts T Seperated from anal canal by anal body....PINEAL BODY-T

Bladder Pelvic splanchnic stimulation contracts the trigone F......ALL TRUE....BUT NOTHING CONTRACTS THE TRIGONE! Ext urethral sphincter innervated by pudendal nv....F....NO PUDENDAL INVLVMT IN URETHERA......SUPP BY SOMATIC FIBS FROM S2-4 Mucosa of urethra is innervated by pudendal nv F Transaction of spinal cord above S2 causes autonomous bladder T I THINK........BECAUSE IT WOULD STILL WORK AUTONOMOUSLY THROUGH REFLEX.......BUT NO SOMATIC ACTION BECAUSE CEREBRAL CONTROL IS LOST.... Afferent innervation of glans of clitoris is by ilioinguinal nv ........F AFFERRENT OF GLANS CLIT IS THROUGH PELVIC SPLANCH......EFF IS THRU PUDENDAL N......AND BLOOD SUPPLY IS THROUGH INT PUD ARE PUD N ALSO SUPPS INF RECTAL NERVE AND A PERINEAL NERVE....

Hypo k+ is found in Prolonged vomiting T In diabetis ketoacidosis T Addison’s disease F HYPERKALEMIA HERE........CORTISOL IS HYPOKALEMIC.......AND NO CORTISOL IN ADDISONS....

Page 211: 2005-2010(March) Mrcog Part 1 Pastpapers Recall Mrcog Total Guide

Foll are below inguinal ligament Ilio inguinal nv T Lateral branch of genito femoral nv T Subcostal nv F Femoral art T

Inf hypogastric (pelvic) plexus Supplies pain fibres to body of ut T Continues as vesical plexus T Lies at base of braod lig T

Sciatic nv S3&4 contribute towards it F L4 TO S3 Lies behind quadratus femoris F......IT LIES ON IRIFORMIS.....WIT NO MORE USCLES ON TOP....QF IS A VERY DEEP MUSCLE Tibial branch supplies short head of biceps femoris ......FASE......SHORT HEAD IS BY COMMON PERONEAL.....LONG HEAD ISBY TIBIAL..... Injection on the outer & upper quadrant of the buttock may cause damage to the nv ....FLASE........THIS IS THE SAFEST AREA

Page 212: 2005-2010(March) Mrcog Part 1 Pastpapers Recall Mrcog Total Guide

Undescended testis More common in premature than term infants T Commonly assoc with gonadal neoplasia T Asoc with 1% of absent testis More common on left side

Corpus luteum Is major source of relaxin T RELAXIN, INHIBIN AND ACTIVIN Contains endothelial cells T I THINK, IT HAS RICH BLOOD SUPPLY.......SO ENDOTHELIAL CELLS TOO I SUPPOSE... Hysterectomy prolonges the life period F I THINK.....HYSTERECTOMY WOULD HAVE NO EFFECT.......OR IT WOULD DECREASE THE LIFE......NO BOOK CONFIRMATION.... 17 hydroxy prog is the main prog secreted F I THINK.......PROGESTERONE IS THE MAIN steroids prevent recruitment of new follicles T STEROIDS LOWER THE LEVEL OF ESTROGEN RECEPTORS..

fertilisation acrosome fuses with zona pellucida T polar body has same karyotype as ovum T fetilised egg moves slower than unfertilised egg in fallopian tube F UNSURE.....BUT WHY SHOULD IT? inner ell mass lysis deciduas during implantation F INNER CELL MASS DOES NOTHING OUTSIDE.....IT BECOMES THE FETUS.....

histo pathologic examination of tissue of conception can diagnose CIN 3 T

Page 213: 2005-2010(March) Mrcog Part 1 Pastpapers Recall Mrcog Total Guide

Cx incompetence F Ectopic preg T Gest Trophblastic Di T

Ring inversion of Y chromosome found the foll will be done Maternal chromosome is tested F Paternal chromosome is tested T Scan to look for male genitalia T Anomaly scan and amniocentesis done for fetus ?T Do karyotype of all previous children of couple F ALL ARE DEDUCTIONS......BECAUSE Y IS RELATED WITH PATERNAL.....AND ASSOCIATED WITH EXPRESSION OF MALE PHENOTYPE.......PLEASE DISCUSS

Right ureter Below sigmoid coon F LEFT URETER Supplied by vaginal art T Lined throughout by urothelium T IF UROTHELIUM IS ANOTHER NAME FOR TRANSITIONAL EPITH Contain glands F.......BLADDER EPITH IS DEVOID OF GLANDS......I THINK URETERS' IS ALSO THE SAME....

Oral absorption of drug depends More availablibility in ionised form than non ionised F.....BECAUSE I DIDNT FIND IT ANYWHERE THAT IONISED FORM INFLUENCED ORAL ABS.....

Page 214: 2005-2010(March) Mrcog Part 1 Pastpapers Recall Mrcog Total Guide

Increased lipid solubility increases penetration in CNS T Enters intestinal cells by simle diffusion T Protein binding increases excretion F Lipid form increases uptake T

Fetal circulation Portal vn drains in IVC T Ductus arteriosus takes blood to pulm art F AWAY FROM IT R atria and L atria communicate thro’ Foramen Ovale T

Voiding in females Occurs above rate of 20 ml/sec T MAX URINE FLOW RATE =20-40 ML/SEC When vesical press is more than 45 mm H2O T VOID PRESS=45-70 MM Initial contraction of trigone F........TRIGONE DOES NOTHING!!!

Enzyme activity is modified by Dephosphorylation T Allosteric esters T Binding to plasma prot T Proteolytic cleaving of inactive precursors T Gene transcription ?F ALL GUESSES......ANYONE KNOWS THE ANSWERS??

Page 215: 2005-2010(March) Mrcog Part 1 Pastpapers Recall Mrcog Total Guide

Functional cyst occurs in Stein levinthal synd T PID F Multiple sclerosis F Gest tropho di F Rx with clomiphene T

Amniotic fluid Hypotonic compared to fetal plasma ?T Absored in lungs ?T Unrelated to swallowing F Increased until term in fetus F DECS TO TERM Contains more HCO3 compared to fetal blood ?T

Action of complement Increase in permeability T Migration of polymorphs T

Genetics

Page 216: 2005-2010(March) Mrcog Part 1 Pastpapers Recall Mrcog Total Guide

South blot detects DNA F ITS ONLE SEROLOGICAL TEST DNA is exact copy of RNA F DNA is transcribed by RNA polymerase T Ribosomes contain RNA T Transcription occurs at any direction along the chain F ONLY 5' TO 3' DNA replication is called transformation F DNA has transcription restriction enzyme ?F DIDNT FIND ANYTHING LIKE THAT ANYWHERE Histones do not contain DNA T

Insulin Binding to tyrosine receptor kinase increases activity T Causes glucose transporter movement to plasma membrane T Increases glycogen synthetase activity T Increases fat deposition T Increase glucose utilisation by CNS ?T

Heparin T ½ is 1.5 hrs T LMW heparin is administered 4 hrly F 8 TO 12 Given in pregnancy causes intracranial haemorrhage in fetus F DOESNT CROSS PLAC COS ITS TOO BIG Longer duration of action in S/C compared to IV administration T

Foll have imp natural reservoir other than human

Page 217: 2005-2010(March) Mrcog Part 1 Pastpapers Recall Mrcog Total Guide

Listeria F Saolmonella typhimurium F Brucella T FROM DAIRY AIMALS

Crohn’s disease Non caseous granulomatous infection T Forms deep fissures T Forms crypt abscesses T Assoc with mucosal polyps F......PSEUDOPOLYPS Recognised to occur in vulva F......AT THE MOST....A FISTULA MIGHT OPEN I THE VULVA......BUT THATS NOT CROHNS OCCURING IN THE VULVA I THINK.... Assoc with intestinal wall thickening ? T......NOT A PROPER STATEMENT......THERE IS INFLAMMATION.....THICKENING IS A VERY LAYMAN TERM I THINK....

Absorption in kdny Glucose occurs in loop of henle F ACTIVE ABSPN IN PCT WITH AA Water is actively absorbed in proximal conv tubules F OPASSIVE DIFFSN IN PCT WITH NA AND CL

Estrogen 17 beta estradiol is formed by aromatisation of testos T increases LH receptors T secreted by theca cells F GRANULOSA CELLS

Page 218: 2005-2010(March) Mrcog Part 1 Pastpapers Recall Mrcog Total Guide

granulosa cells secrete inhibin F CORP LUT DOES humoral immunity is depressed in pregnancy F I THINK.....IF IT WERE TRUE, RHESUS CONFLICT WOULDNT BE AN ISSUE T & B cells are derived from bone marrow T

teratoma usually benign T can occur is assoc with germ cell tumors T secretes hormones F....NOT USUALLY I THINK

during menstruation platelet and fibrin plug is formed after 1st 20 hrs of bleeding F I THINK.....PLATELET PLUG FORMN TAKES PLACE VERY QUICK AND IN VERY SMALL TRAUMAS..... hemostasis is thro’ estr mediated vaso constriction F hemostasis is thro PG mediated spiral vessel constriction F PG ARE CONSIDERED TO BE THE CAUSE.....BUT I DONT KNOW IF ITS AT THE SPIRAL ARTS OR BELOW THEM.......BECAUSE THE APIRAL ARTS ARE ACTUALLY SHED....ISNT IT...

During ovulation Basal vacuoles occur in mid secretory phase T Gland stromal mitosis maximum in mid secretory phase T PG in follicle fluid during ovulation F DIDNT FIND THAT ANYWHERE

Page 219: 2005-2010(March) Mrcog Part 1 Pastpapers Recall Mrcog Total Guide

Increase of estr and prog receptors in proliferaive phase T I THINK LDL receptors are increased in proliferative phase ?F Indomethacin inhibits ovulation F Matrixmetalloproteinases(MMC) is inhibited by progestrones ???!!! WOW NO CLUE Cloiphene can cause deficient secretory phase F ITS ANTI ESTROGEN......SO IT MAY SHORTEN PROLIF PHASE....

Glycogen Synthesis occurs in muscle T Is stored along with water F ITS POLYMERISED......AND TURNS TO CRYSTALS ON STORING.....THIS IS THE REASON THERE IS NO OSMOTIC HAVOC IN CELLS WITH IT

Carbohydrates Yield energy ?17 J/Kg T Major source of energy q=brain &s=0">brain </a> F I THINK ITS GLUCOSE ACTULLY..... Major component of diet T Higher energy compared with fats F FATS HAVE MAX ENERGY

Foll decrease the efficacy of OCP’s Isoniazid F Carbamaepine T Phenytoin T

Page 220: 2005-2010(March) Mrcog Part 1 Pastpapers Recall Mrcog Total Guide

Rifampicin T ALSO PHENOBARB

Female pelvis compared to male Sacrum more curved F LESS CURVED AND MORE MORE BACK TILT Greater sc notch is larger T Sub pubic angle is more T Distance betw pubic sym to acetabulum is less than the diam of acetabulum F......ACETABULUM IS VERY SMALL...AND THE DISTANCE IS GREATER